2ちゃんねる ■掲示板に戻る■ 全部 1- 最新50    

■ このスレッドは過去ログ倉庫に格納されています

【数セミ】エレガントな解答をもとむ3【2018.10】

1 :132人目の素数さん:2018/09/17(月) 01:40:43.02 ID:iDwWzM3i.net
締切りの過ぎた問題をみんなで議論しましょう。

過去スレ:
1. http://rio2016.5ch.net/test/read.cgi/math/1295154182/
2. http://rio2016.5ch.net/test/read.cgi/math/1476702312/

2 :132人目の素数さん:2018/09/17(月) 01:45:59.97 ID:iDwWzM3i.net
          |:;:;:;:;:;:;:;:;:;:;:;:;:;:i;:;:;:;:;:;:;:;:;:;:;:;|      解 今     |
            |;:;:_:;:_:;:_:;:_;:;_:;:l:;_;:_:;:_:;:_:;:_;:|      か  日   |
         |______|_____|      な        l
         |::::| ```````´´´´ : : : :|      か       !
         |::::|  く三)   (三シ : : :|'.      っ      /
          r=Y:f            :.ヘ',     た     /
          |fト|:| tーt:テミヽ .ィチt:ァ‐r : |∧    ら    /
          {{ ー:l  ` ̄ '´   |::.ヾ ̄´ . : :|_,ハ、      /
          ヾ ー!         |::、   : : j_/ >ー‐一'´
          `¨',       -、_;:-    : :/
              l'.    ,r===== 、   :/h
.  ,. -―‐- <. /!:.ヽ  ヾ====='′./: j l\
/          \|: : . \.`""""´/ : / .|:.:.:.:',、
            \   ` ー一'´:   :/  |:.:.:.:.:',`:.ー- 、
                 ヽ     }: : ./  .|:.:.:.:.:.:',:.:.:.:.:.:.:.:.:`:.ー- 、
               '.ー-   j: :/     |:.:.:.:.:.:.:',:.:.:.:.:.:.:.:.:.:.:.:.:.:.:.:.`
  必    も    明.   '.     入    |:.:.:.:.:.:.:.:',:.:.:.:.:.:.:.:.:.:.:.:.:.:.:.:.:
  要   っ    日     ',   / / \   !\:.:.:/.:.:.:.:.:.:.:.:.:.:.:.:.:.:.:.:.:
  に   と   は     i  j./   \|:.:.:.`:ー‐┐:.:.:.:.:.:.:.:.:.:.:.:.:.:
  な    大            |   j′     .!:.:.:
  る   き
  ぞ   な
        努
        力
        が

3 :132人目の素数さん:2018/09/17(月) 01:47:31.23 ID:T7a194so.net
削除依頼を出しました

4 :132人目の素数さん:2018/09/22(土) 20:45:35.98 ID:ztZsZpyH.net
みんなで答え出し合おうよ?

5 :いえやす:2018/09/23(日) 08:20:49.48 ID:n07erhZD.net
締切まで待とう時鳥(ほととぎす)

6 :とあるエレ解常連:2018/09/23(日) 21:38:44.11 ID:5AC0P/Sg.net
斥候部隊の難易度情報は欲しいところです

7 :132人目の素数さん:2018/09/24(月) 10:14:10.44 ID:PKrwWiIA.net
エレガントな解答なんて
無かったんや(迷言)

8 :132人目の素数さん:2018/09/24(月) 11:48:55.01 ID:C29H7b6e.net
出題1
 k=1 のときは見覚えのある関数だが、k≧2 は?

出題2
 ・λ∈R の中で面倒な計算をゴリゴリ敢行するか、
 ・Cまで広げてギリシャの幾何学をする

9 :とあるエレ解常連:2018/09/29(土) 08:24:23.84 ID:VpmYSTVe.net
>>8
ギリシャの幾何学。。
何もぼかせていない気が
だめですよーヒント過多はw

10 :132人目の素数さん:2018/10/02(火) 11:30:20.71 ID:UtBJrYlY.net
https://www.web-nippyo.jp/elegant/
びっくりした。問題が公開された上に、webでの解答も可になった。

11 :132人目の素数さん:2018/10/02(火) 11:55:37.97 ID:a4X/BSXV.net
PDF ファイルを送信かよ

で、送信した解答は公開されるのか???

12 :とあるエレ解常連:2018/10/08(月) 19:21:43.49 ID:DEaqau3L.net
消印締め切りまであと5時間弱です。
時間はまだたっぷりあります。ネバーギブアップで頑張っていただきたい。
要所で粘れるか、それとも簡単に諦めてしまうか、生き様が試されます。
でも今月2問を5時間で解くのははっきり言ってつらい。
私は時弘先生の大ファンですが、彼の問題は秋の行楽シーズンにはそぐわない。
他に誘惑のないじとじとする梅雨の時期、またはコタツがぬくい極寒の時期にじっくり楽しみたいものです。

>>10-11
初回はなんとなく手違いが起こりそうでpdf送信は回避しました。
答案チェックが大変になりそうですが、大丈夫なんでしょうか。

13 :132人目の素数さん:2018/10/09(火) 16:31:35.34 ID:jtiWu+AA.net
10月号

■出題2

{z, w} を固定したとき lim(λ→±∞) |λ-z|/|λ-w| = 1
∴ F(z, w) ≧ 1,
λは {z,w} の垂直2等分線上またはw側にある。

(1) x=u のとき
垂直2等分線は実軸Rに平行
題意より |v| >> |y| としてよい。
|λ-z|/|λ-w| < 1  (λ=x=u で最小)
F(z, w) = 1,
lim(|v|→∞) F(z, w) = 1

(2) x≠u のとき
垂直2等分線は実軸Rと交わる。
交点は λo = (x+u)/2 + (vv-yy)/(2(u-x)), 
もし最大点λがあれば(*)、λoよりもw寄り(遠方)にある。
|λ| > |λo| = O(|v|^2)
w = u + iv,
z は定数  だから
F(z, w) = |λ-z|/|λ-w| = 1+O(1/|v|)
lim(|v|→∞) F(z, w) = 1,

ところで、最大点λは存在するんだろうか?

14 :132人目の素数さん:2018/10/09(火) 17:01:52.52 ID:jtiWu+AA.net
>>13

> 最大点λは存在するんだろうか?

複素数ν = λ + iμ ∈C に対して
|ν-z|/|ν-w| = f の軌跡を考える。(f≧1)

f=∞ では1点w であるが、fが有限のときは wを内包する閉曲線となる。
fを減らせば閉曲線は膨らみ、f=1 では {z, w} の垂直2等分線となる。

(2) の場合、実軸Rと垂直2等分線は交わる。
fを∞から減らして行き、初めてRに接した点がλである。

ところで、λは1つしかないか?

15 :132人目の素数さん:2018/10/09(火) 17:10:07.13 ID:jtiWu+AA.net
>>14

> λは1つしかないか?

ν∈C と f≧1 に対して
 g(ν, f) = f^2|ν-w|^2 - |ν-z|^2 とおく。{z, w} は定点。

 g(ν, f) < 0 ⇔ νは閉曲線の内部
 g(ν, f) > 0 ⇔ νは閉曲線の外部
 g(ν, f) = 0 ⇔ νは閉曲線の周上

 g((ν1+ν2)/2, f) = {g(ν1, f) + g(ν2, f)}/2 -(ff-1)|(ν1-ν2)/2|^2 ≦ {g(ν1, f) + g(ν2, f)}/2,
より
 g(ν1, f) = g(ν2, f) =0 ⇒ g((ν1+ν2)/2, f) ≦ 0,

周上の2点 ν1, ν2 の中点は閉曲線の内部にあるので、閉曲線は凸である。
∴ 実軸Rに初めて接する点λはたた一つ。

こうしてギリシャの幾何学は回避された。ペキン原人やクロマニオン人も解けたかも(?)

16 :132人目の素数さん:2018/10/10(水) 01:20:47.39 ID:MSgGrJGx.net
nを一度だけ使ってってどういうことだ
a_nでもいいのか

17 :132人目の素数さん:2018/10/10(水) 07:33:10.07 ID:5PNYRsCB.net
はい、次の問題もう出てますよー。
https://www.web-nippyo.jp/elegant/

18 :132人目の素数さん:2018/10/10(水) 18:17:30.30 ID:yAzQ854o.net
ハヤイノー
電子時代ジャノー
楕円暗号理論の関連した問題はナイノカノー
格子理論はアッタガノー有限体の楕円曲線理論は美しいのーー
仕事なんかなってられんのーー
こんな美しい世界はナイノー
虚数除法も素晴らしいのーー
楕円曲線と格子理論がクロスするとこの世のものとは思えんのーー

19 :132人目の素数さん:2018/10/10(水) 18:45:51.23 ID:yAzQ854o.net
そうじゃ!!
ディオファントス近似絡みの問題も楽しみにしちょります。
楕円曲線、ディオファン、ガロア、格子絡み、仕事なんかおっぽりだしたくなるべ。

20 :132人目の素数さん:2018/10/10(水) 18:47:13.77 ID:yAzQ854o.net
そうじゃ、わたしが、サボリーマンかず太郎じゃあ。
仕事さぼって、スタバでエレカと格闘じゃ。
文句あっか!!!

21 :132人目の素数さん:2018/10/11(木) 06:12:42.93 ID:dLv4Cwo1.net
出題2は「式より明らか」じゃいかんのかw

22 :132人目の素数さん:2018/10/11(木) 22:12:48.85 ID:OWsrvrcm.net
エレカ、電子公開、goodです。
高校、大学時代愛読してたが、20年前に一度再会、またまた再会。
一生縁が切れそうにないわ。
今は、数論、楕円曲線、暗号にハマってる。
ICMレポも楽しみにしてまっせ。

23 :132人目の素数さん:2018/10/12(金) 00:40:57.27 ID:KGakS2GY.net
大学への数学の宿題もやりたいが、私の頭ではそこまでやると本業に支障がでそう。
自制しよう。
エレカ、数論、楕円曲線、暗号でこっそり楽しもう。
森北の暗号理論と楕円曲線も理解するのは大変だわさ。

24 :132人目の素数さん:2018/10/12(金) 00:43:44.21 ID:KGakS2GY.net
先程、保管してある、1980年代〜1990年代の数セミ、Bacic数学、大学への数学
ひもといてみた。なつかしいのーー。
数セミとも再再会できたし。

25 :132人目の素数さん:2018/10/12(金) 06:19:57.61 ID:YLCrdRFe.net
大学への数学の宿題とエレ解って、どちらの方が難しいの?

26 :132人目の素数さん:2018/10/12(金) 09:55:34.52 ID:0244Q6MN.net
>>21
つーか問題1は一発ネタのような気がするが。
小谷善行先生は不調なのかな?
それとも「存在しない」を証明するのが難しいのか?

「遠山 啓先生に敬意を表して、それっぽく説く」
というのもアリな気がするが。

27 :132人目の素数さん:2018/10/12(金) 13:10:52.79 ID:0244Q6MN.net
>>25
「荒らしに構うのも荒らし」とは云われるが、
いちおう言っとこう。
受験数学より真剣勝負の数学の研究のほうが大変なので、
傾向としては
『エレガントな解答を求む』>『大学への数学』の宿題
だが、エレ解も「いちおう解答は用意されている」し、
たまに「小手試し」的な出題もあるので、
予備校生にとってはエレ解のほうが手ごわいと思う。
まぁ、「お子ちゃま には、エレ解の味はわかんないよな(笑)」
みたいな優越感はあると思われ。

28 :132人目の素数さん:2018/10/12(金) 13:16:43.95 ID:0244Q6MN.net
ようやく題意がわかったように思う。
「示せ」っつーところに重点があるような気がするので
(「それは、数学的に厳密な証明なのか!?」とか
言われると、数学の博士号を取得していないと
解答できなくなる)、「一般的に納得されやすい証明」と
いうことになろうかと思う。

29 :Mr.Moto:2018/10/12(金) 17:22:58.62 ID:0244Q6MN.net
二〇一八年十一月号の問題1について。

四辺形以上の多角形(=多辺形)については、
凸でない場合があって、「凸多辺形」「凸多角形」という
言葉がある。それはいい。線形計画法なんかでは、
「多凸性」というのは、重要な概念だからな。
同時に、四辺形以上の多辺形については、
「辺が交差する」場合がありうる。このとき、
そういった多辺形を、なんと呼ぶべきか?
たとえばの話、五芒星や六芒星を「多辺形」と
認めるのかどうか?
また、その場合、「面積」について、二つの定義があるうる。
どちらを採用するかについて、「数学的な呼び名」、
あるいは定義というのはあるのだろうか。また、それぞれについて、
「数学的な一般的名称」というものがあるんだろうか。
あるとすれば、それは何に準拠しているのか。

そのあたりの意見を聞きたい。

30 :132人目の素数さん:2018/10/12(金) 17:49:29.41 ID:PsrGl5PI.net
エレカ、柏原先生、森重文先生、辻雄先生、神保先生etcも昔解答されておられます。

31 :132人目の素数さん:2018/10/12(金) 23:48:09.81 ID:0244Q6MN.net
>>30
> 森重文先生、
まじか。出典キボンヌ。

32 :132人目の素数さん:2018/10/13(土) 11:31:09.88 ID:m2VlRIRs.net
森重文先生は、京大時代に解答されておられる。エレガントな解答を求む問題集1集or2集
探してみてください。柏原先生、神保先生、辻先生は3集に、名前残してる。皆数学少年じゃったのじゃよ。

33 :132人目の素数さん:2018/10/13(土) 11:32:13.28 ID:m2VlRIRs.net
やっぱ、NOTE掲載を目指そう。
有限体上の楕円曲線研究で。

34 :132人目の素数さん:2018/10/13(土) 11:33:15.57 ID:G0GgdGfh.net
2018年10月号の講評です:

■出題1:レベル6〜7(常連正解率60〜80%)
※スツルムの定理不使用の場合レベル10(正解者0〜2名)

時弘男塾長の出題。例年通り題材は力学系。
正整数kに対し、P_0=1, P_1=x−1, P_n=x^k * P_{n−1}−P_{n−2}でP_n(x)を定め、
小問(1)x>0、(2)kを奇数としてx<0、(3)kを偶数としてx<−1における零点の個数を求める問題。

彼の問題はいつも難しく2015年は正解者たったの2名(昨年はめずらしく易しかったが)。
今回も『どうやって解くんじゃい…』とまず途方に暮れるところから始まる。

が、本問はスツルムの定理を使えば比較的簡単に解ける。
使わずに解くことを要求しているとすれば難易度はレベル10に跳ね上がる。


■出題2:レベル5(常連正解率95%)

加古先生の出題。
複素数z, w, 実数λに対してF=sup|λ−z|/|λ−w|(λ∈R)と定めたとき、
Im(w)の絶対値を∞に飛ばすとFはどうなるか?という問題。

・各複素数の実部虚部を適切に限定して、問題をほぐす。
・素直にFの挙動を調べて極限を考える
という正攻法で解けるので難易度は高くない。

エレガントに解くことを至上命題と考えている猛者は腕の見せ所。
いろんな考え方がありそうです(既に本スレでも示されていますが)。

35 :132人目の素数さん:2018/10/13(土) 15:09:38.13 ID:O/TKH3xV.net
高木寛通先生は、大学への数学の宿題解答者だったみたいだ。
数学少年?

36 :132人目の素数さん:2018/10/13(土) 15:12:12.90 ID:IDEhZSRY.net
>>32
> 皆数学少年じゃったのじゃよ。
そういえば広中先生も京大なんだよなぁ ……
SSS(新数学者集団)とかの話とか、なんか載ってる本とかない?

自分は東日本なんだけど、遠山さんが東大中退だったりするので、
いまいち「東京」って、狭苦しい感じがある。京都って、数学に
向いてる土地柄なのかもしれない、と思って興味があるんだよ。

37 :132人目の素数さん:2018/10/13(土) 15:14:48.59 ID:IDEhZSRY.net
>>35
そういえば阿部寛も数学少年だったらしいぞ(笑)

38 :132人目の素数さん:2018/10/13(土) 16:58:20.46 ID:83u0ux+X.net
ロシアの謎の高校生向け数学物理雑誌、Kvant、レベル高いで。
将来の数学者発掘が目的。ドリンフェルトも愛読???

39 :132人目の素数さん:2018/10/13(土) 18:03:14.86 ID:IDEhZSRY.net
>>38
いや、面白いのは分かるし、たぶん問題の意味もわかるだろうと
思う(数学は世界の共通言語だ)んだけど、
ロシア語とかハンガリー語とかで解答を書けって言われたら、
ちょっと退く部分はある。
まぁ、「やれ」と言われりゃ やらんでもないけど、
なんかしらコンピュータ言語とかに落としこんで
貰えれば、そのほうが楽なような気がする(笑)。

40 :とあるエレ解常連:2018/10/14(日) 10:21:10.93 ID:o5QJ7I6t.net
コテを付け忘れてた。コテなんて要らない気もするが
昔コテ付け忘れるなハゲとか罵倒されたので泣く泣く付けた

11月号はいったいどうしたことだ
出題1は文字通り30秒、頭の中だけで解ける難易度
出題2は一昔前の高校教科書の例題レベル
はっきり言おう、糞であると。

今月はp.40の数オリを解いて楽しめということか・・
というわけで数オリを解いているのですが、どれもこれもエレガントな良問ばかりですばらしい
解答もエレガントさ(センスの良さ)を要求しているところがまたすばらしい
易しくてもエレ解難易度6以上といったところ

数オリ開催日にオーバーエイジ枠を同時開催してくれたらいいのに
エレ解常連がチームを組んで戦うのです。萌えますね

41 :132人目の素数さん:2018/10/14(日) 11:59:11.33 ID:ajXbw4zm.net
エレカも総問題数、1300問になる。
たまには、一休みも必要です。
末永く継続してもらわんといかんからな。
数学者の皆様。

42 :132人目の素数さん:2018/10/14(日) 12:04:03.84 ID:ajXbw4zm.net
フィールズメダリスト、ショルツ先生、IMO金メダリストらしいな。
その他のフィールズ受賞者、ネバリンナ受賞者はどうよ。
佐藤ーテイト予想のテイラーもイギリス代表らしい
量子素因数分解のショアもアメリカ代表らしい

43 :132人目の素数さん:2018/10/14(日) 12:56:31.65 ID:ZGDKsTF8.net
IMOはレベル高いのおーーー
谷山ー志村予想のテイラー先生も出場者だったのか。
ピーター ショアもそうらしい。

44 :学術:2018/10/14(日) 13:12:43.13 ID:yXYU+8iI.net
数学は実戦的だから、スポーツボケを、排除できる司令官に向くだろう。

45 :学術:2018/10/14(日) 13:13:20.37 ID:yXYU+8iI.net
賞をとれることは、戦果として賞金をもらえることだから。結果がよろしい。

46 :学術:2018/10/14(日) 13:13:58.01 ID:yXYU+8iI.net
賞やメダルが、主体的じゃない生き方がおすすめです。若い人は特に。

47 :Mr.Moto:2018/10/14(日) 21:20:04.78 ID:wzFFiQRe.net
>>40
> はっきり言おう、糞であると。
いや、問題2はともかくも、
問題1は けっこう深いぞ?
あれ、じつは構文解析のアルゴリズムの
計算量とかと関連してくる。
単純に解だけ示すんなら、確かに
「文字通り30秒、頭の中だけで解ける難易度」では
あるのだが、「だったら、なんで自然数限定なのか?」
(べつに実数まで拡張しても問題はあるまい?)とか
「点で接している複数の N 角形を認めるかどうか?
その場合の『面積』を、どう定義するか?」とか、
考えどころは ありそうな気はする。それを考えた上で
「エレガントな解答」を出そうと思うと、
それほど簡単な話ではなかろうと思うのだが、どうか。

48 :132人目の素数さん:2018/10/14(日) 21:37:27.35 ID:Omb1GwAa.net
>>47
こんばんは。返信どうも
自分はあんまり研究とか発展とか考えずに喋っております
研究肌の方のコメントは深みがありますな

> (べつに実数まで拡張しても問題はあるまい?)とか

Motoさんだったらどのように実数へ拡張しますか?

> 点で接している複数の N 角形を認めるかどうか?
> その場合の『面積』を、どう定義するか?」とか、

面白いですね。それは考えなかったです
この問題を特にあたっては心配無用なので・・

> それを考えた上で
> 「エレガントな解答」を出そうと思うと、
> それほど簡単な話ではなかろうと思うのだが、どうか。

なんか30秒でそれなりにエレガントな解答ができちゃったので、もう今月は羽を伸ばして紅葉にでも行こうかとw
でもそれじゃ知的満足が得られないってんで数オリに手をだしたら問題が美しすぎてうっとり
エレ解もこうならんかなと

49 :132人目の素数さん:2018/10/15(月) 08:33:06.32 ID:L9PKARy1.net
>>48
> どのように実数へ拡張しますか?
いや、これは単なる例えとして言ってみただけで、
やっても面白くないと思う。
むしろ、出題者の小谷 善行さんは情報工学がご専門なので
有限組合せ問題として考えるのが本筋かな、と。
たとえば、任意の長方形 m × n があったとして、
それに内接する N 角形があったときの最大の N は
いくつかとか、その場合の面積はいくつかとか、
そのあたりの考え方の問題は面白いんじゃないかな、と。
ただ、それをやるとコンピュータによる力業になって
しまいそうなので、エレ解の趣旨から外れる。
問題自体は、もともとプログラミングのほうで
チェッカーボードを使ったパズルがあり、
それがパリティを利用しているので、
「あぁ、それだな」と思った。
それで、ちょっと発展させて、フラクタル図形
とかに向かう方向で、面白い性質が出てくるんじゃ
ないかな、と。あるいは、「1 × 1 の正方形を
辺で接続したときに、頂点の個数と面積と図形の
関係を考える」とか。

50 :Mr.Moto:2018/10/15(月) 09:22:05.84 ID:L9PKARy1.net
すまん。大事な条件を見落としていた。
問題1は「辺の長さが順に 1, 2, 3, … N」なんだな。
見た感じ、「魔円陣」(完全ゴロム環)の
バリエーションみたいな感じだ。
例示された図は
4+6=8+2=10
5+3=7+1=8
だ。Nが4以上の偶数だというのは確定だが、
N=4では解がないのので6以上。
で、8のときに解があるのも例で示されている。
エレガントに解こうとすると、けっこう手ごわいぞ、
これ。プログラム書いて数値実験で追っかけまわして
法則性を割り出す、とかやんないとダメかな?

51 :Mr.Moto:2018/10/15(月) 15:23:24.03 ID:L9PKARy1.net
>>40
> 出題1は文字通り30秒、頭の中だけで解ける難易度
お互い恥をかいたので許そう(-_-!)。
早とちりはイカン、っちゅーこっちゃね。

52 :Mr.Moto:2018/10/15(月) 15:41:36.89 ID:L9PKARy1.net
>>33
「岡、NOTE をねらえ!」

判定の場(court。法廷。「テニスコート」の「コート」も
同義)では 誰でも 独り 独りきり
私の(数学への)愛も 私の苦しみも
数セミ読者しか わかってくれない

続きは誰か書いてくれ。

53 :Mb:2018/10/15(月) 15:56:09.92 ID:L9PKARy1.net
言っとくけど、うちの馬鹿(Mr.Moto)は三味線弾いてるっつーか、
「ヒントとか出しているようで、じつは引っ掛け」とかだから、
信用しないように。
「もっと困れ」(by 横井 庄一)じゃないけど、
「もっと苦しめ」という助言も上のほうから あったので、
せいぜい苦しんでくれ。

54 :学術:2018/10/15(月) 16:03:14.62 ID:8DQ7ySxz.net
エレク 彼岸

55 :132人目の素数さん:2018/10/16(火) 01:15:32.58 ID:5DYkLdwz.net
出題1
 解があるNはすぐ絞れますが、そのあとが問題ですねぇ…

>>53
「横井庄一のサバイバル極意書 〜 もっと困れ!」小学館 Be-pal books (1984/Jan)
 181p.絶版

56 :132人目の素数さん:2018/10/16(火) 06:12:26.81 ID:1kFGegUv.net
偶数ということくらいしかわからん(;´д`)

57 :132人目の素数さん:2018/10/16(火) 08:36:55.58 ID:94QUq9QN.net
>>56
いや、奇数でないことを示すのが
意外に手ごわいっちゅーかキモっちゅーか。
合計が N (N + 1) / 2 なんで、「N または N + 1 の
どっちかが8の倍数」が必要条件なのは分かるが、
たとえば N = 7 のときの解がないことを
エレガントに示すのに手こずっている。
もう一点は、N が8の倍数のときに、必ず
N 角形が存在することの証明がどう示せるかだ。
16角形とかいうと、「後戻り」の可能性もあれば
辺が途中で交差してしまうケースも排除せんと
いかんだろうし。

58 :132人目の素数さん:2018/10/16(火) 09:43:33.12 ID:5DYkLdwz.net
>>57
「N角形になる」のであれば N個の頂点で曲がっているはず。(本問では垂直に)

59 :132人目の素数さん:2018/10/16(火) 10:36:10.31 ID:5oX8F2v0.net
うむ。始点と終点のつながりを考えると、奇数だと明らかに不可。

60 :Mr.Moto:2018/10/16(火) 18:02:53.66 ID:94QUq9QN.net
>>59
うむ。確かに「明らか」なんだが、
それを「中学生にもわかる」ように
「示す」のがエレガンスだと思う。
数学マニアにとっての「明らか」さは、
必ずしも数学初心者にとっての
「明らか」さではないところが
悩みどころ。
整数論(というか、自然数論)の範囲内だと、
「古代バビロニア人にも説明できる」くらいまで
ガッチリ組まないと いかんと思う。

61 :132人目の素数さん:2018/10/16(火) 18:07:52.19 ID:5DYkLdwz.net
・水平な辺
Σ→ + Σ← = 1 + 3 + … + (N-1) = NN/4,
Σ→ = Σ←     (…閉じる)
∴ 8 | NN
∴ 4 | N
∴ N+2 ≡ 2 (mod 4)


・鉛直な辺
Σ↑ + Σ↓ = 2 + 4 + … + N = N(N+2)/4,
Σ↑ = Σ↓ = (偶数)    (…閉じる)
∴ 16 | N(N+2)
∴ 8 | N

で、この後どうするか。 >>58

62 :132人目の素数さん:2018/10/16(火) 18:31:14.99 ID:5jr9jBpY.net
いいのかな?これくらいなら。
とりあえずN=8,12,16,…くらいで実際できるかできないかやってみるこってすな。
ヒントあるし。
しかし水平は偶数、垂直は奇数???

63 :132人目の素数さん:2018/10/16(火) 20:34:53.15 ID:DWPvJqId.net
>>60
えっ、各頂点の角度が90度または270度の多角形が偶数角形なの、そんなに証明が難しいか?

64 :132人目の素数さん:2018/10/16(火) 22:36:54.71 ID:p/yzOhrf.net
>>61
警告!あなた書き過ぎです

65 :Mr.Moto:2018/10/16(火) 22:41:23.29 ID:94QUq9QN.net
>>63
> そんなに証明が難しいか?
「エレガントな解答をもとむ」なんだから、
エレガンスを追求しろと言っている!
『数学セミナー』の創刊者は
遠山 啓先生なんだから、「テープ算」みたいな
「中学・高校生にも直観的に理解できるような
シェーマを提示する」っつーのが、
重要なんだよ。エレ解は、「正解すりゃあ入選」みたいな
甘いモンじゃねぇんだぞ?

66 :Mb:2018/10/16(火) 23:08:53.44 ID:94QUq9QN.net
>>64
激しく同意
むしろ引っ掛けとか
誤誘導のほうが
マシ

67 :132人目の素数さん:2018/10/17(水) 00:18:18.38 ID:CNsWZSmr.net
おはようフェルプス君。
さて、今回の任務は競泳の世界大会で金メダルを取ること…ぢゃなくて、数セミのエレ解問題の正解を応募することだが…
例によって、君あるいは君の仲間達がここのレスを信じて正解を逃したとしても当局は一切関知しないので、そのつもりで。
なお、このレスは自動的には消滅しない。
健闘を祈る。

(かなり古い…)

http://www.youtube.com/watch?v=erUcduVIt2A
http://www.youtube.com/watch?v=8WnNG2XtU8s
http://www.youtube.com/watch?v=TDQ1vwoINo0
http://www.youtube.com/watch?v=4dJ5MVmpjmw

68 :132人目の素数さん:2018/10/17(水) 09:20:07.50 ID:hNlWCQz1.net
>>65
ちょっと何言ってるか分からないです

69 :132人目の素数さん:2018/10/17(水) 09:34:19.98 ID:rmbMwrXm.net
富澤キタ

70 :Mr.Moto:2018/10/17(水) 17:35:36.08 ID:4+/efYTS.net
>>67
安心しろ。『スパイ大作戦』を知らない奴は
多いだろうが、『ミッション・インポッシブル』は
映画でシリーズ続行中だ。
つーか、朝方なにげなく TV をつけてると、
クインシー・ジョーンズの『アイアンサイド』とか
『ミッション・インポッシブル』が、BGM として
流れていたりするので「変わんねぇなぁ ……」と
思う。
まぁ、エレ解スレでするネタじゃねぇとは思うが。

71 :132人目の素数さん:2018/10/18(木) 11:04:10.73 ID:ajxP7QxG.net
1月号のICMレポが楽しみじゃあ。
各種賞受賞者の数オリ出場歴もたのみまっせ!!

72 :132人目の素数さん:2018/10/19(金) 13:14:52.53 ID:yqDpqUXJ.net
以前「幻の0番法」という記事があったのですが
数列の和についての生地だったと思うのですが
どのような内容だったかご存知のかたいるすか?

73 :132人目の素数さん:2018/10/20(土) 08:02:16.50 ID:OqismSZc.net
>>72
それ、「エレ数」だったかなぁ ……
共立出版の『bit』で出てきた
「生贄」の話のような気がする。
n = 1 から始まる数列に、
n = 0 の項を つけ加えて一般化するとか。
たとえば「1 から n までの和」だったら、
「0 から n までの和」としてもイコールだから、
「n ×(n + 1)」でいいとか。
「(n + 1)× 1」よりエレガントな
感じがしないか?

74 :132人目の素数さん:2018/10/20(土) 08:03:35.49 ID:OqismSZc.net
×「(n + 1)× 1」
〇「(n + 1)× n」
orz

75 :132人目の素数さん:2018/10/20(土) 14:42:36.05 ID:/MrLnf1N.net
>>70
5拍子の曲いいね。

「メリーゴーラウンド」 {大木彩乃:「幻の魚」(1999)}, #2
 http://www.youtube.com/watch?v=0BF4YPjDkZw

「スプリット」{大木彩乃:「屋上遊園地」(2000)}, #10
 
"Wind" {明星: "Stoned Town" (2002)}, #1
"Wind" {Akeboshi: "Akeboshi" (2005)}, #1
 http://www.youtube.com/watch?v=SaaRwKlcNaA

「ウタカタ」 {ジムノペディ:「今宵も、うたかた探し」(2004)}, #2
 http://www.youtube.com/watch?v=W3GnrgBAwg0

「神様の舌打ち」{Akeboshi: "Akeboshi" (2005)}, #13
 http://www.youtube.com/watch?v=U_3jGtk4l6A

雅楽の夜多羅拍子も。楽典とは何ぞや?

スレ違いだが。

76 :132人目の素数さん:2018/10/20(土) 22:45:38.33 ID:OqismSZc.net
>>75
まったくのスレ違いだが、
デイヴ・ブルーベックの
“Take Five” を忘れてはいけない。

77 :132人目の素数さん:2018/10/20(土) 23:56:14.17 ID:/MrLnf1N.net
>>76
スレ違いの極みでござるな。

テイク・ファイヴ {デイヴ・ブルーベック・カルテット:「タイム・アウト」 (1959)}, #3
 http://www.youtube.com/watch?v=966iPSlYj2A

右方舞楽 抜頭(夜多羅拍子) (1943)
 http://www.youtube.com/watch?v=EZc_De09gQo

78 :Mb:2018/10/23(火) 17:09:09.90 ID:t71LiLEA.net
ごめん。今月号(二〇〇八年十一月号)の問題1に関してはパス。
うちの所長が、「パズル懇話会」の会合で小谷先生と二次会で
一緒だったので、結果的にヒントを貰っちゃったので
フェアではない、という話になった。

N ≡ 0 (mod 8) であろうことは、わりと証明が簡単だと
思うけれど(ぶっちゃけ、力業で なんとかなると思う)、
「N ≡ 0 (mod 8) のときに、常に解が存在するか?」を
“エレガント” に証明できるかどうかが問われているような
気がする。

ただし、あくまで「気がする」だけなんで、そこいらは
投稿した結果次第で判断してくれ (-_-)

79 :132人目の素数さん:2018/10/23(火) 19:43:18.34 ID:HKtMWUIJ.net
一応解は作れたけど、ちゃんと(交わらない)多角形になってるのを証明するのが面倒だなぁ。

80 :132人目の素数さん:2018/10/23(火) 22:22:27.01 ID:xS8rsyai.net
必要条件の導出のとこもそうだけど、
十分性のとこも腕の見せ所だねぇ。

81 :132人目の素数さん:2018/10/25(木) 15:29:13.77 ID:c0Urnqy9.net
>>79
「少なくとも、一個以上存在する」のを示せばいいんじゃね?
「魔円陣」みたいに、「あると思ったら解がなかった」
みたいな話を排除すりゃいいんだと思われ。

82 :132人目の素数さん:2018/10/25(木) 17:57:11.07 ID:0sa6guuR.net
解があるNはすぐ絞れますが、そのあとが問題ですねぇ… >>55

83 :132人目の素数さん:2018/10/26(金) 20:16:00.43 ID:zxLKzcfw.net
>>82
そうじゃない。
「解がない」ケースは排除できるんだが、
「解がありそうな」ケースにおいて、
具体的な解が あることを
証明できるか どうかが問題なんだ。

84 :132人目の素数さん:2018/10/28(日) 19:00:03.20 ID:VHGHHXX0.net
この中に大学への数学の宿題もやってる人いる?
また、どちらの方が難しい?

85 :132人目の素数さん:2018/10/31(水) 07:53:43.12 ID:o2qBtZC4.net
宿題コーナー:問題をネット公開してないから、田舎住んでるおいらには情報入手がねっく。
仕事忙しくて、休日も都会に出れない。
しかも、今、免停中。

86 :132人目の素数さん:2018/10/31(水) 11:55:46.85 ID:HbGeox4u.net
ICMレポでは受賞者のIMO履歴レポも楽しみにしとります。

87 :132人目の素数さん:2018/10/31(水) 13:05:45.93 ID:r0MWiigz.net
【え! 総人口250万人減少?】 早く移民で水増しないと、■■■が原因だと、無関心層に気づかれる
http://rosie.5ch.net/test/read.cgi/liveplus/1540952533/l50

88 :132人目の素数さん:2018/10/31(水) 23:33:34.82 ID:MZfORzCN.net
>>84
もちろん 数オリです。

[前スレ.543, 554, 566, 577]

89 :132人目の素数さん:2018/11/01(木) 01:10:42.80 ID:ierp+l9s.net
>>85
> しかも、今、免停中。

なにしたのw

90 :132人目の素数さん:2018/11/01(木) 07:06:15.14 ID:5LSLCiuV.net
人引いたっちゃの

91 :132人目の素数さん:2018/11/03(土) 00:47:10.99 ID:/E6xXixt.net
>>84
 そんなこと訊かれちゃ K&ouml;MaL なぁ。

[前スレ.581, 592, 960-963]

92 :132人目の素数さん:2018/11/04(日) 19:22:34.74 ID:dybDQ925.net
宿題よりもエレ解の方が遥かに難しい

93 :132人目の素数さん:2018/11/08(木) 09:23:16.64 ID:7+cigYji.net
エレガント問題のネット公開→good idea。投稿の有無に関係なく問題挑戦→正解が知りたくなる→数セミ購入→販売促進→末永く継続。
数理科学の最新動向を気軽に入手するには必須の雑誌ですからね。
これからも期待してます。

94 :132人目の素数さん:2018/11/08(木) 10:11:35.27 ID:85Uw/O+I.net
ICM各種賞の受賞者のIMO出場歴には関心あり。
Hilbert 第10問題の解決者もIMO金メダリストらしい。
量子素因数分解のショアもメダリストらしい。
計算機科学とIMO(数論、組み合わせmainテーマ)は相性いいみたい。
今回のショルツの数論幾何、p進理論みたいだし。

95 :132人目の素数さん:2018/11/09(金) 12:04:27.56 ID:pvdoV3Z4.net
11月号

■出題1

N=8n に限ることは容易に分かると思う。

例の図を見て最初に思いつくのは
 1〜2n では ↓,←
 2n+1〜6n では ↑,→
 6n+1〜8n では ↓,←
というものだろう。

曲線 x = y - √|y| 上に
 P_k (-k(k+1),-kk)   k=0〜4n
曲線 x = y + √|y| 上に
 Q_k(-k(k+1),-(k+1)^2)  k=0〜4n-1
を取り、
P_0 - Q_0 - P_1 - Q_1 - … - P_4n を結ぶ。

* これらの曲線は、1本の放物線(軸: y=x-1/4)の2本の枝である。

このままでは閉じないから、P_nで180゚ 折り曲げ、さらに P_3n でも180゚折り曲げよう。
このとき P_n および P_3n の接線を横切るから、N角形は自身と交叉しない。
また P_4n は P_0 と重なる、つまり閉じる。

このN角形の面積は 4(11n+2)n^2
3曲線に囲まれた部分の面積は (4/3)n^3
P_k たちが作る4n角形の面積は (4/3)n^3 + (2/3)n,
らしい。

96 :132人目の素数さん:2018/11/09(金) 12:45:24.20 ID:VQ778wZ3.net
>>95
>
> N=8n に限ることは容易に分かると思う。
>
限らんやろ?

水平 : 1 6 12 と 3 5 11
垂直 : 2 8 10 と 4 7 9

にわけて

→12↑2→6↑8→1↑10←11↓4←5↓7←3↓9

でいけるやん。

97 :132人目の素数さん:2018/11/09(金) 13:02:03.68 ID:/YvwhmDF.net
>>96
辺の長さが順に1〜Nになるって問題文の最初に書いてある。

98 :132人目の素数さん:2018/11/09(金) 13:06:36.41 ID:/YvwhmDF.net
今回はみんな問1に興味があるようだね。俺は解けなかったよ。問2だけ応募した。

99 :132人目の素数さん:2018/11/09(金) 15:44:36.63 ID:u44VxTes.net
>>97
え?辺の長さ1〜12になってるけど?

100 :132人目の素数さん:2018/11/09(金) 15:49:07.93 ID:/YvwhmDF.net
>>99
順に、だよ。
→12↑2→6↑8→1↑10←11↓4←5↓7←3↓9
は順になってない。

101 :132人目の素数さん:2018/11/09(金) 15:53:28.75 ID:u44VxTes.net
>>100
あ、ほんと。失礼しました。

102 :132人目の素数さん:2018/11/09(金) 19:49:16.20 ID:GKcYZvRK.net
実をいうと自分もこの順にというのを見落としていて、だいぶたってから気づいた。
問題文に不備はないけど、これを考慮せずに応募した解答者が何人かは居るような気がする。

103 :132人目の素数さん:2018/11/09(金) 20:12:02.08 ID:GZzg+9lA.net
問題としてはそっちの方が難しくね?

104 :132人目の素数さん:2018/11/09(金) 22:34:17.43 ID:8HhBr9kV.net
12月のエレガントな解答はどうかね。
年末楽しめるぞ。
IMOは格式高いみたいだな。
マチアゼビッチもメダリストなんか。

105 :132人目の素数さん:2018/11/09(金) 22:34:36.85 ID:VQ778wZ3.net
でもまぁ8n+4の場合が増えるだけで自由に並べていいなら十分性のチェックがやや楽になるからなぁ。
どっこいかも。

106 :132人目の素数さん:2018/11/10(土) 01:46:12.25 ID:0LaPCkg7.net
11月号

■出題2
 x = a cosθ + b sinθ,
 y = c cosθ + d sinθ,
 ad-bc ≠ 0,
はxy-平面上の楕円である、を証明する問題

パラメータθを消去するのですが、
 dx-by = (ad-bc)cosθ,
 ay-cx = (ad-bc)sinθ,
からすぐに
 (dx-by)/√(dd+bb) = u,
 (ay-cx)/√(aa+cc) = v,
と置くのは良くない…

θをずらしてから消すのがミソ?

107 :132人目の素数さん:2018/11/10(土) 12:11:14.57 ID:EuCYu9xA.net
>>106
2次曲線は楕円、放物線、双曲線に限るという事を認めてしまえば一瞬だけどそれを証明しなさいというやつなんだよね。
容易”、”自明”で済まされることも高校の教科書レベルで許されるまで下げないといけない。
その前提で楕円、双曲線の定義も “うまく座標をとればAx^2 + By^2 = 1の形になる。”として

x^2+y^2 = 1の像をAx^2+By^2+Cxy=1として
2(Ax^2+By^2+Cxy) = r^2((A-B)cos2θ+ Csin2θ+(A+B))=r^2(Dcos(2θ+α)+E)=2と変形される。
回転させればr^2(cos(2θ)+F)=Gとなる。
変形してr^(2cos^2θ+F-1)=Gだから2x^2 +(F-1)(x^2+y^2)=lであり楕円、直線、又は双曲線となる。
コンパクトなので楕円。

とかでどう?たしか大数はこんな感じの解説だった。

108 :132人目の素数さん:2018/11/10(土) 12:48:56.13 ID:TylIaUHL.net
18年11月号の講評:

■出題1:レベル4(常連正解率98%)

小谷先生の出題。辺の長さが順に1,2,...,NとなるN角形の存在を問う問題。

必要条件N=8nを示すのは容易。
問題は十分条件だが、問題文に描かれている8角形の構成をそのまま一般のnに拡張すればよい。
辺が交差しないことを確認する方法は数通りある。
2点の座標を用いて交差条件を不等式で表すという愚直な方法を採れば、
高校数学でさんざんやったXY平面の領域問題に帰着する。

常連にとっては必要性も十分性も解答方針がすぐにピンと来る易問。
十分条件をエレガントに示す楽しみはあるかもしれない。


■出題2:レベル3(大学1年生の正解率95%以上)

岩I先生の出題。
本問は教科書から書き写してきたような一次変換の問題。
どうして本誌名物コーナーにこの問題を出そうと思ったか、理解に苦しむ。

109 :132人目の素数さん:2018/11/10(土) 13:27:12.74 ID:3qbO+/fB.net
確かにIMOは格式高いの。
マチアゼピッチ、ドリンフェルト、ラフォルク、ペレルマン、リチャードテイラー
ショア、皆、メダリストじゃあ。
最も格式高い、理論計算機、数論はメダリスト多し。
今回のフィールズ、ネバリンナはどうかの?

110 :132人目の素数さん:2018/11/10(土) 14:07:27.76 ID:3qbO+/fB.net
12月のエレガントは週明け公表かの?

111 :132人目の素数さん:2018/11/10(土) 14:07:37.01 ID:3qbO+/fB.net
12月のエレガントは週明け公表かの?

112 :132人目の素数さん:2018/11/10(土) 16:38:01.74 ID:O3Li6cBe.net
束の間の静けさ。
今日は、お伊勢参りしちょります。
週明けから、12月号と格闘。
英気を養うべし。

113 :学術:2018/11/10(土) 17:45:42.08 ID:RS+jJjFq.net
数学と 文学と 天文学あわせて、人文科系に新しい天文数学という分野を立ててみたい。

114 :学術:2018/11/10(土) 17:46:10.00 ID:RS+jJjFq.net
星占いのような迷信も取り込んでね。科学的に。

115 :132人目の素数さん:2018/11/10(土) 18:59:47.30 ID:XUkVeMLH.net
占いや心理学は学問ではない

116 :132人目の素数さん:2018/11/10(土) 20:58:49.48 ID:TylIaUHL.net
来月は京大ガロア祭の問題解説もあるようですね
楽しみです

117 :132人目の素数さん:2018/11/10(土) 23:20:37.94 ID:0LaPCkg7.net
>>106
原点O(0,0) から P(x,y) までの距離の2乗は
 x^2 + y^2 = (a cosθ + b sinθ)^2 + (c cosθ + d sinθ)^2
= (aa+bb+cc+dd)/2 + (aa+cc-bb-dd)/2 cos(2θ) + (ab+cd) sin(2θ)
= D'・cos(2θ+α) + E',

原点Oから最も近い点P_min と最も遠い点P_max は θ が 90°ずれている。
と同時に OPmin ⊥ OPmax も成り立つ。

番外問題 焦点はOPmax の方向にある。

118 :132人目の素数さん:2018/11/10(土) 23:57:09.76 ID:TylIaUHL.net
>>93
>エレガント問題のネット公開→good idea。投稿の有無に関係なく問題挑戦→正解が知りたくなる→数セミ購入→販売促進→末永く継続

正解者として名前が載った回の数セミは買って手元に置いておきたい、という読者心理を巧く掴んでいる気がしますね

119 :132人目の素数さん:2018/11/11(日) 00:01:34.55 ID:1Ei/zP4f.net
12月号はピーターフランクルと岩沢先生の解答号でもあります。楽しみですね

http://rio2016.5ch.net/test/read.cgi/math/1476702312/971

120 :132人目の素数さん:2018/11/11(日) 10:20:35.92 ID:JwUfUe23.net
>>107
それがこの問題の模範解答でしょうね。

個人的には座標変換を記述するのに行列を使うか三角関数を使うかは非常にどうでもよく、従って>>108のような素っ気無いコメントになってしまいました。

たまに大学範囲の簡単な問題に対して「中高生でも分かるような解答」を要求する出題者が居ますが、ああいうのは苦手です。

三角関数を捏ねくり回すよりも、一次変換の基礎を書き下して本質を抉った方がよっぽど中高生のためになるんじゃないかと思ってしまいます。

121 :132人目の素数さん:2018/11/11(日) 10:44:02.52 ID:JwUfUe23.net
とはいえ10月は忙しかったので問題が簡単で助かりました。

12月号が届くまでの間、束の間の雑談。

10月号の時弘先生の問題>>34が中高生の範囲で解けるのかどうかは気になります。スツルムの定理を前提知識として要求しているとは思えませんから。

「中高生の範囲」というのも考え始めると良く分からなくなってきますがね。スツルムの定理を理解するのに必要な知識は高校範囲の微分だけですからね。
一次変換だって少し前は高校でやっていたわけで。行列の知識を使わないで解くのが出題者の狙いだったとして、そのココロは私にはよく分からんですね。

122 :132人目の素数さん:2018/11/11(日) 11:10:08.25 ID:82kqsbVR.net
皆さん、束の間の紅葉見物でしょうか?
福岡はまだですね。
北海道、東北は終了?

123 :132人目の素数さん:2018/11/11(日) 11:37:17.52 ID:JwUfUe23.net
>>122
関東の平地は来月頭にかけてピークですかね。
標高高めのところは今がピークか、終わりかけているところでしょう。
今年は時弘先生が10月号にずれてくれたので旅行の計画に余裕が生まれましたw

124 :132人目の素数さん:2018/11/11(日) 11:46:55.95 ID:JwUfUe23.net
雷山千如寺大悲王院の紅葉
いいですなあ

125 :132人目の素数さん:2018/11/11(日) 12:42:22.93 ID:82kqsbVR.net
時弘先生の重量級は旅行シーズンは外してほしいですね。
私は応募しませんでした。
数の幾何学絡みの哲学的問いかけも含んだ深遠な問題待ってます。

126 :132人目の素数さん:2018/11/11(日) 22:10:27.22 ID:rkLPSQph.net
>>125
まったくです。
岩沢先生も行楽シーズンは駄目です
問題が魅力的かつ難しいので。

時枝先生は問題出さないのかなー

とここを見ているかも知れない編集部の方に向けて呟いてみる

127 :132人目の素数さん:2018/11/12(月) 12:55:00.29 ID:XWeln17t.net
電子版、エレガント。まだ更新されとらんどす。
今月はupが遅れとります。

128 :132人目の素数さん:2018/11/12(月) 14:49:52.79 ID:EHCP4EhY.net
エレ解は難しすぎるから、大学への数学の宿題にチャレンジするわ

129 :132人目の素数さん:2018/11/12(月) 17:49:10.23 ID:w4Fdouwh.net
今月の1は、pを既知として良いか否かで違うが、前者だと3秒で解けるから後者なんだろうな。それでも
解を見つけるだけなら簡単。ただ、最適解を、ということならどうだろう。
2は、ちょっと面白そう。

130 :132人目の素数さん:2018/11/12(月) 18:44:25.77 ID:2tyOcDc0.net
問題見つからない。リンクおながいします。

131 :132人目の素数さん:2018/11/12(月) 18:48:34.20 ID:TR6DezZk.net
>>130
いや、雑誌買ったw

132 :132人目の素数さん:2018/11/12(月) 18:56:44.84 ID:2tyOcDc0.net
残念。もうオンラインではやらないのかな?

133 :132人目の素数さん:2018/11/13(火) 00:11:50.62 ID:umZIwRjH.net
>>128
宿題ですか。なつかしいですね。
問題教えてくれれば解いてみたいですけど。

134 :132人目の素数さん:2018/11/13(火) 13:33:05.68 ID:Iloig4zl.net
来ましたよ
https://www.web-nippyo.jp/elegant/

135 :132人目の素数さん:2018/11/13(火) 15:20:09.06 ID:9YhCG0Hp.net
コインはさておき、ジャンケンで決めるなんてのはどうだろう

136 :132人目の素数さん:2018/11/13(火) 15:45:44.26 ID:FBLhmoMV.net
なんか期待値の最小値で評価するのすごい違和感あるな。
p がなんであっても大体この回数では決着がつく=期待値の最大値で評価したくなるけどな。

137 :132人目の素数さん:2018/11/13(火) 16:53:58.95 ID:/uXd0fJd.net
>問題 2 の答えは,いろいろと考えられるが,上記の期待値の最小値が小さいほど良いものとしよう.

この書き方はもしかして出題者側は持ってる答えの最小性の証明が出来てないのかな?
懸賞問題だから “最小性の証明はなくていい。投稿された答えの中で最小なのが勝ち” なのかな?
Aさんのルールより期待値の最小値が小さいやつはすぐ作れるけど、それが最小である自信は全くないや。

138 :132人目の素数さん:2018/11/13(火) 17:58:02.59 ID:gd93v0n7.net
出題2ってwww
あかん、ネタバレになるから言えんwww

139 :132人目の素数さん:2018/11/13(火) 18:00:45.67 ID:Iloig4zl.net
>>136
p=0またはその極限だと永久に決まらないから、なかなか難しいかと。

140 :132人目の素数さん:2018/11/13(火) 19:07:29.49 ID:/uXd0fJd.net
あ、ホントだ。最小値にせざるを得ないのか。
平均値ってするとpの分布与えないとダメだし。
やむを得ないのか。
まぁ十分ムズイし。

141 :132人目の素数さん:2018/11/13(火) 21:29:36.39 ID:eok3e5c2.net
問題文中にさらっと書いてあるけど、できるだけ公平になるようにするっていうのは暗黙の了解なのかな。
もっとも不公平でいいなら1回で終わるけど。

142 :132人目の素数さん:2018/11/13(火) 22:10:18.01 ID:/uXd0fJd.net
だろうね。
pがいかなる値であっても両者の勝つ確率は常に1/2
は暗黙の了解なんだろね。
受験で問題文にこの事明示してなかったら大問題だけど。
幾分エスパーしないといけない要素あるね。
最小性の証明が全くできる気がしない。orz

143 :132人目の素数さん:2018/11/15(木) 18:00:48.42 ID:l3Lnz9+G.net
出題1、意外に難しい。すぐ思い付いた方法、全然公平じゃなかったわ。

144 :132人目の素数さん:2018/11/15(木) 18:19:50.06 ID:+bxD8ezx.net
出題1→あっさり解けてしまった。もしかしたら、出題意図に反した解答かもしれないが。

出題2→問2、3に悪戦苦闘中。

145 :132人目の素数さん:2018/11/16(金) 01:15:11.21 ID:3fj2avs/.net
え?そんなにあっさりとける?
そのルールで期待値が最小値を取ることもあっさりしめせたの?

146 :132人目の素数さん:2018/11/16(金) 04:33:31.42 ID:VGuNCxDQ.net
そうそう。これ正面から解釈して本当に解ある?って疑い始めてるけど。
裏ワザ的解釈なら、とんでもない解があるが。

147 :132人目の素数さん:2018/11/16(金) 07:04:55.79 ID:xmVvCZPI.net
>>119

9月号出題の解答を貼っとく。

■出題1 

2020 = 2×2×5×101
M = 5×10^226 -20 -3×10^223 -5×10^224
 = 49469・・・(221コ)・・・980, (227ケタ)

 * [前スレ.964] (228ケタ) は間違い。

2019 (素数)
M = 6×10^224 -1 -10^19 -10^223
 = 589・・・(203コ)・・・989・・・(19コ)・・・9, (225ケタ)

2018 = 2×1009
M = 6×10^224 -2 -10^35 -10^204
 = 59・・・(19コ)・・・989・・・(168コ)・・・989・・・(34コ)・・・98, (225ケタ)

■出題2
 ラッキーナンバーが 0,3,5,7 のとき確実作戦が存在する。
 その他 (1,2,4,6,8-14) のときは存在しない。

148 :132人目の素数さん:2018/11/16(金) 09:25:48.69 ID:sF85eyia.net
>>146
そう。裏技を用いると、意外な解が得られる。
果たして、それが許されるのか

149 :132人目の素数さん:2018/11/16(金) 10:03:28.97 ID:3fj2avs/.net
なんかそらあかんやろってやつじゃないの?

150 :132人目の素数さん:2018/11/16(金) 12:32:24.77 ID:3fj2avs/.net
自分なりのエスパー解釈。

表、裏の有限列SでSの通りに出る確率をP(S)、l(S)をその長さとしてそのようなSからなる集合A.Bで条件

S、T∈A∪B、S≠TならP(S∩T)=0、
Σ[S∈A]P(S)= Σ[S∈B]P(S)=1/2

を満たすものの中で

Σ[S∈A∪B]P(S)l(S)

の最小値を求めよ。

最小値があるのか知らんけど。

151 :132人目の素数さん:2018/11/16(金) 17:54:51.21 ID:1MCcWI6o.net
出題1分かったー。いや、これは面白い!
でも、平均回数が解析的に計算できる気がぜんぜんいしないぞ。

152 :132人目の素数さん:2018/11/17(土) 02:52:53.66 ID:4vFfZM9R.net
>>147
2019は素数ではない。
2019=3×673

153 :132人目の素数さん:2018/11/17(土) 22:32:57.24 ID:j3PWNWS8.net
>>150
例えば表裏表で勝負が付いた場合は、それより長い表裏表裏なんかは必ず同じグループに入れるなりカウントしないなどの措置が必要になる。
この辺りが面倒なんだよこの問題。

154 :132人目の素数さん:2018/11/18(日) 00:22:05.25 ID:ZJOO9Eig.net
>>153
一個目の条件でオウオとオウオウのような背反でない事象はとらないようにしてる。
まぁ難しい。
これかな?と思い当たるのはあるんだけど最小性の証明ができてないからなぁ。
まぁできても応募する気なんぞサラサラないからいいんだけど。
しかしおそらく出題者は最小性の証明持ってるだろうし出来ないのなんかムカつくので熟考中。
問題文の文面ではとりあえず答えだけ合ってればいいみたいな事は書いてたけど、おそらく何人かは最小性の証明付きで応募してくるだろうね。

155 :132人目の素数さん:2018/11/19(月) 14:20:51.49 ID:OsmklUOL.net
出題1の(2)って、
最小値1回ってありうるのかな。無論、公平な賭けを前提に。

156 :132人目の素数さん:2018/11/19(月) 14:39:42.36 ID:vaYg27wd.net
それで題意にあった解答になってると思うならそれで投稿してみればよし。

157 :132人目の素数さん:2018/11/19(月) 17:53:22.98 ID:ejOV8r8r.net
>>155
そうならないように問題文で巧みにブロックされてるように見えるが、俺の想像の範囲外なら分からん。
とりあえず応募

158 :132人目の素数さん:2018/11/20(火) 15:04:36.36 ID:vbTkRlXv.net
大学への数学の宿題とエレ解って、どちらの方が難しいの?

159 :132人目の素数さん:2018/11/21(水) 12:54:39.03 ID:2IiwMfBx.net
おまえら、東大理系数学解けるのかよ?

160 :132人目の素数さん:2018/11/21(水) 14:12:40.54 ID:aZSxXt4N.net
え?当たり前
でもスピードは現役に敵わんかも。

161 :132人目の素数さん:2018/11/21(水) 17:54:11.14 ID:LpL6uPAW.net
でも、日本の入学試験史上最難問と言われる98年後期理系第3問は解ける気がしない。
http://examist.jp/legendexam/1998-tokyo/

162 :132人目の素数さん:2018/11/22(木) 19:45:54.42 ID:abQO36xj.net
高橋くん、大学への数学の宿題に正解してたね
やっぱ、天才だね!

163 :学術:2018/11/23(金) 09:16:31.61 ID:SGy5FeB4.net
東大が毎年難問ではない。私立もあるし国立地方、県立、内部進学の数学まであるのに。

164 :132人目の素数さん:2018/11/24(土) 08:02:45.28 ID:mKOFZSkZ.net
宿題教えて下さい

165 :132人目の素数さん:2018/11/24(土) 18:38:30.94 ID:R9V2c3jb.net
問2、教えてください

166 :132人目の素数さん:2018/11/24(土) 18:46:09.37 ID:SYmj1AN9.net
自分で解けw

167 :132人目の素数さん:2018/11/24(土) 20:21:41.97 ID:R9V2c3jb.net
やだ!
誰か解いてください

168 :132人目の素数さん:2018/11/26(月) 07:26:40.85 ID:YFHQ7bYc.net
大学への数学の宿題、教えてください

169 :132人目の素数さん:2018/11/26(月) 07:56:43.63 ID:T9vb5MHL.net
うるせーw

170 :132人目の素数さん:2018/11/27(火) 06:29:35.11 ID:MxsTPpzh.net
早く教えてください

171 :132人目の素数さん:2018/11/27(火) 06:34:22.56 ID:Mh9FUlYr.net
なんだ荒らしか

172 :132人目の素数さん:2018/11/27(火) 19:20:30.29 ID:0qmOU9Vl.net
私は嵐のメンバーではありません

173 :132人目の素数さん:2018/11/27(火) 23:32:05.69 ID:QMhuYErk.net
嵐 寛壽郎 (1902/12/08〜1980/10/21) 映画俳優、映画プロデューサー。「鞍馬天狗」
石原裕次郎 (1934/12/28〜1987/07/17) 俳優、声優、歌手。「嵐を呼ぶ男」

174 :132人目の素数さん:2018/11/28(水) 20:56:09.11 ID:SiTlvnoQ.net
>>173
お歳がバレまするぞ

175 :132人目の素数さん:2018/12/01(土) 18:35:35.59 ID:m3XT35CL.net
嵐を呼ぶバンド:LUNA SEA

176 :132人目の素数さん:2018/12/01(土) 20:38:57.06 ID:yHu5Lcyz.net
吹けよ嵐 呼べよ嵐 嵐よ叫べ

177 :代弁しているのか?:2018/12/01(土) 20:38:59.57 ID:wDltouXA.net
金さん いいかげんにしてください。

178 :132人目の素数さん:2018/12/01(土) 21:04:59.37 ID:KmsVVdfD.net
おいらはドラマ、ヤクザのドラマ

179 :132人目の素数さん:2018/12/01(土) 23:27:47.59 ID:8GJD9Dqu.net
>>176
プログレ世代か

180 :132人目の素数さん:2018/12/02(日) 18:05:47.46 ID:eR3DlEVm.net
おまえら、紙の本と電子書籍なら、どちらが好み?

181 :132人目の素数さん:2018/12/02(日) 18:06:42.98 ID:aEOBuVbM.net
砂に書いたラブレター

182 :132人目の素数さん:2018/12/02(日) 18:38:45.21 ID:jrn3d5/7.net
>>181
うん、古い

183 :132人目の素数さん:2018/12/09(日) 10:16:01.29 ID:0KuZLaTQ.net
出題1、最後が無限総乗になって解けなかったけど、あれはそのままで良いのかな?

184 :132人目の素数さん:2018/12/11(火) 16:52:32.25 ID:ePCNQaV0.net
結局出題1の皆さんの見つけた最小値はいくら?

185 :132人目の素数さん:2018/12/11(火) 23:10:28.51 ID:1nYGcrrI.net
出題1の問題2は
・もしpの値が分かっていれば → p=1/2の時は普通に表裏で勝負を付け、それ以外の時は問題文の方法でやる。期待値の最小値はp=1/2の時の1回。
・pに関する情報が一切無ければ → 表裏のどちらが出やすいかは2人とも知らないので、1度のコイン投げで決めても公平である。
というわけで、いずれにせよ期待値の最小値は1。

しかし問題文をよく読むと「手元にあるコインは、どう見ても歪んでいて」とあるので、
pの値は確定している分けでは無く、逆に情報が全くないわけでは無い(どちらが
出やすいか程度は分かっている)模様。なので、この手のメタ的な方向からの
アプローチは無効だと思われる。

186 :132人目の素数さん:2018/12/12(水) 01:15:41.61 ID:tDlJqh5P.net
18年12月号の講評です。
今月は2問とも歯応えのある良問でした。

https://www.web-nippyo.jp/10317/
(すでに1月号の問題が出ている)

■出題1:レベル6以上(常連正解率75%以下)

上原隆平先生の良問。
表の出る確率pの歪んだコインを使って公平な勝負を行う。
問1は2回投げて表→裏、裏→表ならそれぞれAさん、Bさんの勝ち。
それ以外はやり直しとするルールのもと、期待値と期待値の最小値、それを与えるpを求める。
問2は問1より期待値が小さくなるルールを考える。

問1は簡単な計算問題。問2は難しい。
問1の勝敗条件をキープしつつ、ある回数を区切りとして
勝ち負けの条件を再帰的に定める方法がある。
この方法を採ると期待値は無限積となり最小値は問1の値を下回る。

>>151氏は早い段階で無限積に到達している。
以前お世話になった早解きさんでしょうか。


■出題2:完答レベル8以上(常連正解率20%以下)

奥田隆幸先生のエレガントな難問。
n個の実数a_1,a_2,...,a_nがある。
s_kを(1)a_kを-1倍、(2)a_{k-1},a_{k+1}にa_kを加える操作と定義する。
この操作を繰り返しすべて非負となったらゴールというゲーム。
問1は初期値によらずゴールできること、
問2は手順によらずゴール状態が1通りであること、
問3は左右対称形からスタートするとゴールも左右対称であることを示す。

問1はマイナスを掃き出す手順を1つ見つければよいだけ。
問2は行列群を考える方法があるが、解き切るのは難しい。
問3は問2の結果であるゴールの一意性を前提とすればゴリ押しでもなんとかいける。
つまり左右対称形でゴールできる手順を1つ見つければよい。
それは比較的簡単に見つかるが厳密な論証は簡単ではない。

エレガントに解いた方のコメントもとむ。

187 :132人目の素数さん:2018/12/12(水) 02:06:24.20 ID:Lw0Q+tOh.net
出題2は原題が Peter Winkler のパズル本に殆ど同じ出題があります。

Winkler, Peter. Mathematical Puzzles: A Connoisseur's Collection (p.88). Taylor & Francis - A. Kindle 版.

こっちは円形に並んでるのでややむずい。
――
Flipping the Polygon
The vertices of a polygon are labeled with numbers, the sum of which is positive. At any time, you may change the sign of a negative label, but then the new value is subtracted from both neighbors’ values so as to maintain the same sum.
(なぜか間違ってる参考図。略)
Prove that, inevitably, no matter which labels are flipped, the process will terminate after finitely many flips, with all values non-negative.
――
その問題の来歴の解説
――
Flipping the polygon

This puzzle generalizes one that appeared at the International Mathematics Olympiad in 1986 (submitted by a composer from East Germany, I am told) and subsequently termed “the Pentagon Problem.”
――
本の解答

元の数列a[k]とその級数S[k]を考える。
ただしa[0] = 0を追加しておく。
a[k]が全て0以上⇔S[k]が広義単調増大。
操作s[k]を行うとSはS[k-1]とS[k]の交換になる。
S[k]を交換していけばS[k]を広義単調増大にすることは可能で結果は一意的。
よって(1),(2)終わり。
(2)から(3)も左右対称の操作をして結果が同じになることから出る。

―― 例 縦線の右がa[k]、中がS[k]、右がその列に行った操作 ――
0 -5 3 4 -3 1 │ 0 -5 -2 2 -1 0 │ s[1]
-5 5 -2 4 -3 1 │-5 0 -2 2 -1 0 │ s[2]
-5 3 2 2 -3 1 │-5 -2 0 2 -1 0 │ s[4]
-5 3 2 -1 3 -2 │-5 -2 0 -1 2 0 │ s[3]
-5 3 1 1 2 -2 │-5 -2 -1 0 2 0 │ s[5]
-5 3 1 1 0 2 │-5 -2 -1 0 0 2 │ S[k]は広義単調増大。
(一番左はa[0]なので気にしなくて良い。)

188 :132人目の素数さん:2018/12/12(水) 04:48:12.08 ID:neKV7OZz.net
"Pentagon Problem"
http://mattbaker.blog/2014/02/25/the-pentagon-problem/
(図あり)

国防総省もたいへんでござる…

189 :132人目の素数さん:2018/12/12(水) 07:11:05.90 ID:6dGteMzi.net
>>184
こうかな?
https://www.wolframalpha.com/input/?i=product%5B1%2B2%5E(1-2%5Ei),%7Bi,0,%E2%88%9E%7D%5D

190 :132人目の素数さん:2018/12/12(水) 07:47:03.36 ID:tDlJqh5P.net
Winkler本はここ1,2年で2問でてますね。

191 :132人目の素数さん:2018/12/12(水) 07:51:15.14 ID:tDlJqh5P.net
>>186
> 問2は行列群を考える方法があるが、解き切るのは難しい。

行列の方法は見通しが悪そうに見えたのですが、そんなことはなかったですね。

> ■出題2:完答レベル8以上(常連正解率20%以下)

Winkler本にある有名問題ということでレベル5〜6(常連正解率75%以上)に訂正。

192 :132人目の素数さん:2018/12/12(水) 13:56:02.02 ID:TM+kVGT5.net
解説お願い致します

193 :132人目の素数さん:2018/12/12(水) 15:30:44.77 ID:neKV7OZz.net
>>189
Π[i=0,∞] (1 + (1/2)^{(2^i) -1}) = 3.4014709905728…

194 :132人目の素数さん:2018/12/12(水) 15:58:52.05 ID:3DeVBkfN.net
誰か最小性の証明できました?

195 :132人目の素数さん:2018/12/12(水) 18:31:25.97 ID:2BmCwuwo.net
出題1の小問2
題意把握ミスかもしれないが、
第三者Cに公平に
確率2分の1→表を「H」、裏を「T」
確率2分の1→表を「T」、裏を「H」
と定めてもらう。(無論A、BさんはCの定め方を知らない。)
この決め方なら、コイントス常に
1回かつ公平(どちらも勝率が等しい)な勝負になると考えた。

196 :132人目の素数さん:2018/12/12(水) 19:21:19.03 ID:duFxvgM0.net
>>195
なるほど。で、次にその確率1/2をどうやってこのコインで作り出すかを考えないと
→最初に戻る

197 :132人目の素数さん:2018/12/12(水) 20:26:30.60 ID:RpsOFyat.net
>>196
確かに。
「第三者に公平に」決めてもらうためのコインが必要になりますね。
堂々巡りになってしまうのか。

198 :132人目の素数さん:2018/12/12(水) 23:24:37.53 ID:tDlJqh5P.net
1月号が届きました

>>34
> ■出題1:レベル6〜7(常連正解率60〜80%)
> ※スツルムの定理不使用の場合レベル10(正解者0〜2名)

>>121
> 10月号の時弘先生の問題>>34が中高生の範囲で解けるのかどうかは気になります。スツルムの定理を前提知識として要求しているとは思えませんから。
>
> 「中高生の範囲」というのも考え始めると良く分からなくなってきますがね。スツルムの定理を理解するのに必要な知識は高校範囲の微分だけですからね。
> 一次変換だって少し前は高校でやっていたわけで。行列の知識を使わないで解くのが出題者の狙いだったとして、そのココロは私にはよく分からんですね。

時弘先生の解答編を流し読み。
スツルムの定理を知っていることが前提の問題でしたw

199 :132人目の素数さん:2018/12/12(水) 23:28:19.07 ID:tDlJqh5P.net
読者に媚びない時弘先生は今年も健在でした
10名が全問正解というのだから素晴らしい
来年も楽しみです

200 :132人目の素数さん:2018/12/15(土) 12:52:30.34 ID:qSLQQ1BM.net
1月号の出題2は既知の定理の別解法を求めるもの
こういうタイプは好きではない

201 :132人目の素数さん:2018/12/15(土) 19:17:19.24 ID:2nMJaIre.net
解答解説お願い致します

202 :132人目の素数さん:2018/12/16(日) 19:00:29.01 ID:B7KfzRA/.net
>>194
私の方法は、
http://blog.world-mysteries.com/wp-content/uploads/2011/11/Pascal_Sierpinski.png
のようなパスカルの三角形の一番上からスタートして、表なら左下、裏なら右下に
一つずつ移動し、偶数に当たったら終了するというもの。勝ち負けは、最後の表裏で決める。

最小性の証明は書けなかったけど、こんな感じかな。

パスカルの三角形は、そこに至るまでの経路数を意味するから、コインを停止
するまでの表裏の出方の総数に等しい。(ただし、それ以前に勝負が付いた時も
その回数までコインを空投げするとする。)

各コイン投げ停止位置では、この総数のうち半数がAの勝ち、半数がBの勝ち
でないと上手く行かない(要証明)。そのため、コイン投げが停止するのは
偶数の位置である必要があり、その中では、上に書いた方法では偶数に当たった時点で
停止しているため最小の手段を実現している。

問題は(要証明)のところで、ちゃんと示すのは色々と難しそう。

203 :132人目の素数さん:2018/12/16(日) 21:58:05.02 ID:Absg6hYo.net
>>202
おお、すばらしい。なんかこの方針で示せそう!

204 :132人目の素数さん:2018/12/16(日) 22:08:47.62 ID:wQjA8Rbq.net
う〜ん、示せそうだね。
答え聞くまではこれ最小性の証明つけられたやついないかもと思ってたけど、これ聞いた後だといっぱいいても不思議ないように思えてくるねww

205 :132人目の素数さん:2018/12/17(月) 09:42:18.95 ID:PFCMvANc.net
美味しそうなシェルピンスキーのビスケットだこと

206 :132人目の素数さん:2018/12/17(月) 10:02:33.86 ID:PO2HkIoB.net
右下の18のとこ、色間違えてるな

207 :132人目の素数さん:2019/01/05(土) 17:46:43.72 ID:5PQTtaHI.net
解答を書き込んでください

208 :よくわかりません:2019/01/05(土) 18:11:00.15 ID:mo6hpHrt.net
ひまだねええ

209 :132人目の素数さん:2019/01/05(土) 21:58:46.70 ID:sUTk3cKM.net
今月の2は、各種公式をどこまで使って良いか迷うなあ。

210 :132人目の素数さん:2019/01/06(日) 00:08:32.32 ID:CCXFLf8g.net
>>209
自分の知識を使ってよいか迷う問題は十中八九悪問です。
ってまだ締め切り前でしたね、すまそ

211 :低学歴超変態食糞愛好家井口千明の連絡先:葛飾区青戸6−23−16:2019/01/06(日) 11:55:46.66 ID:HlqGsCVU.net
☆★☆【神よこの者たちはもはや人間ではない悪魔であるこのような悪魔どもを一匹残らず殺してくださいお願いします】★☆★

《超悪質!盗聴盗撮・つきまとい嫌がらせ犯罪首謀者の実名と住所/死ねっ!! 悪魔井口・千明っ!!》
【要注意!! 盗聴盗撮・つきまとい嫌がらせ犯罪工作員】
◎井口・千明(東京都葛飾区青戸6−23−16)
※盗聴盗撮・嫌がらせつきまとい犯罪者のリーダー的存在/犯罪組織の一員で様々な犯罪行為に手を染めている
 低学歴で醜いほどの学歴コンプレックスの塊/超変態で食糞愛好家である/醜悪で不気味な顔つきが特徴的である

【超悪質!盗聴盗撮・嫌がらせつきまとい犯罪者の実名と住所/井口・千明の子分たち】
@宇野壽倫(東京都葛飾区青戸6−23−21ハイツニュー青戸202)
※宇野壽倫は過去に生活保護を不正に受給していた犯罪者です/どんどん警察や役所に通報・密告してやってください
A色川高志(東京都葛飾区青戸6−23−21ハイツニュー青戸103)
※色川高志は現在まさに、生活保護を不正に受給している犯罪者です/どんどん警察や役所に通報・密告してやってください

【通報先】
◎葛飾区福祉事務所(西生活課)
〒124−8555
東京都葛飾区立石5−13−1
рO3−3695−1111

B清水(東京都葛飾区青戸6−23−19)
※低学歴脱糞老女:清水婆婆 ☆☆低学歴脱糞老女・清水婆婆は高学歴家系を一方的に憎悪している☆☆
 清水婆婆はコンプレックスの塊でとにかく底意地が悪い/醜悪な形相で嫌がらせを楽しんでいるまさに悪魔のような老婆である
C高添・沼田(東京都葛飾区青戸6−26−6)
※犯罪首謀者井口・千明の子分/いつも逆らえずに言いなりになっている金魚のフン/親子孫一族そろって低能
D高橋(東京都葛飾区青戸6−23−23)
E長木義明(東京都葛飾区青戸6−23−20)
F若林豆腐店店主(東京都葛飾区青戸2−9−14)
G肉の津南青戸店店主(東京都葛飾区青戸6−35ー2

212 :132人目の素数さん:2019/01/06(日) 15:43:59.45 ID:PzQmca0Z.net
今月の2は、ベクトル公式を使って良いか迷うなあ。

213 :132人目の素数さん:2019/01/08(火) 18:06:43.65 ID:BR9lPKcp.net
大学への数学の宿題の解答を書き込んでくださいませ

214 :132人目の素数さん:2019/01/09(水) 01:23:21.36 ID:3v0fKymE.net
>>212
9日でござる。今夜もよく冷えるでござる....

今月号の2 ベクトル公式を使った解答の例

球面上に3点L,M,Nを
BOC面に垂直な向きにOL
COA面に垂直な向きにOM
AOB面に垂直な向きにON
となるようにとる。

MON面に垂直な向き ・・・・ OA
NOL面に垂直な向き ・・・・ OB
LOM面に垂直な向き ・・・・ OC
となる。(相反系)

上の定義から次が成り立つ。
OB×OC = sin(a) OL,
OC×OA = sin(b) OM,
OA×OB = sin(c) ON,
OM×ON = sinα OA,
ON×OL = sinβ OB
OL×OM = sinγ OC,

4面体O-ABCの体積をV
4面体O-LMNの体積をv
とおくと、
V = (1/6)OA・(OB×OC) = (1/6)sin(a) OA・OL,
v = (1/6)(OM×ON)・OL = (1/6)sinα OA・OL,
V/v = sin(a)/sinα,
b,β および c,γ についても同様。

215 :132人目の素数さん:2019/01/09(水) 12:36:44.39 ID:3v0fKymE.net
>>214

それで「幾何学的な考察から導い」たと言えるかねぇ・・・・

今月の1

便宜のため、周期的に延長する。
 x_{n+i} = x_i
 s_{n+i} = s_i + s_n
 (i=0,1,・・・・,n-1。もっと先まで伸ばしてもよい。)

f(x^(j)) = max{ min{s_(j+1)-s_j, s_(j+2)-s_j, ・・・・, s_(j+n)-s_j}, 0}
 = max{ min{s_(j+1), s_(j+2), ・・・・, s_(j+n)}, s_j} - s_j
 = min{s_(j+1), s_(j+2), ・・・・, s_(j+n)}, s_j} - min{s_j, s_(j+1), s_(j+2), ・・・・, s_(j+n)} (*)
n個先のsの方が大きいから、いくら追加しても min は変わらない。
∴ f(x^(j)) = σ_(j+1) - σ_j
ここに σ_k = min{s_k, s_(k+1), ・・・・, s_(2n-1), ・・・・} とおいた。

∴ (与式) = σ_n - σ_0 = s_n,

*) max{t, s_j} - s_j = t - min{s_j, t} を使った。

216 :132人目の素数さん:2019/01/09(水) 12:54:50.96 ID:5Ua6rKse.net
解答を書き込んでください

217 :132人目の素数さん:2019/01/10(木) 22:41:34.29 ID:HfguaWkg.net
>>214
そんなとこですかねえ。私は外積の三重積を開く公式を使っちゃいましたが。
sinが出てきて余弦定理がダメとなると、どうしても外積が出ちゃいますね。
幾何学的にできなくも無いでしょうが、外積の公式を証明するような流れになりそう。

218 :132人目の素数さん:2019/01/11(金) 04:43:59.67 ID:SKsrg5eA.net
>>214 も V(4面体の体積) = (1/6)(3稜のスカラー三重積) を使うのでござるな。

もっと幾何学的な考察から導くなら
 V(4面体の体積) = (1/3)S(底面積)h(高さ)
 S(底面積) = (1/2)sin(?)
とか行きたいところでござる。

219 :132人目の素数さん:2019/01/11(金) 19:35:24.29 ID:MP6vD1RL.net
最新問題きてますよ

220 :132人目の素数さん:2019/01/11(金) 20:44:21.38 ID:GWdjm0Zb.net
解答をお願い致します

221 :132人目の素数さん:2019/01/12(土) 15:12:13.69 ID:ilBN4PEM.net
出題1は偏微分とガンマ関数使ってちょろちょろやったら解けた
... が、俺でも思いつくような解法はエレガントじゃないんだろうなあ

222 :132人目の素数さん:2019/01/12(土) 17:20:56.99 ID:IDJn4geO.net
今月の出題のネット版、iPhoneで見ると求める式の最後(k+l)Ckが抜けているのでご注意を。

223 :132人目の素数さん:2019/01/12(土) 17:21:44.53 ID:IDJn4geO.net
↑出題1の話です

224 :132人目の素数さん:2019/01/12(土) 22:52:39.95 ID:AOPo8GCY.net
出題1は
 Σ[i+k=n] Σ[j+L=m] f_1(i, j) f_2(k, L)
の形だから、級数(= 生成関数)の積を使ってゴリゴリやったら解けそうだ
... が、俺でも思いつくような解法はちっともエレガントぢゃねえんだよなぁ

225 :132人目の素数さん:2019/01/12(土) 22:58:45.44 ID:u/uFvfkF.net
まだ応募期限来てないやつの話はやめとけよ。
こうやったら解けたの解けそうだだのあかんやろ。

226 :132人目の素数さん:2019/01/12(土) 23:02:37.17 ID:ilBN4PEM.net
ここそういうスレなんじゃないの?
それでも気を使って答えアップしなかったけど。

227 :132人目の素数さん:2019/01/12(土) 23:06:53.98 ID:u/uFvfkF.net
どういうスレかは誰が決めるもんでもない。
そういう問題じゃないやろ?
ここで雑誌の企画の妨害になる事してどうすんねん?
数学がどうこういう以前にそもそも人間として守らなあかん一線はあるやろ?
アホか?

228 :132人目の素数さん:2019/01/13(日) 00:01:28.96 ID:mMrZFLKO.net
>>1 に書いてあるけど。。。

229 :132人目の素数さん:2019/01/13(日) 12:13:51.78 ID:0a+BwHOH.net
19年1月号の講評:

■出題1:レベル4〜5(常連正解率95%以上)

徳重先生の良問風(?)な問題。

列x=(x_1, x_2,...,x_n)の先頭i個の部分和をs_iとする。
s_n>0のとき、xをj個ずらした列をx^(j)として、
f:x^(j)→max{min{s_{j+1},s_{j+2},...,s_j},0}
の和Σ_{j=0 to n-1} fがs_nに等しいことを示す問題。

>>215のようなエレガントなmin-max演算が出来ないと解けない、ということはない。
和を保存しつつ列を縮小する手術を考え、任意のxが非負列に変換されることを示す方針もある。


■出題2:レベル4(常連正解率〜100%)

長い問題文だがようするに球面正弦定理を示す問題。
よく知られた証明じゃつまらないので 幾何学的な考察から導け と制限が付けられている。
正弦定理だけに。(←これが言いたかっただけ)

230 :132人目の素数さん:2019/01/13(日) 16:10:15.33 ID:NXrSb6XM.net
初めて投稿してみようと思うのですが、
皆さんは証明などする際に、
論文のようにアイデアのクリティカルな部分は丁寧に書いて、ごく簡単と思われる部分は省略していますか?
それとも大学入試のように全ての場合についてしっかり議論してますか?

231 :132人目の素数さん:2019/01/13(日) 19:16:29.83 ID:gApxxpSc.net
その辺のさじ加減も証明の美しさに関わってくるし、書き手の腕の見せ所ではあるな。

232 :229:2019/01/13(日) 19:17:52.28 ID:E001cIR0.net
>>230
問題の難易度によりけりですね。
込み入った論理展開が必要なときは相対的に自明と思われる補題は証明を省くことがあります。
一方で簡単な問題では、論文レベルでは証明を省くような自明な帰結であっても、それを書かないと解答が「自明」で終わってしまうので丁寧に書き下すことがあります。(これがめんどくさいんだよ)

余談ですが、難しいことで有名な時弘先生の出題で、先生の論文の証明の行間を埋める問題が出されたことがあります。
その問題のエレ解正解者はたったの2名。
プロのレベルは凄いもんだなぁと思いました

233 :132人目の素数さん:2019/01/13(日) 21:25:50.44 ID:NXrSb6XM.net
>>231-232
非常に参考になりました
ありがとうございます

234 :132人目の素数さん:2019/01/15(火) 14:30:45.30 ID:nI37BuON.net
問題1は素数大富豪の人かよw

235 :132人目の素数さん:2019/01/15(火) 17:22:51.76 ID:v5Q3+ZFm.net
エレ解よりも大学への数学の宿題のが遥かに難しいよな

236 :132人目の素数さん:2019/01/15(火) 17:49:43.31 ID:uJxU5NAt.net
もちろん、数オリですが。

237 :132人目の素数さん:2019/01/16(水) 08:55:41.15 ID:E1byQYoF.net
数オリよりもはるかにエレ解のが難しいだろ

238 :132人目の素数さん:2019/01/16(水) 17:53:17.30 ID:lOjtUToz.net
いいえ、大数です。

239 :132人目の素数さん:2019/01/16(水) 19:53:33.19 ID:E1byQYoF.net
おまえら、大学への数学も定期講読してるのか?

240 :132人目の素数さん:2019/01/16(水) 20:34:26.10 ID:0Pwa3p0Q.net
>>234
なんだよ素数大富豪って? 説明したまえ!

241 :132人目の素数さん:2019/01/16(水) 21:15:16.22 ID:zMiw10ee.net
>>240
こういうことじゃないかな
https://dic.nicovideo.jp/a/%E7%B4%A0%E6%95%B0%E5%A4%A7%E5%AF%8C%E8%B1%AA
素数大富豪は、トランプゲーム「大富豪」をヒントに開発されたゲーム。
考案者は数学者の「せきゅーん」氏。

242 :132人目の素数さん:2019/01/17(木) 17:29:32.39 ID:w+hJ6CW4.net
本スレの「じゃんけん」を次のように定義する

 大数・宿題 は 数セミ・エレ解 に勝つ  >>235
 数セミ・エレ解 は 数オリ に勝つ  >>237
 数オリ は 大数・宿題 に勝つ

243 :132人目の素数さん:2019/01/22(火) 14:12:20.51 ID:9d/gEaUn.net
大学への数学の宿題の解答をお願い致します

244 :132人目の素数さん:2019/01/23(水) 06:01:53.60 ID:0KQkAS3a.net
そんなこと言われても KoMaL なぁ・・・

245 :132人目の素数さん:2019/01/23(水) 08:33:45.44 ID:6vayjVN5.net
ハンガリーのケマルって、難しいんですか?

246 :132人目の素数さん:2019/01/25(金) 05:12:51.50 ID:dfwh8WQW.net
本格インド料理の KOMAL をご存知ないからコマル。
メルカード武庫川(西宮市)の1階にあります。

香港の Math. Excalibur もどうぞ。
http://www.math.ust.hk/excalibur/

247 :132人目の素数さん:2019/01/27(日) 21:59:19.39 ID:bt3bUrbm.net
ハンガリーの数学雑誌:ケマルって、難しいんですか?

248 :132人目の素数さん:2019/01/28(月) 11:00:24.06 ID:cFJGCXF0.net
無関係な食べ物ネタを貼る馬鹿って小数点くん?

249 :132人目の素数さん:2019/01/28(月) 23:23:27.59 ID:0jpfHZEI.net
兵庫のKOMALを知ってる>>246氏は常連の中の常連。
(店の常連って意味じゃないよ)

250 :132人目の素数さん:2019/01/29(火) 04:23:03.09 ID:nVAMKPEn.net
クズであることに変わりはないな

251 :132人目の素数さん:2019/02/06(水) 23:08:05.08 ID:v/kRPHFT.net
問1はどれだけ簡単な形になるのか

252 :132人目の素数さん:2019/02/07(木) 10:13:24.98 ID:xf+t3w/A.net
Σ記号の無い形かな

253 :132人目の素数さん:2019/02/07(木) 10:40:16.60 ID:0ncIsLEm.net
>>252
そのΣのある形とない形、実際に無い形のほうが計算が簡単だと思う?
m, nが大きい場合でも明らかに簡単?

254 :132人目の素数さん:2019/02/07(木) 12:21:41.76 ID:xf+t3w/A.net
形で言えば、Σの中身に似てるけど、2項ぐらいかな。

255 :132人目の素数さん:2019/02/07(木) 12:35:08.62 ID:0ncIsLEm.net
>>254
うーん…実際に計算が簡単になってるか、という観点ではどう?

256 :132人目の素数さん:2019/02/09(土) 13:12:10.94 ID:jjMLHQvb.net
>>224
9日でござる。今日もよく冷えるでござる。(インフルに注意)

■出題1
n,mを非負整数とするとき、
 A(n,m) = Σ[i+k=n] Σ[j+L=m] f_1(i, j) f_2(k, L)
をより簡単な形で表わす問題。

i+k=n と j+L=m を見れば、生成関数を使う方針が浮かぶ。
 G (x,y) = Σ_(n,m) A (n,m) x^n y^m
 = {Σ_(i,j) f_1 (i, j) x^i y^j} {Σ_(k, L) f_2 (k, L) x^k y^L}
 = g_1(x, y) g_2(x, y)
ここで g_1, g_2 は f_1, f_2 の生成関数。i,j, k,L, m,n はすべての非負整数を亘る。

本問では f_1(i,j) = (-1)^i f_2(i, j) ゆえ
 g_1(x, y) = g_2(-x, y)   ⇒  G(x,y) はxの偶関数。

二項公式より
g_2(x,y) = Σ_s (2s+1) {Σ[k+L=s] C[k+L,k] x^k y^L} = Σ_s (2s+1)(x+y)^s = (1+x+y)/(1-x-y)^2,
g_1(x,y) = (1-x+y)/(1+x-y)^2,

G(x,y) = g_1(x,y) g_2(x,y) = 1/[(1-y)^2 -xx] + 4y/[(1-y)^2 -xx]^2
                                        ← 等比級数の和
 = Σ_n' {1/(1-y)^(2n'+2) + 4(n'+1) y/(1-y)^(2n'+4)} x^(2n')
                                        ← (一般化)二項公式
 = Σ[n:偶数] Σ_m {C(n+m+1,m) + 2(n+2)C(n+m+2,m-1)} x^n y^m,

A(n,m) = C(n+m+1,m) + 2(n+2) C(n+m+2,m-1)    (n:偶数)
     = 0   (n:奇数)

257 :132人目の素数さん:2019/02/09(土) 14:56:00.97 ID:jjMLHQvb.net
■出題2

・問1
 a = (a1, a2,・・・・, ak)
 #S(a) ≦ 2^k
 max{S(a)} = Σ[j=1,k] a_j
等から
 2^{k-1} ≦ n < 2^k,
 k[n] = 1 + [ log(n)/log(2) ]

・問2
 n = 4, 2^k -1, 2^k -2 かな。

258 :132人目の素数さん:2019/02/11(月) 10:23:58.99 ID:tU/X7aLT.net
↑いつもながらエレガントな解答ですね。
私は出題1が解けませんでした。
生成関数というものを使ったことがなく,2項展開と2項係数の公式で何とかしようとして失敗しました。

259 :132人目の素数さん:2019/02/12(火) 11:14:40.55 ID:3yrFAP8W.net
>>193
これより小さい解があったんだ!びっくりした。

260 :132人目の素数さん:2019/02/12(火) 11:26:02.32 ID:a/v4+a1X.net
まじ!?kwsk!!

261 :132人目の素数さん:2019/02/12(火) 12:50:10.57 ID:3yrFAP8W.net
例えば>>202 の図で言えば、偶数にたどり着いたら止めて最後に右下に進めばA、左下ならBの勝ちという方法が考えられるが、左右対象の位置で勝ち負け判定を入れ替えても、全体としての公平性は保たれる。
例として5段目の2つの4のいずれかにたどり着いた場合のみ勝ち負けを入れ替える。すると、4段目の3に着いた時点で、次に右下に行っても左下に行っても勝ち負けは同じになる。つまりそれ以上やる意味が無いからそこでゲームを止められる。
というように、地味に枝刈りをやっていくという方法。残念なのはこの筆者、コイン投げを途中で打ち切ってその段階の確率しか計算していないこと。まあ指数関数的に確率は減るからそれで良いかもしれんが、無限大回数までやったらどうなるかも知りたかったな。

262 :132人目の素数さん:2019/02/12(火) 14:03:28.90 ID:I/DWxBux.net
ちなみに最適解については未解決問題だそうだ。

263 :132人目の素数さん:2019/02/12(火) 18:40:48.50 ID:sSUCVV5a.net
今度からは、締め切り前に解答を書き込んでね

264 :132人目の素数さん:2019/02/12(火) 23:56:12.80 ID:ksfsSxAl.net
今度からは、 >>1>>5 を読んでから書き込んでね

265 :132人目の素数さん:2019/02/13(水) 01:38:02.23 ID:047aZGMD.net
>>3を読んでから書いてね

266 :132人目の素数さん:2019/02/13(水) 12:47:24.27 ID:8IWBhq9C.net
締め切り前に書き込むようにしようね、今度からは

267 :132人目の素数さん:2019/02/14(木) 15:56:40.78 ID:VBqvc7mU.net
問題2の意味が分からない

268 :132人目の素数さん:2019/02/14(木) 19:56:14.62 ID:HIfdLN9a.net
解答を書き込んでください

269 :132人目の素数さん:2019/02/15(金) 00:00:11.31 ID:af8RfF+1.net
>>267
 同感。
 すべての球にmを与えると たくさんできそうだが。
 正の数値だから無理数でもいいだろうし。
 「なるべく簡明にまとめた説明を工夫してください。」と言いたい。

270 :132人目の素数さん:2019/02/15(金) 01:48:35.53 ID:QYMtQpjE.net
>>269
すべての球を同じ値にするって事?
だったら四面体の頂点の4個(1つでも列とみなすとして4列)しかないんじゃない?

271 :132人目の素数さん:2019/02/15(金) 01:53:34.20 ID:QYMtQpjE.net
あ、k=1の場合を考えると、2個連なってる列でも良いから、4段積みの場合でさらに9列か。

272 :132人目の素数さん:2019/02/15(金) 13:57:27.20 ID:DLhUGCQ9.net
四面体の頂点って一直線上に隣接して並ぶってみなしていいのか?

例えば、3段積みなら一直線上に並ぶ球が18種類あるって意味と捉えたのだが
k=2が6種類、k=1が12種類の計18

273 :132人目の素数さん:2019/02/15(金) 14:08:54.29 ID:DLhUGCQ9.net
頂点は問題の本質でないからどっちでもいいとして、問題文の意味は通ると思うけど

274 :132人目の素数さん:2019/02/16(土) 15:34:41.24 ID:fRyCy6GA.net
>>259 >>260

2018-12月号出題1の解答例

(0) 2回ごとに
 pp → 未定
 pq → ○
 qp → ●
 qq → 未定
とすると、未定率は 1/2
 T_0 = 4

(1) 4回ごとに
 pppp → 未定
 pppq → □
 ppq  → ■
 pq  → ○
 qp  → ●
 qqp  → □
 qqqp → ■
 qqqq → 未定
とすると、未定率は 1/8,
 T_1 = 22/7 = 3.143

(2) 8回ごとに
 pppp pppp → 未定
 pppp pppq → △
 pppp ppq  → ▲
 pppp pq  → △
 pppp qp  → ▲
 pppp qqp  → △
 pppp qqq  → ▲
 pppq   → □
 ppq    → ■
 pq    → ○
 qp    → ●
 qqp    → □
 qqqp   → ■
 qqqq ppp  → △
 qqqq ppq  → ▲
 qqqq pq  → △
 qqqq qp  → ▲
 qqqq qqp  → △
 qqqq qqqp → ▲
 qqqq qqqq → 未定
とすると、未定率 1/128
 T_2 = 394/127 = 3.1023622

275 :132人目の素数さん:2019/02/16(土) 16:13:56.99 ID:FFe2XY7i.net
解答をどしどし書き込んでね

276 :132人目の素数さん:2019/02/16(土) 16:27:40.93 ID:fRyCy6GA.net
>>274

2^k 回目に
 p・・・・p q・・・・q
 q・・・・q p・・・・p
の出る確率が等しいことを利用すれば、
2^k -1 回目には 全p、全q 以外はすべて決着する。
2^k 回目も同じ。

277 :132人目の素数さん:2019/02/16(土) 16:29:45.80 ID:0eB6ecaX.net
【イラクの伊藤詩織】 ナディア・ムラド(26)がノーベル平和賞、自民党とISは、米軍傀儡のレイプ集団
https://rosie.5ch.net/test/read.cgi/liveplus/1550283084/l50

278 :132人目の素数さん:2019/02/17(日) 01:19:17.74 ID:o7kGLptn.net
>>272
 4頂点に m, 6稜の中点に 2m を与えるのでござるな。
 しかしnが大きいと 正四面体の内部を通過する場合もあるから・・・・

279 :132人目の素数さん:2019/02/17(日) 13:22:54.93 ID:6Biy5dSB.net
今月号の問題2は説明が分かりづらい。俺の理解力が足りないのかな。

280 :132人目の素数さん:2019/02/17(日) 18:47:54.15 ID:o7kGLptn.net
直線の向きに関しては、「隣接して」の語が重要と思われる。

281 :132人目の素数さん:2019/02/18(月) 01:00:43.37 ID:3JHzAJ6Y.net
>>274

(2) 8回ごとに
 pppp pppp → 未定
 pppp pppq → △
 pppp ppq  → ▲
 pppp pq  → ○'
 pppp qp  → ●'
 pppp qqp  → △
 pppp qqq  → ▲
 pppq   → □
 ppq    → ■
 pq    → ○
 qp    → ●
 qqp    → □
 qqqp   → ■
 qqqq ppp  → △
 qqqq ppq  → ▲
 qqqq pq  → ○"
 qqqq qp  → ●"
 qqqq qqp  → △
 qqqq qqqp → ▲
 qqqq qqqq → 未定

だった。

282 :132人目の素数さん:2019/02/18(月) 01:31:34.01 ID:3JHzAJ6Y.net
>>276 を利用すれば j回目に決着する確率 a[j] は次の漸化式を満たす。
a[1] = 0
a[2] = 1/2
2^(k-1) < j ≦ 2^k - 2 について
 a[j] = r_k・a[j - 2^(k-1)],   r_k = (1/2)^{2^(k-1) - 1}
a[2^k -1] = (2^k -2)・(1/2)^(2^k -1),       (← >>276)
a[2^k] = r_k・(1/2)a[2^(k-1)] = (1/2)^(2^k -1),

T_∞ = Σ(j=1,∞) j・a[j] = 3.1022064858592

T_2 = 394/127 = 3.10236220472441 よりわずか乍ら小さい。

ただし、これが最適解かどうか不明。   >>262

283 :132人目の素数さん:2019/02/18(月) 17:28:04.24 ID:CWoBzJSS.net
今月の解答を書き込んでください

284 :132人目の素数さん:2019/02/18(月) 17:46:23.58 ID:3JHzAJ6Y.net
>>282

a[1] = 0,
a[2] = 1/2,
a[3] = (1/2)^2,
a[4] = (1/2)^3,
a[5] = 0,
a[6] = (1/2)^4,
a[7] = 3(1/2)^6,
a[8] = (1/2)^7,
a[9] = 0,
a[10] = (1/2)^8,
a[11] = (1/2)^9,
a[12] = (1/2)^10,
a[13] = 0,
a[14] = (1/2)^11,
a[15] = 7(1/2)^14,
a[16] = (1/2)^15,
a[17] = 0,
a[18] = (1/2)^16,
a[19] = (1/2)^17,
a[20] = (1/2)^18,
a[21] = 0,
a[22] = (1/2)^19,
a[23] = 3(1/2)^21,
a[24] = (1/2)^22,
a[25] = 0,
a[26] = (1/2)^23,
a[27] = (1/2)^24,
a[28] = (1/2)^25,
a[29] = 0,
a[30] = (1/2)^26,
a[31] = 15(1/2)^30,
a[32] = (1/2)^31,
a[33] = 0,
・・・・

285 :132人目の素数さん:2019/02/18(月) 22:28:41.26 ID:bsJWKG8r.net
384=8!! 

53760=2(10!!)+12!!

8755200=8(12!!)+13(14!!)

1805690880=15(14!!)+12(16!!)+9(18!!)

471092428800=10(16!!)+15(18!!)+16(20!!)+5(22!!)

153043438141440=4(18!!)+2(20!!)+3(26!!)

60836834554675200=(20!!)+17(22!!)+15(24!!)+16(26!!)+12(28!!)+(30!!)

規則性を見つけてくれ〜(・ω・)ノ

286 :132人目の素数さん:2019/02/18(月) 23:34:12.21 ID:3W69H09M.net
>>283
大数スレにすっこんでろ カーッ(゚Д゚≡゚д゚)、ペッ

287 :132人目の素数さん:2019/02/19(火) 14:05:54.05 ID:ccqXPta2.net
宿題の解答をお願い致します

288 :132人目の素数さん:2019/02/21(木) 14:00:32.86 ID:8uzaI/2Z.net
宿題の答えって、180°-3φになる?

289 :132人目の素数さん:2019/02/21(木) 16:32:44.99 ID:8uzaI/2Z.net
みんなは、宿題どうなった?

290 :132人目の素数さん:2019/02/21(木) 21:30:19.12 ID:2QxwoA4m.net
ならんよ

291 :132人目の素数さん:2019/02/22(金) 02:08:37.07 ID:C/jpUJqi.net
>>213 >>243 >>287-289

大数スレ
http://rio2016.5ch.net/test/read.cgi/math/1395323240/l20

292 :132人目の素数さん:2019/02/22(金) 12:51:28.08 ID:K76H8vJO.net
宿題、90°-2φになったんだけど、みんなはどうなった?

293 :132人目の素数さん:2019/02/22(金) 19:45:58.44 ID:5yozgAif.net
45+3φ

294 :低学歴脱糞老女・清水婆婆の連絡先:葛飾区青戸6−23−19:2019/03/03(日) 09:58:20.93 ID:KV/cokeJ.net
【超悪質!盗聴盗撮・つきまとい嫌がらせ犯罪者の実名と住所を公開】
@井口・千明(東京都葛飾区青戸6−23−16)
※盗聴盗撮・嫌がらせつきまとい犯罪者のリーダー的存在/犯罪組織の一員で様々な犯罪行為に手を染めている
 低学歴で醜いほどの学歴コンプレックスの塊/超変態で食糞愛好家である/醜悪で不気味な顔つきが特徴的である
A宇野壽倫(東京都葛飾区青戸6−23−21ハイツニュー青戸202)
※色黒で醜く太っている醜悪黒豚宇野壽倫/低学歴で人間性が醜いだけでなく今後の人生でもう二度と女とセックスをすることができないほど容姿が醜悪である
B色川高志(東京都葛飾区青戸6−23−21ハイツニュー青戸103)
※色川高志はyoutubeの視聴回数を勝手に短時間に何百何千時には何万回と増やしたり高評価・低評価の数字を一人でいくつも増やしたり減らしたりなどの
 youtubeの正常な運営を脅かし信頼性を損なわせるような犯罪的業務妨害行為を行っています
※色川高志は現在、生活保護を不正に受給している犯罪者です/どんどん警察や役所に通報・密告してやってください

【通報先】
◎葛飾区福祉事務所(西生活課)
〒124−8555
東京都葛飾区立石5−13−1
рO3−3695−1111

C清水(東京都葛飾区青戸6−23−19)
※低学歴脱糞老女:清水婆婆 ☆☆低学歴脱糞老女・清水婆婆は高学歴家系を一方的に憎悪している☆☆
 清水婆婆はコンプレックスの塊でとにかく底意地が悪い/醜悪な形相で嫌がらせを楽しんでいるまさに悪魔のような老婆である
D高添・沼田(東京都葛飾区青戸6−26−6)
※犯罪首謀者井口・千明の子分/いつも逆らえずに言いなりになっている金魚のフン/親子孫一族そろって低能
E高橋(東京都葛飾区青戸6−23−23)
※高橋母は夫婦の夜の営み亀甲縛り食い込み緊縛プレイの最中に高橋親父にどさくさに紛れて首を絞められて殺されそうになったことがある
F長木義明(東京都葛飾区青戸6−23−20) ※日曜日になると風俗店に行っている

295 :132人目の素数さん:2019/03/09(土) 09:16:02.92 ID:RyYVa7GK.net
1問目難しかった

296 :132人目の素数さん:2019/03/10(日) 03:49:25.53 ID:5hnBzXCy.net
2019年3月号
■出題2 はやさしいですね。

 各球に1つずつ正の数値を与えるのですが…
 正4面体の4つの面を S_1〜S_4 とします。
 S_i 面を下にして置いたとき、球が下から 1+L_i 段目だったとします。
 L1+L2+L3+L4 = n-1,
 そこで、この球に自然数 (n-1)!/(L1!・L2!・L3!・L4!) を与えます。
 「一直線上に隣接して並ぶ球」は、稜の一つに平行になります。
 たとえば 稜34 に平行な球列の場合、面S3, S4 に平行なので L3, L4 が一定にです。
 また上の式から L1+L2 = n-1-L3-L3 (=k) も一定です。
 このk+1個の球列は
 (n-1)!/(L1!・L2!・L3!・L4!) = {(n-1)!/(L3!・L4!・k!)} {k!/(L1!・L2!)} = m {k!/(L1!・(k-L1)!)}
と表わせるので和列です。

ところで、 k+1個の球が並んだ和列は各向きに(n-k)個、つまり 6(n-k)個あります。
k=1,2,・・・,(n-1) で合計すれば 3n(n-1) 個になります。

297 :132人目の素数さん:2019/03/10(日) 09:56:00.22 ID:XWEaZV98.net
出題1、体積の縛りがなくてもこんなに難しいのだな。
https://www.dropbox.com/s/jhlxbyrxgw8xgjt/cube_cut.pdf?dl=0

298 :132人目の素数さん:2019/03/17(日) 16:32:30.65 ID:/y619UMF.net
>>214 >>215 が誰か見当がつく・・・・

すでに4月号に没頭でござるか

299 :132人目の素数さん:2019/04/09(火) 09:58:52.12 ID:sDGeXCoR.net
拙者は風車の弥七って忍びの者でござる。
このスレには誰も居らぬでござるな。
されば天井裏に忍んで宣伝を貼って参ろう。

武田鉄矢 主演 「水戸黄門」 第二弾
2019/05/19 から毎週日曜 夜6:00-6:54 (BS-TBS)
http://thetv.jp/news/detail/180917/

[前スレ.462, 620, 649, 650, 663]

300 :132人目の素数さん:2019/04/09(火) 23:31:05.87 ID:daZcPnQY.net
講評待ち

301 :132人目の素数さん:2019/04/09(火) 23:37:53.05 ID:daZcPnQY.net
>>299
いい時間にやるねえ水戸黄門
ファンが多いんだろうなあ

302 :132人目の素数さん:2019/04/10(水) 02:04:25.79 ID:x+zqr5Tw.net
今月も10日になった。 桜が満開・・・・

2019年4月号

■出題1
ガウス整数 z に対し、z = 5q + r (rの実部・虚部とも -2 〜 2) となるガウス整数 q,r が1組だけある。

・0,±1,±i はガウス素数でない。
  ∵ 定義より。
・r = 0 のとき z はガウス素数でない。
  ∵ q=r=0 なら z=0 で上記に帰着する。q≠0 なら |q|≧1,5 = (2+i)(2-i) = (1+2i)(1-2i) と分解される。
・|r|^2 = 5 かつ q≠0 のとき z はガウス素数でない。
  ∵ z = 5q + r = rr~q + r = r(r~q+1),|r| = √5 > 1,|r~q + 1|≧ |r~||q| - 1 ≧ √5 -1 > 1.
あとは q r≠0 ならばzが題意を満たさないことを云う。

■出題2
(1) 2色の場合は、外辺上に間隔も色も同じ2ペア(or 3頂点)があれば単色三角形を持つ。
  4段格子の外辺に、それがあることを示す。
(2) 略 (三つ巴など。何個かある。)
(3) 3色の場合は、間隔と色でさらに分類する。
  2592段以上の場合は単色三角形を持つことが分かった。
  (実際はずっと小さな段数でも成立つのかも・・・・)

303 :132人目の素数さん:2019/04/10(水) 09:05:16.55 ID:x+zqr5Tw.net
残念ながら今日は雨だす。。。
.
.
花は盛りに、月は隈なきをのみ見るものかは。
雨に対ひて月を恋ひ、垂れこめて春の行衛知らぬも、なほ、あはれに情深し。
咲きぬべきほどの梢、散り萎れたる庭などこそ、見所多けれ。
歌の詞書にも、「花見にまかれりけるに、早く散り過ぎにければ。」とも、「障ることありてまからで。」なども書けるは、 「花を見て。」と言へるに劣れることかは。
花の散り、月の傾くを慕ふならひはさることなれど、ことに頑なる人ぞ、「この枝かの枝、散りにけり。今は見どころなし。」などは言ふめる。
                                      兼好法師「徒然草」137段

304 :132人目の素数さん:2019/04/12(金) 19:53:54.33 ID:Ft4A/3fN.net
・・・などと云っているうちに 御老公の出題でござる。

・5月号出題2

f(P) は 点Pの座標 (x,y,z) について3次以下 (5次以下) の多項式

「立体角」Ωを使えば I(f) = (1/4π)∫f(P)dΩ

と理解するのでござるか?

305 :132人目の素数さん:2019/04/12(金) 22:44:30.87 ID:veTSCGve.net
>>302
出題2の(3)難しかった

306 :132人目の素数さん:2019/04/13(土) 09:20:48.90 ID:ujI4PsHM.net
3月の出題2(3)、『等周期の6個が同色⇒単色三角形が存在』が言える
このアプローチで解いた人いる?

307 :132人目の素数さん:2019/04/13(土) 13:42:47.10 ID:EI29cEsh.net
・4月号 出題2の(3)
 拙者は(1)の解法を流用したので、かなり泥臭いでござる。

・あらすじ
 n段の三角格子の(外周)辺上の頂点の数 …… n+1個
 最多色の頂点の数 …… m ≧ [n/3] +1,
 そのペアの数 …… C(m,2) とおり
 ペアの距離(1〜n) と 第3頂点の色(2種) で2n組に分類する。
 最大組に含まれるペア …… L ≧ [(C(m,2)-1)/2n] +1,
 ペアのペアの数  …… C(L,2)
 ペア間のずれ(1〜n-1) で分類する。
 最大組に含まれるペアのペア …… k ≧ [(C(L,2)-1)/(n-1)] +1,
 n≧2592 ⇒ m≧865 ⇒ L≧73 ⇒ k≧2 ⇒ 単色三角形が存在

308 :132人目の素数さん:2019/04/13(土) 14:27:18.81 ID:CWxRnGcy.net
もはや人ではなく鳩

309 :132人目の素数さん:2019/04/13(土) 16:11:16.48 ID:P8JJuWmJ.net
とりあえず10000という数字を華麗に無視すればVan der Waerdenでもいける。

Thm (Van der Waerden)
r,kを自然数とするとき自然数W(r,k)が存在して1〜W(r,k)までの自然数のいかなるr色の塗り分けに対しても長さk以上の同色の等差数列がとれる。

一辺の長さがW(3,W(2,3)+1)以上の正三角格子(と呼ぼう)をR,W,Yに塗り分ける。
ある辺上に長さがW(2,3)+1の同色に塗られた等差数列が出現する。
Rに塗られているとして公差をaとする。
これらのなかのaだけ離れた2点を頂点とする正三角形のもう一つの頂点はこの辺からaだけ離れたところで長さW(2,3)の等差数列をなす。
このなかにRに塗られたものがあれば終了。
すべてW,Yのときはこのなかに長さ3の同色に塗られた等差数列が出現する。
Wに塗られているとして公差をbとする。
これらの中のbだけ離れた2点を頂点とする正三角形のもう一つの頂点はこの辺からbだけはなれたところで長さ2の等差数列?をなす。
このなかにR,Wにぬられたものがあれば終了。
Yに塗られているとして公差?をcとする。
これらの中?のcだけ離れた2点を頂点とする正三角形のもう一つの頂点はこの辺からcだけはなれたところにポツンとある。
これがR,W,Yなんでもこいや。

310 :132人目の素数さん:2019/04/13(土) 18:41:00.30 ID:CWxRnGcy.net
>>309
巨人の肩に立つのもまたエレガント哉

311 :132人目の素数さん:2019/04/13(土) 23:55:48.34 ID:3GtiLhdc.net
よくよく考えたらV(3,4)=293を利用したら一辺の長さ293の格子正三角形の3色塗り分けは必ず単色三角形含むね。
V(3,4)≦10000をエレガントに示せれば文句なしになるんだけど。

312 :132人目の素数さん:2019/04/14(日) 01:02:43.49 ID:LgnGaG/f.net
>>311はなかったことにorz

313 :132人目の素数さん:2019/04/14(日) 04:43:17.59 ID:BM/pWIx5.net
>>309
 W(2,3) = 9,
 n+1 ≧ W(3,10) ⇒ 成立

 W(3,10) は大きそう・・・・


>>306
 n+1 ≧ W(3,6) ⇒ 成立

(1) を使えば単色三角形を持つことが分かる。

W(3,6) はどうでしょう?

W(3,2)=4, W(3,3)=27, W(3,4)=293, ・・・・

う〜む。

314 :132人目の素数さん:2019/04/14(日) 08:55:17.63 ID:oGML2lWA.net
ファン・デル・ヴェルデン数が上から押さえられていることを使えば簡単に解けますが、あまりにあまりに大きい哉

315 :132人目の素数さん:2019/04/14(日) 08:56:57.08 ID:oGML2lWA.net
>>302
> 2592段以上

あんたが大将水戸黄門

316 :132人目の素数さん:2019/04/15(月) 03:12:49.99 ID:8IDqhR4r.net
>>313
 W(3,5) > 2173,
 W(3,6) > 11191,
 W(3,7) > 48811,
 W(3,8) > 238400,
 W(3,9) > 932745,
 W(3,10) > 4173724,
 W(3,11) > 18603731,
らしい。

http://en.wikipedia.org/wiki/Van_der_Waerden_number 

317 :132人目の素数さん:2019/04/15(月) 07:26:55.74 ID:ZZz0J292.net
(1)使うならn≧W(3,5)のとき単色3角形ができるんじゃないの?
残念ながらW(3,k)が決定してるのはk=3,4だけみたいだけど。

318 :132人目の素数さん:2019/04/15(月) 14:06:04.43 ID:8IDqhR4r.net
>>306>>309 のように「等間隔な」同色列を使えば (1) を応用できるし
 簡潔でエレガントな解答だろうけど・・・・

それを要求すると、段数nがベラボーな大きさになっちゃうのがナニだ。。。  >>314

319 :132人目の素数さん:2019/04/15(月) 17:39:56.11 ID:jZSRa5NW.net
一辺587でいけるかも。

以下複素平面上で考えるとしてζ=exp(iπ/3)、αを任意にとり、β=ζαとする。
色は{R,W,Y}とする。
C上の点z = sα+tβ (s,t∈R)に対しp(z) = s、q(z) = tと定める。
α、βの張る格子をLとおく。
a = W(3,4)-1=292とおく。
p≧0,、q≧0、p+q≦2aをみたすLの点全体をTとおき{R,W,Y}で彩色する。
a≦k≦2aにたいし線分a≦p(z)≦2a、q(z)=kを満たすL格子のなかに同色の長さ4の等差数列がとれる。
初項をa(k)、公差をd(k)、色をc(k)とおく。
1≦d(k)≦a/3、c(k) = {R,W,Y}により相異なるk1,k2でd=d(k1) = d(k2)、c=c(k1)=c(k2)となるものがとれる。
c={Y}としてよい。
二つの等差数列を順にz1,z2,z3,z4,w1,w2,w3,w4とおく。
z中心にwをπ/3回転させた点f(z,w)は(1-ζ)z+ζwである。
zi、wj、vij=(1-ζ)z1 + ζw1 + (i-1)d(1-ζ)α + (j-1)dζαは正三角形をなし,すべてTの点でz1,z2がY色なのでvijはR,W色のいずれかである。
これら16点の中には(1)より単色3角形が存在する。

320 :132人目の素数さん:2019/04/16(火) 01:55:40.35 ID:BFbJOdl7.net
>>319
訂正。Tは p≧0,、q≧0、p+q≦4a。
よって一辺の長さは4a+1=1169。

321 :132人目の素数さん:2019/04/16(火) 12:33:05.54 ID:ORPGLRHi.net
これ以上書くとうざいかもしれないのでやめとくけど877でもいけた。

322 :132人目の素数さん:2019/04/16(火) 23:38:16.57 ID:JnAR3eFU.net
>>321
そんなこと言わんと続けてけろ

323 :132人目の素数さん:2019/04/19(金) 01:44:58.53 ID:+B6nkes7.net
>>322ありがとう。お言葉に甘えてn=877の解かいてみる。

>>319と同じa,α,β,ζ,p,q,Lをとる。
T = {z|p≧0,q≧0,p+q≦3a}
とする。
>>319と同様にして領域
0≦p≦a,a≦q≦2a}
において公差dが整数の長さ4の数列z1,z2,z3,z4,w1,w2,w3,w4がとれる。
q(z1) ≦ q(w1)としてよい。
ziを中心にwiをπ/3だけ負の方向に回転した点をvijとおく。
vijは設定から全てT上にあり4段の格子三角形を含むので(1)より終。

まだまだ減らせる予感はありあり。

324 :132人目の素数さん:2019/04/20(土) 09:57:14.14 ID:ycjJX9Ok.net
>>319
>zi、wj、vij=(1-ζ)z1 + ζw1 + (i-1)d(1-ζ)α + (j-1)dζαは正三角形をなし,すべてTの点でz1,z2がY色なのでvijはR,W色のいずれかである

問題文から、単色三角形は各辺が三角格子に対して平行である必要がある
たとえばzi, wj, vijが単色三角形になるのはziとwjが同一直線上にあるときに限られる

325 :132人目の素数さん:2019/04/20(土) 10:00:36.18 ID:JhHQ+Wyb.net
>>224
 出題者の用意した容易な解答だった。

〔朱世傑の公式〕
 Σ[j=0,m] C[n+j,n] = C[n+m+1,m]

(略証)
 C[n+j,n] = C[n+j+1,n+1] - C[n+j,n+1]
から出る。

〔朱-ファンデルモンドの公式〕Chu-Vandermonde formula
i+k=n のとき
 Σ[j+L=m] C[i+j,i] C[k+L,k] = Σ[j=0,m] C[n+j,n] = C[m+n+1,m]

(略証)
 Σ[j=0,∞) C[i+j,i] x^j = 1/(1-x)^(i+1),
 Σ[L=0,∞) C[k+L,k] x^L = 1/(1-x)^(k+1),
辺々掛けて x^m の係数を比べる。

326 :132人目の素数さん:2019/04/20(土) 16:46:24.72 ID:A+W91r5W.net
>>324
ホントだ。
文章読んでなかった。例2はダメな例なのか。
斜めありなら200以下の解答もできたんだけどな。

327 :132人目の素数さん:2019/04/20(土) 22:41:45.28 ID:ycjJX9Ok.net
>>307
> ペア間のずれ(1〜n-1) で分類する。
> 最大組に含まれるペアのペア …… k ≧ [(C(L,2)-1)/(n-1)] +1,

> k≧2 ⇒ 単色三角形が存在

ここを詳しく西郷頼
kが2以上で単色三角形が存在、というところ

328 :132人目の素数さん:2019/04/20(土) 22:43:26.66 ID:ycjJX9Ok.net
>>327
おっと失礼。自明でござった

329 :132人目の素数さん:2019/04/20(土) 22:44:14.70 ID:ycjJX9Ok.net
うーん>>307は見事

330 :132人目の素数さん:2019/04/20(土) 23:02:29.21 ID:ycjJX9Ok.net
いやまだちょっとわからんかった。。
2つの等間隔のペアの第三頂点が同色だとしても、単色三角形をなす最後の頂点は別の色である可能性はないかな?
それは2n個に分けた別の類だから

331 :132人目の素数さん:2019/04/20(土) 23:36:29.14 ID:ycjJX9Ok.net
>>330
いや今度こそ分かった
ペアのペアのペアが存在すると、最後の頂点がどの色でも単色三角形になりますな


おみごと

332 :132人目の素数さん:2019/04/21(日) 00:24:47.41 ID:ET8VwjE0.net
>>324さんの指摘をいただいて証明チェックしてみた。
>>319,>>323は問題ない。

>vij=(1-ζ)z1 + ζw1 + (i-1)d(1-ζ)α + (j-1)dζα これ間違い
>vij=(1-ζ)z1 + ζw1 + (i-1)d(1-ζ) + (j-1)dζ 正しくはこっち

を1≦i≦4、1≦j≦4で動かしたときの変化の方向ベクトルはdζ、d(1-ζ)でこれはdをπ/3,-π/3回転させたものでもともとdが辺に平行なのでdζ、d(1-ζ)も辺に平行。
よって各辺がもとの正三角形の辺に平行というしばりがあっても大丈夫。
しかし残念ながら自分のノートに書いてた200を切る解は辺が元の辺に垂直の単色三角形の非存在を利用してたのでアウト。
残念。

333 :132人目の素数さん:2019/04/21(日) 00:47:15.46 ID:9Rhfeojo.net
>>325>>325
朱世傑 (1249〜1314)

数セミ増刊「100人の数学者」日本評論社(1989/May) p.31

334 :132人目の素数さん:2019/04/21(日) 00:52:19.43 ID:qAtOnHpw.net
>>332
実は未だにちゃんと読めてないのだけど質問

例えばi=j=1のときzi, wj, vijは題意を満たす正三角形とは限らないが、証明のスジには影響なし?
z1, z2が同色と述べているので何か自分が読み違っている気がしてならない

335 :132人目の素数さん:2019/04/21(日) 01:19:08.33 ID:ET8VwjE0.net
>>324
zi、wj、vijの配置は関係ありません。
    v11、v12、v13、v14
  v21、v22、v23、v24
 v31、v32、v33、v34
v41、v42、v43、v44
とvijの16点の中に各辺がもとの三角形の辺に平行で4段、2色に塗り分けられている事を利用してます。

336 :132人目の素数さん:2019/04/21(日) 07:36:20.59 ID:qAtOnHpw.net
>>335
そうなんですか。

>>319
>zi、wj、vij=(1-ζ)z1 + ζw1 + (i-1)d(1-ζ)α + (j-1)dζαは正三角形をなし,

とあったので。

337 :132人目の素数さん:2019/04/21(日) 08:48:07.25 ID:DY2tH8OT.net
>>336
あ、しまった。そこ関係あります。
やっぱりだめですね。

338 :132人目の素数さん:2019/04/21(日) 09:12:31.98 ID:qAtOnHpw.net
>>337
> やっぱりだめですね。

そうなんですか(←何もわかってないヒトw)

残念です
しかしこのスジのように、外辺以外も積極的に使えるとnを減らせていいんですけどね

339 :132人目の素数さん:2019/04/21(日) 09:18:14.31 ID:DY2tH8OT.net
>>338
残念です。
うーん、辺の向きに縛りつけられると途端に自由度減っちゃ居ますね。
そのまま同じ公差、色の長さ4の等差数列1組を真横に見つけにかかるとW(3,4)^2前後になるので90000前後になってしまう。

340 :132人目の素数さん:2019/04/21(日) 13:05:31.36 ID:9Rhfeojo.net
>>325

Σ[j=0,∞) C[j,0] x^j = Σ[j=0,∞) x^j = 1/(1-x),

xで i回微分して i! で割れば

Σ[j=0,∞) C[i+j,i] x^j = 1/(1-x)^(i+1),

341 :132人目の素数さん:2019/04/24(水) 14:40:41.08 ID:vK+1FJs+.net
>>304

"spherical n-design" とか云うらしい。

J.J.Seidel (1919〜2001)

342 :132人目の素数さん:2019/05/01(水) 21:42:46.28 ID:WcJK5X24.net
>>307
この問題三角形が辺に平行という縛りがある限りこれしか基本戦略なさそうですね。
ちょっと>>307さんの方法を詰めるとn=1808まではいけたけど、しかしメチャメチャめんどくさくなってエレガントな解法の真逆の路線を疾走しないといけない。

343 :132人目の素数さん:2019/05/10(金) 22:29:56.43 ID:63rFX3UC.net
拙者は風車の弥七って忍びの者でござる。
このスレには誰も居らぬでござるな。
されば天井裏に忍んで講評を待つでござる・・・・

344 :132人目の素数さん:2019/05/11(土) 00:46:14.13 ID:Qz8IJs2N.net
>>343
2問目解いた人いますかねえ?

345 :132人目の素数さん:2019/05/11(土) 05:51:29.15 ID:XGJyhqkH.net
2019年5月号

■出題2

 I(f) は Σ上での平均であり (4π) のベキが出てきて面倒である。
 そこで単位球上での平均 I~(f) をガウス積分を使って計算すれば
 I~(x^h y^k z^L) = (h-1)!!(k-1)!!(L-1)!!/(h+k+L+1)!!  (h,k,L とも偶数か0のとき)
 h,k,L のいずれかが奇数のときは 0 である。(消滅則とよぶ。)
 立方体S~の稜の向きをx,y,z軸にとれば、頂点は(±1/√3, ±1/√3, ±1/√3)
 正八面体T~の体対角線をx,y,z軸とすれば、頂点は(±1,0,0) (0,±1,0) (0,0,±1)
 これらの配置はxy平面、yz平面、zx平面 について面対称だから消滅則が成り立つ:
  S~(x^h y^k z^L) = T~(x^h y^k z^L) = 0    (h,k,Lのいずれかが奇数のとき)
 よって h,k,L とも偶数または0のものを考えればよい。
・問1
 多項式f が3次以下ならば 1, xx, yy, zz に限る。
 I~(1) = S~(1) = T~(1) = 1,
 I~(xx) = S~(xx) = T~(xx) = 1/3,
・問2
 上記の配置では、S~の頂点と T~の頂点が斥け合う形で、都合がよい。
 多項式fが5次以下ならば上記のほかに x^4, xxyy, … がある。
 I~(x^4) = 1/5,  S~(x^4) = 1/9,  T~(x^4) = 1/3,
 I~(xxyy) = 1/15,  S~(xxyy) = 1/9,  T~(xxyy) = 0,
 ∴ I~(f) = (3/5)S~(f) + (2/5)T~(f),
 ∴λ = 3/5,
・問3
 正12面体と正20面体の配置を次のようにとる。
 消滅則が成立つように xy平面、yz平面、zx平面 について面対称とする。U~の頂点とV~の頂点は斥け合う。
 正12面体U~の頂点は、3つの平面上の長方形 (φ^2:1) の頂点(12点)、および立方体S~の頂点(8点)。
 正20面体V~の頂点は、3つの平面上の長方形(1:φ) の頂点(12点)。
 多項式fが5次以下の場合は上記と同様に計算して I~(f) = U~(f) = V~(f),
 多項式fが9次以下の場合を計算することにより  I~(f) = (9/14)U~(f) + (5/14)V~(f),
 ∴λ = 9/14.

346 :132人目の素数さん:2019/05/11(土) 08:10:43.96 ID:XGJyhqkH.net
I~、S~、T~ の計算
 h,k,L とも偶数または0とする。

(x^h)(y^k)(z^L)exp{-(xx+yy+zz)/2} を全空間で積分しよう。

(1)極座標系で(半径aの球体で)積分すると、
 dΩ = sinθ dθ dφ, として、
 ∬(r sinθcosφ)^h (r sinθsinφ)^k (r cosθ)^L exp(-rr/2) dΩ rr dr
 = ∫(sinθcosφ)^h (sinθsinφ)^k (cosθ)^L dΩ ∫[0,a] exp(-rr/2) r^(h+k+L+2) dr
 = 4π I~(x^h y^k z^L) ∫[0,a] exp(-rr/2) r^(h+k+L+2) dr,
 → 4π I~(x^h y^k z^L)(h+k+L+1)!! √(π/2)   (a→∞)

(2) デカルト座標系で別々に積分する(一辺が2bの立方体で積分する)と
 ∫[-b,b] x^h exp(-xx/2)dx・∫[-b,b] y^k exp(-yy/2)dy・∫[-b,b] z^L exp(-zz/2)dz
 → (h-1)!!√(2π)・(k-1)!!√(2π)・(L-1)!!√(2π)    (b→∞)
よって
 I~(x^h y^k z^L) = (h-1)!!(k-1)!!(L-1)!!/(h+k+L+1)!!

また、S~(x^h y^k z^L) = (1/3)^((h+k+L)/2),
   T~(x^h) = 1/3,   (h≧2)
   T~(x^h y^k) = T~(x^h y^k z^L) = 0  (h,k,L≧2)

347 :132人目の素数さん:2019/05/12(日) 01:08:36.90 ID:B2mXwahY.net
>>345 のφは黄金数です。 φ= (1+√5)/2 = 1.618034

>>346 のφはz軸のまわりの方位角です。(極座標系)

紛らわしくてスマソ

348 :132人目の素数さん:2019/05/12(日) 17:01:46.34 ID:LltDAQnP.net
さすが。

1問目はデカルト則つかわずに解けるんでしょうか

349 :132人目の素数さん:2019/05/12(日) 20:45:42.75 ID:B2mXwahY.net
2019年5月号

■出題1

題意を満たす任意のn次多項式を P(x) とする。
ロルの定理より、P '(x)、P "(x) も題意を満たす。
{P(x)の0でない係数の個数} = {P "(x)の0でない係数の個数} + δ(xの係数) + δ(定数項)
ここで (xの係数) = (定数項) = 0 と仮定すると P(x) = 0 が重根0をもち、題意と矛盾。
∴少なくとも一方は0でない。
{P(x)の0でない係数の個数} ≧ {P "(x)の0でない係数の個数} + 1 ≧ c_(n-2) + 1,
c_n は、題意を満たすn次多項式に対する、0でない係数の個数の最小値。(1≦c_n≦n+1)
P(x)は任意だったから c_n ≧ c_(n-2) +1,
これと c_1 = 1, c_2 = 2 から c_n ≧ [n/2] +1,
あとは、等号が成立する具体例を示せばよい。


(多項式が微分可能であることは明らかと思われる)

350 :132人目の素数さん:2019/05/13(月) 23:10:02.52 ID:3tnZa2ew.net
>>349
巧みですなあ

351 :132人目の素数さん:2019/05/14(火) 13:38:45.65 ID:spD4KjCm.net
>>348
n次多項式P(x) は題意を満たし P(0)≠0 とする。
P(x)=0 は虚数根をもたないから、デカルトの符号法則より
 (正根の個数) = {P(x)の係数の符号反転の数} ≦ (0でない係数の個数) - 1,
 (負根の個数) = {P(-x)の係数の符号反転の数} ≦ (0でない係数の個数) - 1,
よって
 n = {P(x)=0 の実数根の個数} = (正根の個数) + (負根の個数) ≦ 2(0でない係数の個数) -2,
∴ (0でない係数の個数) ≧ [n/2] + 1,
P(0)=0 の場合も n-1次多項式 Q(x) = P(x)/x とおけば題意により Q(0)≠0 だから上式が成立つ。
以下省略

352 :132人目の素数さん:2019/05/19(日) 15:38:55.15 ID:9g/K/0vL.net
>>298

すでに6月号に没頭でござるか。

生成関数やチェビシェフ多項式を使うのは中身が見えないから「明示的」ぢゃないんだろうな。。。

353 :132人目の素数さん:2019/05/31(金) 17:32:40.84 ID:RppNqGYw.net
明示の意味がはっきりしない。
簡単で、余計な付属なしの答えという意味かな

354 :132人目の素数さん:2019/06/03(月) 22:42:15.67 ID:e1OJdgiY.net
>>353
Σを使うか使わないか、なんてのも考えどころ?

355 :132人目の素数さん:2019/06/04(火) 13:40:05.25 ID:o+gkvWNO.net
√を使えばΣは要らねぇ、なんてのも考えどころ。

356 :132人目の素数さん:2019/06/08(土) 15:31:21.96 ID:Bn0CqP0B.net


357 :132人目の素数さん:2019/06/09(日) 07:46:01.34 ID:chDojVzE.net
発展問題を除けば新読者歓迎号みたいな難易度だったがまあよし

358 :132人目の素数さん:2019/06/10(月) 00:42:06.13 ID:0ZLkhJ7v.net
2019年5月号
■出題2

漸化式
 f_(n+1)(z) = (z + 1/z)f_n(z) - f_(n-1)(z),
すなわち
 F_(n+1)(x) = x F_n(x) - F_(n-1)(x),
特性多項式 t^2 - x t + 1,
特性根 (x±y)/2,  ここに y=√(xx-4),

このままでも解けますが、f_n(z) の項の1つ飛ばしの和を
 h_n(z) := Σ[k=0,n] z^(-n+2k) := H_n(x)
とおき
 f_n(z) = h_n(z) + h_(n-1)(z),
 F_n(x) = H_n(x) + H_(n-1)(x),
としてもよい。上と同様にして漸化式
 H_(n+1)(x) = x H_n(x) - H_(n-1)(x),
 H_0 = 1,
 H_1(x) = x,
より
 H_n(x) = (1/y)({(x+y)/2}^(n+1) - {(x-y)/2}^(n+1))   (x≠±2),
  = (1/2)^n Σ(k=0,[n/2]) C(n+1,2k+1) x^(n-2k)・(xx-4)^k,
 H_n(-2) = (-1)^n・(n+1),
 H_n(2) = n+1,

359 :132人目の素数さん:2019/06/11(火) 02:49:08.97 ID:a3rUuuK+.net
↑6月号でござった。スマソ


>>351
P(x)=0 が虚数根をもつ場合は、実根がその分(偶数個)少なくなる。

数セミ増刊「数学100の定理」日本評論社(1983) p.65-66

360 :132人目の素数さん:2019/06/12(水) 04:26:32.92 ID:HaAncPiV.net
>>299
水戸黄門 第2部 (BS-TBS版)
http://thetv.jp/program/0000954074/

361 :132人目の素数さん:2019/06/14(金) 22:24:21.08 ID:/L38U7QY.net
昇順から降順にできないnってあるの?

362 :132人目の素数さん:2019/06/15(土) 20:24:04.98 ID:IIziHdYA.net
第一種の合流型超幾何関数(クンマー)

1F1[a; b; z] = 1+Σ[k=1, ∞] {a(a+1)・・・・(a+k-1)/b(b+1)・・・・(b+k-1)} z^k/k!

1F1[-n; -2n; z] = {n!/(2n)!} Σ[k=0, n] {(2n-k)!/(n-k)!k!} z^k

363 :132人目の素数さん:2019/06/18(火) 11:21:13.16 ID:mcKYO9vg.net
解答解説お願い致します

364 :132人目の素数さん:2019/06/24(月) 14:55:13.04 ID:mwe/y+rW.net
>>361
PCで力技で計算してるが、見つからんなあ。大きなnじゃないとダメか。

365 :132人目の素数さん:2019/06/26(水) 16:45:42.75 ID:vhiq1iMT.net
>>364
nがあるかどうかはともかく,どんな実装してるかに興味あるんだが

366 :132人目の素数さん:2019/06/26(水) 17:47:20.90 ID:FYvfUZxU.net
>>365
Mathematicaで書いた数行のプログラムだよ。締め切り過ぎたらここに出すか。

367 :132人目の素数さん:2019/06/29(土) 16:40:58.14 ID:DHiuKlHq.net
【数セミ】エレガントな解答をもとむ3【2018.10】
ふうL@Fu_L12345654321
学コン1傑いただきました!
とても嬉しいです!

https://pbs.twimg.com/media/D-IuUuqVUAALnAB.jpg

https://twitter.com/Fu_L12345654321/status/1144528199654633477
(deleted an unsolicited ad)

368 :132人目の素数さん:2019/06/30(日) 16:58:33.04 ID:nbUDy6dD.net
大数スレは↓ですよん。
http://rio2016.5ch.net/test/read.cgi/math/1395323240/

369 :132人目の素数さん:2019/07/03(水) 19:43:27.28 ID:dqLWAG/2.net
4330
ふうL@Fu_L12345654321
学コン1傑いただきました!
とても嬉しいです!

https://pbs.twimg.com/media/D-IuUuqVUAALnAB.jpg

https://twitter.com/Fu_L12345654321/status/1144528199654633477
(deleted an unsolicited ad)

370 :132人目の素数さん:2019/07/10(水) 02:12:15.35 ID:miBhNnEC.net
さて、10日でござる。
今年は梅雨入りが遅くて、まだ明けませぬ。鬱陶しい・・・・

>>360
水戸黄門 第2部 (BS-TBS版)
第一話 5/19 中津 (大分県)
第二話 5/26 朝倉 (福岡県)
第三話 6/02 日田 (大分県)
第四話 6/16 延岡 (宮崎県)
第五話 6/23 宮崎
第六話 6/30 鹿児島
第七話 7/07 長崎
第八話 7/14 佐賀

371 :132人目の素数さん:2019/07/10(水) 19:45:08.15 ID:+JfXx3Us.net
7月号は良問でしたね

372 :132人目の素数さん:2019/07/11(木) 07:16:40.46 ID:uF4uMgUV.net
ニッコリ再びか

373 :132人目の素数さん:2019/07/11(木) 21:35:49.79 ID:csydILa4.net
8月号は近大数コンの宣伝とプログラミング演習か。

374 :132人目の素数さん:2019/07/12(金) 00:24:00.21 ID:wFPQ8Nqi.net
2019年7月号

■出題1
乗算×に対して分配上位
  x * (y×z) = (x*y) × (x*z)
である演算 * と、加算+に対して分配下位
  x + (y o z) = (x+y) o (x+z)
である演算 o とを見つける問題。

x * y = a^{log_a(x)×log_a(y)}     (a>0, a≠1 なる定数)
単位元:a
とすると、定義域が x>0, y>0 になってしまう。

 x o y = max{x, y}
は加法よりも低レベルな演算で「準加算」と呼ぶらしいが、単位元がうまく出ない。
 x o y = min{x, y} としても同じだろうけど。
日曜数学会 (2016)

375 :132人目の素数さん:2019/07/12(金) 21:08:59.61 ID:BmzvzBnS.net
max でも定義域を x>=0, y>=0 にしたら単位元を 0 とすることはできる。
定義域を R∪{-∞} として単位元を -∞ とするってのは反則かなあ。

376 :132人目の素数さん:2019/07/13(土) 02:26:50.59 ID:VYoufByK.net
2019年7月号

■出題2

(1)それ自身と異なるどのような順列にも変換できないような順列の例

3ずつ減らせば(増やせば)よい。
 ( ・・・・, 5, 2, ・・・・, 4, 1, ・・・・, 6, 3)

例)
n≦6  なし
n=7  (5, 2, 7, 4, 1, 6, 3)
n=8  (8, 5, 2, 7, 4, 1, 6, 3) など
n=9  (8, 5, 2, 7, 4, 1, 9, 6, 3) など

377 :132人目の素数さん:2019/07/13(土) 10:38:58.08 ID:Jh+tNa4u.net
■出題2の(2)はゴミゴミした帰納法で突破できた

378 :132人目の素数さん:2019/07/14(日) 04:32:39.12 ID:CCtjKtiD.net
今月号の出題1はかなり簡単。

379 :132人目の素数さん:2019/07/17(水) 05:22:53.70 ID:d6A0g1cK.net
>>345 >>346 >>347
5月号

■出題2 (3)

正12面体Uの頂点の位置は
 (0, ±g/√3, ±1/(g√3))
 (±1/(g√3), 0, ±g/√3)
 (±g/√3, ±1/(g√3), 0)

 (±1/√3, ±1/√3, ±1/√3)
正20面体Vの頂点の位置は
 (±1/√(g√5), ±√(g/√5), 0)
 (0, ±1/√(g√5), ±√(g/√5))
 (±√(g/√5), 0, ±1/√(g√5))
としました。ただし
 g = (1+√5)/2 = 1.618034    黄金比
UとVの対称面を揃えているので消滅則が成り立つはず。

一方、8月号の解説では UとVの5回軸を揃えたようですが、
いずれにしても、多項式fが9次以下の場合はI(f)と一致しそうですね。(浦安市 K氏)

380 :132人目の素数さん:2019/07/17(水) 05:56:57.78 ID:d6A0g1cK.net
Cubature公式とか云うらしい。

http://www.comb.math.keio.ac.jp/wakate13/abstract.pdf
http://www.jstage.jst.go.jp/article/sugaku/68/1/68_0681024/_pdf/-char/ja

381 :132人目の素数さん:2019/07/17(水) 06:09:20.80 ID:bqGXGpsH.net
カボチャ公式?

382 :132人目の素数さん:2019/07/19(金) 06:22:33.69 ID:f4PYd0eY.net
>>379
5月号

■出題2 (3)

8月号掲載では、5回軸を揃えています。これをz軸とすれば
 (x, y, z) = (sinθcosφ, sinθsinφ, cosθ)

正20面体Uの12頂点
 θ1 = 0,
 θ2 = arctan(2),  φ = 2jπ/5,
 θ3 = π - θ,   φ = (2j+1)/5,
 θ4 = π,

正12面体Vの20頂点
 θ1 = arctan(3-√5),  φ = (2j+1)/5,
 θ2 = arctan(3+√5),  φ = (2j+1)/5,
 θ3 = π - θ2,    φ = 2jπ/5,
 θ4 = π - θ1,    φ = 2jπ/5,

ただし j = 0,1,2,3,4 です。

383 :132人目の素数さん:2019/07/20(土) 11:00:35.84 ID:bSAoQnjE.net
0045
ふうL@Fu_L12345654321
学コン1傑いただきました!
とても嬉しいです!

https://pbs.twimg.com/media/D-IuUuqVUAALnAB.jpg
https://twitter.com/Fu_L12345654321/status/1144528199654633477
(deleted an unsolicited ad)

384 :132人目の素数さん:2019/07/21(日) 22:41:07.82 ID:4k/Gtesi.net
ご老公「助さん格さん、懲らしめてやりなさい!」 >>367 >>369 >>383

 (荒らし同士で言い争ってもしかたねぇか・・・・)

385 :132人目の素数さん:2019/07/22(月) 20:02:21.40 ID:vDQA99OD.net
>>341

t次以下の任意の多項式 f(x,y,z) について
 Σ[P∈X] w(P)・f(P) = I(f),
 Σ[P∈X] w(P) = 1,
が成り立つとき、
(X,w) を weighted spherical t-design と呼ぶらしい。

〔Fisher型 不等式〕
 |X| ≧ ([t/2]+1)・([(t+1)/2]+1)

 Delsarte-Goethals-Seidel (1978) ・・・・ 等重率( w(P)=1/|X| )の場合。

(例)
 t=2  ≧ 4点 (正4面体)
 t=3  ≧ 6点 (立方体、正8面体)
 t=4  ≧ 9点
 t=5  ≧ 12点 (正12面体、正20面体、立方体+正8面体)
 t=6  ≧ 16点
 t=7  ≧ 20点
 t=8  ≧ 25点
 t=9  ≧ 30点 (正12面体+正20面体)

>>380
 △f = 0  (調和多項式) ⇒  I(f) = 0.

386 :132人目の素数さん:2019/07/23(火) 02:30:17.74 ID:EWg30VSr.net
>>379>>382 も サッカーボール/フラーレン の32面の中心ですね。

つまり U+V は サッカーボール (v=60, e=90, f=32) を反転したもの。

J.M.Goethals & J.J.Seidel, Nieuw Arch. Wisk., 29, p.52 (1981)
 "The football"

387 :132人目の素数さん:2019/07/28(日) 12:59:59.93 ID:YMtuI2+B.net
正12面体も正20面体も 頂点と面を合わせて32個ある
∴ 正20面体の頂点を切り落とすと32面になる。(サッカーボール / フラーレン)
http://www.shimanet.ed.jp/minami/link/homepage-naga005/grapes-001/tamentai-6-4-8-20-12-6.pdf

正12面体は立方体の8頂点 (±1/√3, ±1/√3, ±1/√3) を含む。
これを3軸にとったのが >>379 のU

388 :132人目の素数さん:2019/07/28(日) 21:53:29.96 ID:MjwRFhds.net
>>372
焼き直し、だけでなく簡単化とは

389 :132人目の素数さん:2019/07/29(月) 18:45:49.79 ID:6L7yKrQP.net
わしの勝ちぢゃな。最初からのニッコリがこれより長く連続することはなかろうて。

390 :132人目の素数さん:2019/07/29(月) 22:26:00.96 ID:diykrDV4.net
読者の解答をまともに読んでないことが分かっちゃった人

391 :132人目の素数さん:2019/08/05(月) 08:04:42.28 ID:3e9wpFpc.net
>>378
まじすか

392 :378:2019/08/09(金) 06:31:59.54 ID:O+R8cMNd.net
8月号の出題1

n次の円分多項式をf(n:x)、簡単のためexp{2πi(n/m)}=ex(n;m)と書くことにする。

一般化して、以下の命題を考える。
「qを奇素数、mをm≡-1 (mod q)をみたす2以上の整数、kを正の整数とする。
f(q:m^k)=pの値が素数となるとき、以下の合同式が言える。
m^(m^k +1)≡1 (mod p)がいえる。」…※

kがqのべき乗ではないと仮定する。
kは0以上の整数jと2以上でかつqと互いに素な整数sを用いてk=q^j・sと書ける。
q^(j+1)より小さく、かつqで割り切れない正の整数rを任意にとるとき、
f(q^(j+1):{ex(r;q^(j+1) )}^s)=f(q^(j+1):{ex(rs;q^(j+1) )})=0
したがって、f(q^(j+1):x^s)はf(q^(j+1):x)で割り切れることがいえる。

よって、f(q:m^k)=f(q^(j+1):m^s)はf(q^(j+1):m)で真に割り切れるから、
f(q:m^k)の値が合成数となることがいえるが、これは不合理。
よってkはqのべき乗であることがいえるので、k=q^jとかける。

x≡-1 (mod q^h) (ただし、hは正の整数)がいえるとき、x^(q-1)-x^(q-2)…+1≡q≡0 (mod q)
より、x^q +1=(x+1){x^(q-1)-x^(q-2)…+1}≡0 (mod q^(h+1) )がいえるので、
帰納的に、m^k +1≡m^(q^j) +1≡0 (mod q^(j+1) )がいえる。

明らかにm^{q^(j+1)}≡1 (mod p)であることがいえるので、このことより
m^(m^k +1)≡1 (mod p)が示された。

よって、※の命題はいえた。

393 :378:2019/08/09(金) 06:38:12.42 ID:O+R8cMNd.net
あとは、>>392であげた※の命題の前提条件が成立することを確かめるだけなのだが
m=1のときは明らか、m>1のとき、m≡1となると仮定すると、m^(2k) +m^k +1≡3≡0 (mod 3)より
m^(2k) +m^k +1の値が合成数となってしまうので、m≡-1 (mod 3)であることがいえる。
よって、>>392であげた※の命題の前提条件は成立する。

394 :132人目の素数さん:2019/08/09(金) 14:37:09.64 ID:ZLWTEHzz.net
q>3 のときは
f(q:m^k) が素数という条件から m≡-1 (mod q) は出ないと思われ。

q=5, m=2, k=1
f(5:2) = 2^5 - 1 = 31,

q=7, m=2
f(7:2) = 2^7 - 1 = 127,
f(7:2^7) = 4432676798593,

* qが奇素数のとき f(q:x) = (x^q -1)/(x-1) = x^(q-1) + x^(q-2) + ・・・・ + x + 1,

395 :132人目の素数さん:2019/08/09(金) 17:46:29.14 ID:3e9+Hf+g.net
あの式を3次の円分多項式と見る発想が出てこないんだよなあ
お見事です

396 :132人目の素数さん:2019/08/10(土) 00:23:16.03 ID:yw88vPvQ.net
kがべき乗であることを示すところが難しかった

397 :132人目の素数さん:2019/08/10(土) 17:44:49.82 ID:ttJXucaK.net
今までの出題で印象にのこってる問題って何かある?

398 :132人目の素数さん:2019/08/20(火) 19:39:15.33 ID:RMnkNk3x.net
今月の出題2だが、自力で解くにせよ、解答を失敬するにせよ
その「解答」に納得することはなさそうw

399 :132人目の素数さん:2019/08/23(金) 21:06:55.75 ID:FOOrdYVG.net
意味深だねー
まだ問題読んでないけど

400 :132人目の素数さん:2019/08/24(土) 08:19:59.89 ID:tClIWhSz.net
>>386

U+V の各頂点を通る法平面からなる32面体がサッカーボール/フラーレンですね。
32面が同一球面に外接するだけでなく、60頂点が同一球面に内接します。
稜の長さの比は sin(24゚) / sin(36゚) = {√(3+6/√5) -1}/2 = 0.6919817 です。

  sin(24゚) = sin(60゚-36゚) = {(√3)/2}cos(36゚) - (1/2)sin(36゚) = 0.406737
  sin(36゚) = (1/4)√{2(5-√5)} = 0.587785
  cos(36゚) = (1/4)(1+√5) = 0.809017

401 :132人目の素数さん:2019/08/24(土) 23:30:26.42 ID:GgnTyOYD.net
問1だけど、たとえば頂点のみを通った場合も無傷でないと見做すのか?平面の厚みはゼロでないから

402 :132人目の素数さん:2019/08/25(日) 02:53:29.70 ID:o6vREx4x.net
「薄い銀紙で一つずつ包装されている・・・・」 → 薄いとはいえ、有限の厚さをもつ。

「平面の厚さはゼロに限りなく近いとする。」

これから判断すると、銀紙の厚さ > 平面の厚さ

頂点のみを通る平面で切った場合は、銀紙が擦り減るだけで、中のチーズは無傷と思われます。

403 :132人目の素数さん:2019/08/25(日) 07:10:06.57 ID:mHsMRDBl.net
銀紙!そこ読み飛ばしてたw

404 :132人目の素数さん:2019/08/25(日) 11:04:01.35 ID:o6vREx4x.net
>>400
 cos(36゚) = (1+√5)/4 = g/2, 
 tan(36゚) = √(5-2√5) = {5^(1/4)}/g^(3/2) = 0.726542528

稜の長さ
 5角形と6角形の境界は
  (4/g)sin(36゚) tanα = 2 tan(36゚) tanα = 0.491522313
 6角形どうしの境界は
  (4/g)sin(24゚) tanα = {2/√(g√5)} (1-2tanα) = 0.34012445
 ここに α = (1/2)arctan(3-√5),
 tanα = (1/2){√(3g√5) - gg} = 0.33826121
 g = (1+√5)/2 = 1.618034

外接円の半径は
 R = √{1 + [(2/g)tanα]^2} = 1.0838907667

405 :132人目の素数さん:2019/08/25(日) 12:46:22.65 ID:WkOS+mAP.net
ピーター良問だな

406 :132人目の素数さん:2019/09/04(水) 11:54:26.66 ID:b58s+zSY.net
>>404

単位球に内接する正20面体をVとする。
30本の稜の長さは
 L = (4/√5)sin(36゚) = 1.05146222424
中心〜20の△面までの距離は
 h = √{(2+√5)/(3√5)} = 0.7946544723

さて、Vの12の頂点を深さdまで切り落とそう。(切頂20面体)
切り口は正5角形である。
中心〜頂点の距離は gd で、辺の長さは
 gd・2sin(36゚) = 1.9021130326d ・・・・5角形と6角形の境界
残った稜の長さは
 L{1 - (√5)gd}   ・・・・ 6角形どうしの境界
中心〜60頂点の距離 (外接円の半径) は
 R = √{(1-d)^2 + (gd)^2},

ここで中心〜32面までの距離が等しくなるようにすると
 d = 1 - h = 0.2053455277
 L{1 - (√5)gd} = gd・2sin(24゚) = 0.27028141536
 R = 0.861318645231

407 :132人目の素数さん:2019/09/04(水) 18:22:38.63 ID:b58s+zSY.net
>>406
Vの隣り合う2頂点 P1, P2 に対して
 OP = 1,
切頂20面体 (32面体) の頂点は
 ↑Q1 = {1-(√5)gd/2} ↑P1 + {(√5)gd/2} ↑P2,
 ↑Q2 = {(√5)gd/2} ↑P1 + {1-(√5)gd/2} ↑P2,
 R = OQ

半径hの球面に外接するとき (フラーレン)
 ↑Q1 = 0.62852645 ↑P1 + 0.37147355 ↑P2,
 ↑Q2 = 0.37147355 ↑P1 + 0.62852645 ↑P2,
 R/h = 1.0838907667

408 :132人目の素数さん:2019/09/08(日) 12:48:56.70 ID:f5Vr9RPx.net
問1は中心を外すと途端にわからなくなる

409 :132人目の素数さん:2019/09/08(日) 22:47:02.21 ID:f5Vr9RPx.net
辛い月だったピーターめこんちくしょう

410 :132人目の素数さん:2019/09/09(月) 01:48:52.91 ID:GeJx+nRu.net
>>384
「水戸黄門」(第2弾) も終わっちまった。こんちくしょう
http://www.cal-net.co.jp/mitokomon/02/

411 :132人目の素数さん:2019/09/09(月) 21:07:24.27 ID:mibJjB0T.net
出題2は面白かった。いろんな意味で。
編集部から二度に渡ってヒントを出せと言われたらあんな書き方になるのかなと

412 :132人目の素数さん:2019/09/09(月) 21:08:12.24 ID:mibJjB0T.net
ピーター先生の問題は解ききれず無念

413 :132人目の素数さん:2019/09/10(火) 00:55:49.96 ID:dcTq0f7X.net
ピーター解けた人エレガントなコメもとむ

414 :132人目の素数さん:2019/09/10(火) 02:35:50.70 ID:yUERuKgH.net
2019年9月号

■出題1
・問題の平面がある軸 (z軸としよう) に平行のとき
 これをxy平面に投影すると直線になる。
 この直線は 箱の表面と2回、チーズ同士の境界(6面) と1回づつ、最大でも8回しか交差しない。
∴ 生じる線分は7個以下、チーズ7個/段 以下 しか切れない。
 各段に9個以上が無傷で残る。(計 36個以上)

・平面が x軸、y軸、z軸のどれにも平行でないとき
 (立方対称から) z = d -ax -by (a,b,d>0) としてもよい。

さて、64個のサイコロを 体対角線方向の組に分類する。
 (1,1,1) - (2,2,2) - (3,3,3) - (4,4,4)

 (1,1,2) - (2,2,3) - (3,3,4)
 (1,2,1) - (2,3,2) - (3,4,3)
 (1,1,2) - (2,2,3) - (3,3,4)

 (1,2,2) - (2,3,3) - (3,4,4)
 (2,1,2) - (3,2,3) - (4,3,4)
 (2,2,1) - (3,3,2) - (4,4,3)

 (1,1,3) - (2,2,4)  (1,3,1) - (2,4,2)  (3,1,1) - (4,2,2)
 (1,2,3) - (2,3,4)  (1,3,2) - (2,4,3)  (2,1,3) - (3,2,4)
 (2,3,1) - (3,4,2)  (3,1,2) - (4,2,3)  (3,2,1) - (4,3,2)
 (1,3,3) - (2,4,4)  (3,1,3) - (4,2,4)  (3,3,1) - (4,4,2)
これら19組46個の他に「単独チーズ」が18個ある。

これらを平面 z = d-ax-by (a,b,d>0) で切ったとき、
切れるチーズは各組に1個以下。    (← これがミソ)
∴ 46個のうち27個以上が無傷で残る。

これで解けたらエレガントな解答になるんだが、
無傷で残るチーズはまだ他にもある…

415 :132人目の素数さん:2019/09/10(火) 02:43:28.18 ID:yUERuKgH.net
そこで次に、箱隅にある6個の単独チーズに着目しよう。
(∵ この6個を1度に切るのは、どう考えても無理っぽい。)
反対向きの
 (1,4,1) と (4,1,4)  (4,1,1) と (1,4,4)  (1,1,4) と (4,4,1)
の3ペアに分ける。
平面 z = d-ax-by := f(x,y) が上記の2ペア、たとえば
 (1,4,1) と (4,1,4)  (4,1,1) と (1,4,4)
の4個を切ったとすると
 f(1,4) < 1, f(3,0) > 3, f(4,1) < 1, f(0,3) > 3,
fは線形だから
 f(1,1) = {3f(3,0)+3f(0,3)-f(4,1)-f(1,4)} / 4 > 4,
 f(3,3) = {3f(4,1)+3f(1,4)-f(3,0)-f(0,3)} / 4 < 0,
⇒ 他のペア (1,1,4) と (4,4,1) は無傷で残る。

いま 「この平面が箱隅の6個の単独チーズのうち 5個以上を切った」と仮定する。
 その5個は 上記の3ペアのうち2ペアを含む。
∴ 他ペアの2個は無傷で残る。(矛盾)
∴ 箱隅の6個の単独チーズのうちの2個以上が無傷で残る。

以上から、計29個以上が無傷で残ることが分かる。

あとは 29個だけが無傷となる(35個切れる)ような実例を挙げる。

416 :132人目の素数さん:2019/09/10(火) 03:01:22.31 ID:yUERuKgH.net
サイコロ・スレ
http://rio2016.5ch.net/test/read.cgi/math/1487760195/123-132
http://rio2016.5ch.net/test/read.cgi/math/1483161590/208-217

417 :132人目の素数さん:2019/09/10(火) 17:47:37.02 ID:yqgHpuKY.net
平面の問題だったら簡単になるから切り口に注目して解いた

418 :132人目の素数さん:2019/09/11(水) 23:55:49.71 ID:1IJ7FHVd.net
>>414
 突如サイコロになるwww

>>417
 切り口に注目して簡単になる?

419 :132人目の素数さん:2019/09/12(木) 00:35:54.30 ID:Eos6rtl2.net
チーズと解答に書きたくない気持ちは分かる。cubeでいいじゃん。でも銀紙の設定は秀逸

420 :132人目の素数さん:2019/09/13(金) 05:33:53.87 ID:wXRdciyr.net
>>418
無限に長い16個の正四角柱を斜めに切断した後で水平方向に5回切るって考えた。
こうすると切り口が4x4斜方格子になってそこに5本平行線を通す問題になる。
(切り口の多角形の数=切られたチーズの数)

421 :132人目の素数さん:2019/09/13(金) 12:38:12.82 ID:Bk4U8Cej.net
なるへそ

422 :132人目の素数さん:2019/09/16(月) 04:04:44.09 ID:FF+PWEgn.net
>>374
 x o y = log_a(a^x + a^y)  もあります。
 ただし a>0, a≠1.

>>375
 鋭い御指摘です。この場合は定義域を拡大する必要がありますね。
 (10月号の解説を参照)

>>376
 (3[(n-2)/3]+2, …, 5, 2, 3[(n-1)/3]+1, …, 4, 1, 3[n/3], …, 6, 3)
のほかに
奇数と偶数を交互に並べる方法もあるようです。それぞれ2ずつ増やします。
・nが奇数のとき
 (5, 2, 7, 4, …, n, n-3, 1, n-1, 3)
・nが偶数のとき
 (5, 2, 7, 4, …, n-1, n-4, 1, n-2, 3, n)

423 :132人目の素数さん:2019/09/16(月) 19:07:36.73 ID:y83x4sKL.net
7月号出題1は単位元-∞を認めるのが題意だったみたいですね。
でもこの場合の-∞における連続性ってどう議論したらいいんでしょう。

424 :132人目の素数さん:2019/09/16(月) 23:55:35.31 ID:FF+PWEgn.net
>>411
 2番目のヒントは無い方がいい鴨
 X_α の各要素の桁数が不明では使いにくいとオモタ

425 :132人目の素数さん:2019/09/17(火) 01:07:52.42 ID:bUA1Q0Od.net
>>424
自分もXは使わなかった。しかしヒントにはなった
Xの存在が過去問題になったってホント?
ものの数行で存在を示せるキガス

426 :132人目の素数さん:2019/09/17(火) 01:30:54.40 ID:eD/9WN7h.net
(例)
αのn番目までの値を逆向きに並べて2進数とみなせば
   X_α(n) = Σ[k=1,n] {α(k)-1}・2^(k-1),
n桁 (2進法) になる。

427 :132人目の素数さん:2019/09/18(水) 00:04:46.18 ID:Pu45bTZg.net
2019年9月号    遅くなったけど・・・・

■出題2
題意の条件を満たす実関数f(x)の存在を示す問題。
1番目のヒントに従えば、
α∈B に対応する集合族 A_α ⊆ R が存在。 (← 条件1,2)
α → x ∈ (A_α⊆) R の対応は単射。  (← 条件4)
∴ 値域 R~(⊆ R) からBへの逆写像 h: R~→B が存在。
全射g: B→R をもって来て
 f(x) = g(h(x))  (x∈R~)
   = 0     (x∈R-R~)
とすればf(x)も全射で題意を満たす。  (← 条件3)
それでは、1番目のヒントを示そう。

428 :132人目の素数さん:2019/09/18(水) 00:15:51.35 ID:Pu45bTZg.net
条件3 から考えると、最初に
 A_α = { c(α)+r | r∈Q }
の形のものが浮かぶ。つまり
 x〜y  ⇔  x-y∈Q
で定まる同値関係〜による同値類である。
ただし c(α) = Σ[k=1,∞] (α_k -1)/2^k とやると
α≠β であっても α_k≠β_k なるkが有限個ならば
 c(α)〜c(β) で 条件4を満たさない。
α≠β なら α'_k≠β'_k なるkが無限個になる必要がある。
そこで αの随伴数列 α'∈B を
 α1; α1, α2; α1, α2, α3; α1, α2, α3, α4; ・・・・
と定義しよう。その上で
 c(α) = Σ[k=1,∞] (α'_k -1)/2^k,
とおくと a-b は ±Σ (1/2)^(三角数+m) のような和になり、
たぶん無理数だと思うけど・・・・
いつも通りだが上手くいかない...orz

429 :132人目の素数さん:2019/09/18(水) 00:57:07.09 ID:Pu45bTZg.net
しょうがねゑ。
 c(α) = Σ[k=1,∞] 10^(-k!)・(α'_k -1),  ・・・・ 欠項リウヴィル数
とおこう。
 c(α) - c(β)
も欠項リウヴィル数、したがって超越数。。。

430 :132人目の素数さん:2019/09/18(水) 02:00:31.63 ID:XJhzfOw4.net
出題2ってヒント無視したらいくらでも作れそうだけど。

例えば十進展開した時あるNが存在して小数第N位は5、それ以降は4か6しか出ない数xをしごろ数とか呼ぶとし、
s(x)をそのしごろ数の出だし、出だし以降の桁を全部5に変えた数をその軸c(x)と呼ぶとする。
A(x)={y | c(y) = c(x)}の最小値をa(x),最大値をb(x)、
C(s)={c(x) |xはしごろ数,s(x)>s}とでも置く。
容易に任意のsに対してC(s)は稠密でc(x)がC(s)に属するとき、c(x)-a(x),b(x)- c(x)<10^(-s)だから、与えられた区間(a,b)に対してしごろ数xをA(x)⊂(a,b)を満たすようにとれる。
一方でA(x)は連続体無限である事と、相異なるc(x)とc(y)に対してA(x)とA(y)はdisjointだから各A(x)に制限したfの値域がR全体になるように割り当てられる。

とかじゃダメかな?

431 :132人目の素数さん:2019/09/18(水) 03:44:28.32 ID:Pu45bTZg.net
なるほど。
Qを丸々同値類にするのは (いくら可算とは言え) ダメか。
もっと細分すべき哉。

432 :132人目の素数さん:2019/09/18(水) 15:38:01.02 ID:Pu45bTZg.net
>>430
しごろ数x∈ R~ (⊆R) を
s(x)-1より上の桁: [10^(s(x)-1)・x]・(1/10)^(s(x)-1) と
s(x)+1より下の桁: {10^(s(x)-1)・x}・(1/10)^(s(x)-1) に分けるのでござるな。
(s(x)の桁は5)

上の桁が同じもの同士をまとめて A(x) とする。
下の桁を (1/10)^s(x)・Σ[k=1,∞] (2α_k -3)(1/10)^k
と表わしてα(x)∈B を定める。

全射g: B→R を持ってきて
 f(x) = g(α(x)))     (x∈R~)
    = 0        (x∈R-R~)
とおく。

433 :132人目の素数さん:2019/09/19(木) 14:05:36.76 ID:/rMlpsDI.net
しごろ数xでは「4」「6」以外は有限個であり、「5」の最下位をs(x)とする。
s(x)より上の桁は可算無限個(〜N)あり、有理数の組(a,b)への全射がある。
s(x)より下の桁はBと同型(〜2^N)であり、Rへの全射gがある。

上も下も2種以上の数字が必要で、s(x)を決定するにはもう1種の数字(5)が必要。

434 :132人目の素数さん:2019/09/19(木) 14:11:17.74 ID:/rMlpsDI.net
>>432 訂正

下の桁を (1/10)^s(x)・Σ[k=1,∞] (2α_k +2) (1/10)^k
と表わしてα(x)∈B を定める。

435 :132人目の素数さん:2019/09/20(金) 13:26:25.45 ID:KyAOfC1j.net
2630
かずきち@dy_dt_dt_dx 8月28日
学コン8月号Sコース1等賞1位とれました!
マジで嬉しいです!
来月からも理系に負けず頑張りたいと思います!
https://twitter.com/dy_dt_dt_dx
(deleted an unsolicited ad)

436 :132人目の素数さん:2019/09/21(土) 19:21:04.02 ID:Nou2F8U6.net
>>374

n次正方行列 A,B に対して「準加算」A o B = C を
 C(j,k) = Max{min{A(j,1), B(1,k)}, min{A(j,2), B(2,k)}, ・・・・・, min{A(j,n), B(n,k)}}
で定義します。
単位行列Eも、普通のように、主対角線上の元がすべて1で、他はすべて0 と定義します。
n次正方行列Aに対して
 A o B = E
となるn次正方行列Bが存在するための、Aに対する条件を求めて下さい。

数セミ増刊「数学の問題」第(3)集、日本評論社 (1988) ●8

1以上の元は各行、各列にちょうど1つずつある。その他の元は0以下である。

437 :132人目の素数さん:2019/09/29(日) 08:27:53.78 ID:yMiUWc4N.net
>>407

正20面体Vを切頂してフラーレンを作ろう。
稜 P1-P2 を
 0.628526450656913 : 0.371473549343087
の比に内分した点を Q1 とすると
 R = OQ1 = 0.8613186452310
これを単位球面まで延長すると
 1.16101051049675↑Q1
 = 0.729725815337893↑P1 + 0.431284695158857↑P2,

438 :132人目の素数さん:2019/10/01(火) 11:56:00.78 ID:9+EG76aR.net
出題2 (1) で N=0 も許すと、無条件で成立つことになるが…

(例)
 x(n,0) = (1/2)^n のとき x(n,t) = x(n,0)(3/4)^t

439 :132人目の素数さん:2019/10/03(木) 01:45:32.21 ID:R/Jf3zn8.net
>>379 >>382
正20面体Vの12頂点 (Uの面心)
正12面体Uの20頂点 (Vの面心)
それらの30稜の中点からなる32面体G
について
 V(f), U(f), G(f)  ・・・・ 5次以下

 (5/14)V(f) + (9/14)U(f),
 (5/21)V(f) + (16/21)G(f)  ・・・・・ 9次以下

 (125/462)V(f) + (81/462)U(f) + (256/462)G(f) ・・・・・ 11次以下

440 :132人目の素数さん:2019/10/03(木) 11:44:59.68 ID:R/Jf3zn8.net
>>439

Gの30頂点の座標は 5回軸の回りの極座標で表わせば
 θ = arctan(1/g), arctan(g), π/2, π-arctan(g), π-arctan(1/g),
 φ = mπ/10, (m:整数)

441 :132人目の素数さん:2019/10/10(木) 08:31:45.08 ID:OMt0mHKD.net
11月号問2はほとんど物理の問題じゃね?前回は組体操の問題を出した人だし。

442 :132人目の素数さん:2019/10/10(木) 21:41:30.00 ID:4MNDsrsX.net
てことで、10月号のことは忘れよう。。。

443 :132人目の素数さん:2019/10/11(金) 16:07:06.01 ID:Q+QVtJOo.net
>>407 >>437
 正20面体V (12頂点),
 Vを切頂したフラーレンF (60頂点),
 Vの30稜の中点からなるG (30頂点),
について
 V(f), F(f), G(f) ・・・・ fが5次以下ならI(f)と一致

 (5/21)V(f) + (16/21)G(f),
 0.9379003725443 F(f) + 0.0620996274557 V(f),
 1.2688362607403 F(f) - 0.2688362607403 G(f),
         ・・・・ fが9次以下ならI(f)と一致

 2.76605664248605 G(f) + 0.70104277651910 U(f) - 2.46709941900515 F(f) 
         ・・・・ fが11次以下ならI(f)と一致

444 :132人目の素数さん:2019/10/12(土) 02:54:12.10 ID:hFZo15fW.net
↑のU(f)は誤り。V(f)が正しい。

>>439>>443 を組合せて
1.3507153208422 {125 V(f) +81 U(f) +256 G(f)}/462
- 0.3507153208422 {2.76605664248605 G(f) +0.70104277651910 V(f) -2.46709941900515 F(f)}
   ・・・・ fが15次以下ならI(f)と一致

445 :132人目の素数さん:2019/10/12(土) 12:06:57.39 ID:z18YnhgS.net
10月号の話をしよう
俺は2問目は出来たぜ
1問目は出来なかったぜ

446 :132人目の素数さん:2019/10/12(土) 12:09:08.62 ID:z18YnhgS.net
11月号2問目は何を答えりゃいいの?ネットに答えは出てるし

447 :132人目の素数さん:2019/10/12(土) 23:05:25.46 ID:hFZo15fW.net
「指ハブ」に指を差し込むと、引っ張っても抜けない・・・・その理由

448 :132人目の素数さん:2019/10/12(土) 23:48:25.29 ID:0obxgbFd.net
構造を知ろうとネットで検索すると答えも一緒に見つけちゃうんだよな
「数学で説明」ってのが何を意図してるのかさっぱり
「物理で説明」くらいに思っておいていいんだろうか

449 :132人目の素数さん:2019/10/13(日) 02:35:00.45 ID:6F2PPbdU.net
数学で説明するフリをすれば「正解者」に入れまつよ。

450 :132人目の素数さん:2019/10/15(火) 18:21:48.81 ID:eja156vF.net
>>407 >>437

正20面体Vの頂点P1,P2
∠P1-O-P2 = atan(2),
cos(∠P1-O-P2) = 1/√5,
OQ^2 = (3+√5)d - (1/3) = 0.7418698086225
R = OQ1 = 0.8613186452310

(2-(√5)gd)/2 = {√((25+11√5)/6) - g}/2
  = 0.628526450656913
(√5)gd/2 = {g√5 - √((25+11√5)/6)}
  = 0.371473549343087

(2-(√5)gd)/(2R) = √{[815 +73√5 -√(6(25345-11299√5))]/(4・449)}
  = 0.72972581533788
(√5)gd/(2R) = √{[1625 -185√5 -√(30(12905+5701√5))]/(4・449)}
  = 0.43128469515885

Q1・Q2 = 3(5-√5)(1-d)^3 - (5/3) - 4/√5
  = 0.70534378687741

cos(∠Q1-O-Q2) = 0.95076491680804

451 :132人目の素数さん:2019/10/16(水) 13:28:27.31 ID:5dVhgqq0.net
>>450 訂正
まん中あたり
(√5)gd/2 = {g√5 - √((25+11√5)/6)}/2 = ・・・・

452 :132人目の素数さん:2019/10/18(金) 12:33:03.41 ID:J/lPU/wp.net
11月号出題1は正直面白かった。みんなこれぐらいならスラスラ解けるのか?

453 :132人目の素数さん:2019/10/21(月) 04:57:15 ID:m5R6mUaJ.net
2019年10月号

■出題2
(1) N=0 を許すと無意味な条件になる。 >>438
 N≠0 と改変すべきか?

(2)
 N≦n ならば x(n,t) = α  (t≧0)
 x(N-1,t) = x(N-1,1)  (t≧1)
 ・・・・
 x(N-k,t) = x(N-k,k)  (t≧k)
 ・・・・
 同様にして x(n,t) は t=n までに確定する。
 しかし nは -∞ まで伸びている。
 すべてのnで一斉に確定するような有限な T はあるか?

(min[a,b] はa,bのうち大きくない方を表わし、a,bについて単調非減少である。)

454 :132人目の素数さん:2019/10/21(月) 05:10:00.73 ID:m5R6mUaJ.net
>>453
同様にして x(n,t) は t=max[N-n,0] までに確定する。

455 :132人目の素数さん:2019/10/22(火) 15:46:45.35 ID:o5l85dPN.net
>>453
いかなるNに対しても(1)を仮定すれば題意が満たされること、を示す

出題は何もおかしくないと思われ

456 :132人目の素数さん:2019/10/22(火) 16:12:14.60 ID:nYvyjN1O.net
いや出題はある整数Nに対してだよ。
もしこのままだとN=0に対してのみ成立してる条件下でも証明しないといけなくなる。

457 :132人目の素数さん:2019/10/22(火) 17:08:04.99 ID:o5l85dPN.net
周期Nが何であれ、周期性があるなら題意を満たすことを示せ、ってことですよ

あなたのいうとおりN=0は自明。他のすべてのNで示せればOK

458 :132人目の素数さん:2019/10/22(火) 17:09:31.08 ID:o5l85dPN.net
ああごめん笑 N=0はだめだね。勘違いしたよすまそ。でも誰もN=0なんて考えもしないんじゃ。君以外。

459 :132人目の素数さん:2019/10/22(火) 17:17:53.49 ID:o5l85dPN.net
単に正の条件を書き損ねただけかと
N=0のケースは馬鹿らしくて考えもしなかったです

>(1) t=0 において,ある整数 N が存在し,任意の n に対して x^0_{n+N}=x^0_{n}が成り立つ.

460 :132人目の素数さん:2019/10/22(火) 17:28:45.60 ID:nYvyjN1O.net
>>458
オレは>>453さんではないよ。
N=0のケースが考えから抜けたのは筆が滑ったんだろうし、目くじらたてるようなもんでもないがもちろん書くべきだし>>453さんの指摘はもっともだろ?

461 :132人目の素数さん:2019/10/22(火) 17:40:25.82 ID:o5l85dPN.net
うむ。

462 :132人目の素数さん:2019/10/25(金) 08:36:11.71 ID:onRxBb1C.net
>>394
8月号の出題1

2以上の自然数mと自然数kに対して
 (m^k)^2 + (m^k) + 1
は素数であるとする。

m^k≡1 (mod 3) とすると、与式が3の倍数となるので不可。
∴ m≠1 (mod 3) かつ kは奇数 とする。
また、円分多項式の議論から k=3^e となる。
(ただし、これらは必要条件であって、十分ではない)
このとき
m≡-1 (mod 3) ならば m^(m^k +1) ≡ 1 (mod p)
m≡0 (mod 3) ならば m^(m^k) ≡ 1 (mod p)
となるか

http://rio2016.5ch.net/test/read.cgi/math/1468160365/155-157

463 :132人目の素数さん:2019/10/26(土) 08:27:04.27 ID:0A6cR2+S.net
10月号の出題2を解いた人いる?

464 :132人目の素数さん:2019/10/27(日) 03:50:48.88 ID:nRsaMl4S.net
出題2
すべてのnで一斉に x(n,t) が確定するような有限な T はあるか?
(min[a,b] はa,bのうち大きくない方を表わし、a,bについて単調非減少である。)
>>453

465 :132人目の素数さん:2019/10/27(日) 04:02:58.99 ID:nRsaMl4S.net
>>462
(m^k)^2 + (m^k) + 1 = p より
m^(3k) = 1 + (m^k -1)p ≡ 1 (mod p)

pは3の倍数でないから、m≠1 (mod 3)
・m≡-1 (mod 3) のとき
 m+1 は3の倍数。
 また m^k +1 が3の倍数ならば
 {m^(k+1) +1}/(m^k +1) = (m^k)^2 -(m^k) +1 = (m^k +1)^2 - 3(m^k) は3の倍数。
∴ k = 3^e のとき m^k +1 は 3k の倍数。
∴ m^(m^k +1) ≡ 1 (mod p)

・mが3の倍数のとき
 m^k は 3^k の倍数。
また 3^k ≧ 3k と k = 3^e から
 3^k は 3k の倍数。
∴ m^(m^k) ≡ 1 (mod p)

466 :132人目の素数さん:2019/10/27(日) 06:38:47.33 ID:xWjyTaHG.net
>>463
こんなのはどう?

M=sup{x(n,0)}, m=inf{x(n,0)}とおく。M<∞、m>0とのき各nにおいてt≧M/mで全てのx(n,t)は定数。

∵) まず以下を機能法で示す。
x(n,t)>x(n,t+1)⇒x(n,t+1)≧m(t+1)。
--
t=0では成立。
t<Tで成立するとしてt=Tのときx(n+1,t-1)=x(n+1,t)、x(n+2,t-1)=x(n+2,t)のいずれも成立するならばx(n,t)=x(n,t+1)である。
いずれかはそうでないときはx(n+1,t),x(n+2,t)のいずれか一方はmt以上であるのでx(n,t+1)≧m(T+1)。
--
特に(n,t)>x(n,t+1)⇒x(n,0)≧m(t+1)であるから待遇をとつてt>x(n,0)/m-1⇒x(n,t+1}=x(n,t)。
特にt>M/m-1⇒x(n,t+1}=x(n,t)により主張は成立する。

467 :132人目の素数さん:2019/10/27(日) 23:35:08.13 ID:B6svoP/0.net
条件もクソもないと、オヌシはそう言われるのじゃな?

468 :132人目の素数さん:2019/10/27(日) 23:36:16.07 ID:8GwELOLx.net
とにかくm,Mが存在すればよいと

469 :132人目の素数さん:2019/10/27(日) 23:46:32.86 ID:WU0BbLNb.net
そうそう。
さすがに条件がなんもないと不等式
1>a+a^2
を満たすaを持ってきて
x(n,0)=a^n
とやってしまうとx(n,t)はtについて狭義単調減少しちゃうからなんか条件入れないとダメだけど(1),(2)の必要条件になってる位にはゆるくて、求める主張がそれ程苦もなく示せる位にはきつい、程よいのはないものかと探してみました。

470 :132人目の素数さん:2019/10/28(月) 01:22:23.21 ID:EcGpIqVn.net
8月号の出題1はヒントが罠な感じがする。
kがどんな数になるか考えろって言ってるけどkの情報特に触らなくてもいいんじゃないか?
----
(m^k)^3≡1 (mod p)

とフェルマーの小定理から

(m^k)^((m^k)^2+m^k)≡1 (mod p)

だからg=(3,(m^k)^2+m^k)とおくとき

(m^k)^g≡1(mod p)

だけど(m,3)=1だったからg=(3,m^k+1)であり、特にg|m^k+1から主張を得る。
----
でよさそうな。

471 :132人目の素数さん:2019/10/28(月) 01:40:16 ID:wyx3fbhK.net
あ、間違った>>470はいらんkが入ってる。
逝ってくる

472 :132人目の素数さん:2019/10/28(月) 07:09:38.84 ID:M55VqgNP.net
フェルマーの小定理から
 m^(p-1) ≡ 1 (mod p)
ですが、指数の
 p-1 = (m^k)(m^k + 1)
は m^k よりかなり大きいので、本問では使いづらいでござる。
 m^(3k) ≡ 1  (mod p)
の方が小さくて使いやすい・・・

473 :132人目の素数さん:2019/10/28(月) 09:44:06.27 ID:M55VqgNP.net
>>466
{x(n,0) | nは整数} の要素は有限個だから、最大値M と 最小値mが存在する。
 0 < m ≦ x(n,0) ≦ M

〔補題1〕
 m ≦ x(n,t) ≦ M,
(略証)
 x(n,t+1) ≧ m は x(n,t) の定義から tについての帰納法で。
 x(n,t+1) ≦ x(n,t) ≦ ・・・・ ≦ x(n,0) ≦ M,

〔補題2〕
 x(n,t) > x(n,t+1) ⇒ x(n,t+1) ≧ m(t+2),

(略証)
tについての帰納法による
まづ定義式と題意から
 x(n,t) > x(n,t+1) = x(n+1,t) + x(n+2,t) ・・・・(*)
t=0 のときは
 x(n,1) = x(n+1,0) + x(n+2,0) ≧ 2m,
t>0 のとき
 x(n+1,t-1) + x(n+2,t-1) ≧ x(n,t) と (*) から
∴ x(n+1,t-1) > x(n+1,t) と x(n+2,t-1) > x(n+2,t)
の少なくとも一方は成り立つ。
帰納法の仮定により
 x(n+1,t) ≧ m(t+1) または x(n+2,t) ≧ m(t+1),
∴ x(n,t+1) = x(n+1,t) + x(n+2,t) ≧ m(t+1) + m = m(t+2),

特に t ≧ [ M/m -1] = T ⇒ x(n,t) = x(n,t+1)

474 :132人目の素数さん:2019/10/28(月) 09:56:57.20 ID:PY+6CwG3.net
小定理使えばこうだった。
----
m^(3k)≡1 (mod p)

とフェルマーの小定理から

m^((m^k)^2+m^k)≡1 (mod p)

だからg=(3k,(m^k)^2+m^k)とおくとき

m^g≡1(mod p)

である。
もしjが3以外と素因子tを持てば

x^2+x+1 | x^2t+x^t+1 (∵ 右の項は x が1の原始ベキ根のとき0)

により

m^(2k/t)+ m^(k/t) + 1 | m^2k+m^k + 1

となり仮定に反するからkは3のベキ。
一方mは3と互いに素だから(m,3k)=1ちよりg=(3k,m^k+1)であり、特にg|m^k+1から主張を得る。
----
一般化すれば
----
nを3以上の自然数、kを自然数、mをnと互いに素な自然数、fをn次の円分だ公式でp=f(m^k)が素数とする。
さらにφ(n)/2は奇数とする。
このときf(0)=1によりp-1はm^kの倍数で

m^((p-1)/m^k)≡1 (mod p)。
----
になった。
やっぱりkがnにない素因子tをもてばf(m)がf(m^(k/t))の倍数になってしまうので(nk,p-1)=(nk,(p-1)/m^k)になってしまう。
実例があるかは不明だけど。

475 :132人目の素数さん:2019/10/28(月) 10:15:43.56 ID:qwWsB6li.net
あ、φ(n)/2が奇数ならnはq≡3(mod 4)の素数のn=qか2qのどっちかしかないのか。
あんまおもんないな。

476 :132人目の素数さん:2019/10/30(水) 08:23:31.75 ID:BmR+wraF.net
 m^a ≡ m^b ≡ 1 (mod n)
のとき
 m^g ≡ 1 (mod n)
ただし g = gcd(a,b)、gcd(m,n)=1

(略証) ユークリッドの互除法により
  g = k*a - L*b
となる整数 (k,L) がある。(終)

本題では
 3k | (m^k +1) | (p-1)
∴ g = 3k,
∴ フェルマーは不要・・・・

k = (3^e)t,  (t,3) = 1

477 :132人目の素数さん:2019/11/07(木) 22:12:43.72 ID:3snBEz3A.net
>>464 >>473

各 x(n,t) を 初期状態 { x(n,0) | nは整数} の汎関数と考えよう。

min[a,b] は a,bについて単調非減少ゆえ、
ある x(c,0) を増やしたとき x(n,t) のどれも減少しない。
ある x(d,0) を減らしたとき x(n,t) のどれも増加しない。

で, どうなる?

478 :132人目の素数さん:2019/11/08(金) 23:59:43.92 ID:TJwXx40r.net
締め切りでーす

479 :132人目の素数さん:2019/11/09(土) 16:32:56 ID:czrA/bm0.net
>>477
「先祖数」y を次のように定める。
 y(n,0) = 1
 x(n,t+1) = x(n,t) のときは y(n,t+1) = y(n,t)
 x(n,t+1) = x(n+1,t) + x(n+2,t) のときは y(n,t+1) = y(n+1,t) + y(n+2,t)
このとき
 x(n,t) ≧ m・y(n,t)
一方、
 x(n,t) ≦ M
よって
 y(n,t) ≦ M/m,
各nについては有限回だが、nは無限の遠方まで伸びている・・・・

480 :132人目の素数さん:2019/11/10(日) 01:23:07.75 ID:v9KtaDvq.net
>>476 は11月号 出題1のヒントかとオモタ。

481 :132人目の素数さん:2019/11/10(日) 13:36:14.68 ID:v9KtaDvq.net
2019年11月号

■出題1  上の句がm字,下の句がn字の短歌がある。
 それぞれが回文であり,また全体の m+n字 も回文である。
 すべてが同じ字になってしまうのは (m,n) の組がどんな場合か?

(解答例)
m=n のとき  上の句と下の句は(逆向きに)同じである。
 それぞれが回文だから、[(m+1)/2] 種まで可能である。
m>n のとき  下の句は、上の句の初めのn字と(逆向きに)同じだから捨てよう。
 初めのn字を上の句、残り m-n字を下の句とした短々歌を考える。
 上の句、全体は回文である。下の句も元の中央部分だから回文である。
 ∴ (m,n) は (n,m-n) に帰着する。

m<n のとき
 上の句は、下の句の末尾のm字と(逆向きに)同じだから捨てよう。
 はじめの(n-m)字を上の句、末尾のm字を下の句とした短々歌を考える。
 下の句、全体は回文である。上の句も元の中央部分だから回文である。
 ∴ (m,n) は (n-m,m) に帰着する。

m≠n である限り全長が短くなるから、有限回で m=n に到達する。
上に述べた手順はユークリッドの互除法と同じだから、最後に g=gcd(m,n) に到達する。
したがって、[(g+1)/2] 種まで可能である。

m,nの最大公約数 g=gcd(m,n) が 1か2 の場合。・・・・・ (答)

■出題2  指ハブを入手できませんでした。。。

482 :132人目の素数さん:2019/11/10(日) 14:09:14 ID:qI1OjZfY.net
句全体を0〜n-1のZ/nZの代数で考える。
fを上の句と下の句をそれぞれ両方反転させる変換、gを全体を反転させる変換とする。
この時
f(x)=-x+m-1、g(x)=-x+m+n-1
だから
gf(x)=x+n。
よって(m,n)=1なら明らかに非自明解なし。
(m,n)=2なら全ての偶数文字目と奇数文字目は同一である必要があり、さらにg不変性も考えれば自明解しかない。

483 :132人目の素数さん:2019/11/11(月) 19:21:27.56 ID:HO1Gv+Br.net
>>481
エレガントで賞

484 :132人目の素数さん:2019/11/14(木) 02:02:16.84 ID:QBs+///B.net
今月の出題1は、結構有名な命題だよね。

そして、出題2は、素数大富豪考案者が再降臨された模様。

485 :132人目の素数さん:2019/11/14(木) 03:10:13.57 ID:CYKGROTS.net
2019年9月号 

■出題1
 29個が正解。
 解答者52人中
  初等幾何による正解者 9人  (例 >>417 >>420)
  代数幾何による正解者 6人  (例 >>414 >>415)
  結果に到達した人   9人
  その他        28人

 (2019年12月号 解説)

486 :132人目の素数さん:2019/11/23(土) 15:59:58 ID:lHHxttJQ.net
昔エレガントな解答求むで雨宮先生が出した問題を2chで誰かがかっこよく解いて話題になった話しがあるらしいんですが何の話かわかる人いますか?
格子点に正の数を配置する問題で隣接する点でなんか不等式満たすやつ求めるやつらしいんですけど。

487 :132人目の素数さん:2019/11/23(土) 21:26:35.51 ID:FRVpxoPK.net
>>484
>今月の出題1は、結構有名な命題だよね。

そうなのか

488 :132人目の素数さん:2019/12/07(土) 22:55:35.14 ID:2dG5P0ys.net
明日締め切り!

489 :484:2019/12/09(月) 01:22:21.91 ID:MK7AdcCW.net
締め切り過ぎたので、出題1の回答をw

次の命題を示す。
2m-1個の整数の集合には、和がmで割り切れるようなm個の整数が必ず存在する。
x_1,x_2,・・・・,x_(2m-1)を0≦x_1≦x_2≦・・・≦x_(2*m-1)なる、整数とする。

ここでは、剰余を問題とするので、0≦x_1≦x_2≦・・・≦x_(2*m-1)<mと考えて良い。
まずmが素数の場合を考える。

整数列y_i(1≦i≦m-1)を、y_i=x_(i+m)-x_iと定義する。

ある値iに対して、y_i=0となるとき
明らか

任意のiに対して、y_i≠0となるとき

ここで、整数の集合S_0,S_t(ただし、tは1以上m以下の自然数),R_tをこう定義する。
S_0={0}
S_t={f_1*y_1+f_2*y_2+・・・+f_t*y_t|f_1,・・・・,f_tは0あるいは1}
R_t={s+y_t| sはS_(t-1)の元}

明らかに、S_(t+1)=(S_t)∪{R_(t+1)}である。

ここで、S_tのmod mでの、S_tの相異なる元の個数をk個とすると、
k<mのとき、mod mでの、S_(t+1)の相異なる元の個数はk+1個以上となることを示す。・・・☆

S_t={s_1,・・・・,s_k}とおくと、R_(t+1)={s_1+y_(t+1),・・・・,s_k+y_(t+1)}

R_(t+1)の任意の元の和を取ると、s_1+・・・+s_k+k*y_(t+1)
S_tの任意の元の和を取ると、s_1+・・・+s_k

mは素数だから、k*y_(t+1)はmでは割り切れない。よって、R_(t+1)には、mod mでS_tのどの元とも等しくない元が存在する。

よって、☆は示された。

☆とS_0={0}より、k<mのとき、S_kの元の個数はk+1以であることがわかる。

よって、S_(m-1)の元の個数はm以上となるので、
S_(m-1)は、mod mでの任意の元を含むから、とくに
x_1+・・・+x_m+f_1*y_1+・・・+f_(m-1)*y_(m-1)がmで割り切れるように、f_1,・・・,f_(m-1)の値を選ぶことが出来る。

x_i+f_i*y_i=x_i あるいは x_(i+m)だから、x_1+・・・+x_m+f_1*y_1+・・・+f_(m-1)*y_(m-1)が求める、m個の数字の和となる。

490 :132人目の素数さん:2019/12/09(月) 01:34:10.08 ID:MK7AdcCW.net
数学的帰納法で、以下の命題を示す。
「2m-1個の整数の集合には、和がmで割り切れるようなm個の整数が必ず存在する。」…★

m=1のとき、明らか。

m≦h-1(hは自然数)のとき、命題★は正しいと仮定する。

m=hのとき

hが素数のとき、>>489より命題★は正しい。

hが合成数のとき、

h=a*bとかける

2*a*b-1>2*a-1
帰納法の仮定より、和がaで割り切れる、a個の整数の集合T_jが2*b-1組取ることが出来る。

T_1の元の和,T_2の元の和,・・・,T_(2*b-1)の元の和をそれぞれ、a*t_1,a*t_2,・・・,a*t_(2*b-1)とすると
帰納法の仮定より、t_1,・・・,t_(2*b-1)のなかで、bで割り切れるb個の整数が存在する。

これを、t_(i_1),・・・・,t_(i_b)とする。

a*{t_(i_1)+・・・・+t_(i_b)}はa*bで割り切れる。

そもそも、a*t_(i_1),・・・,a*t_(i_b)達は、a個の整数の和であるから
a*{w_(i_1)+・・・・+w_(i_b)}は、x_1,・・・,x_(2*h-1)から取ったh個の整数の和である。

よって、hが合成数のときもこの命題★は正しい

m=hのときも、この命題★は正しいことが示された。

よって、帰納法により任意の自然数mに対して、この命題★は正しい。

491 :132人目の素数さん:2019/12/09(月) 01:44:12.67 ID:MK7AdcCW.net
>>489-490で示した命題「2m-1個の整数の集合には、和がmで割り切れるようなm個の整数が必ず存在する。」
を考えれば、出題1の回答は明らかに「不可能」だということができます。

上記の命題は、あのポール・エルデシュが証明した命題の一つです。
https://www.renyi.hu/~p_erdos/1961-25.pdf

492 :132人目の素数さん:2019/12/09(月) 02:22:27.86 ID:LcNpRSGz.net
整数論スレでフライングぎみのがあったな

https://rio2016.5ch.net/test/read.cgi/math/1572639526/92

493 :132人目の素数さん:2019/12/09(月) 07:24:34 ID:I16zcHtM.net
はへー。もっと簡単に解けるのかと思ってたわ。

494 :132人目の素数さん:2019/12/09(月) 07:52:20 ID:VkmKjK9n.net
いや、一般化してるから長いだけで17で9の倍数ならこなフライングのやつでいけるでしょ?
5個の中から3の倍数の三つ組がとれるなら3個づつとっていけば17個なら5組取れる。
3の倍数が5個あれば3組選んで9の倍数になる。

495 :132人目の素数さん:2019/12/09(月) 18:52:44.52 ID:LcNpRSGz.net
でもどうなんだろ?
もちろん簡明さでは17個の中から9の倍数の組み見つけるケースに絞ってしまえば楽だしスッキリは行くけどエルデシュの証明には抗しがたい美しさがあるな。

496 :132人目の素数さん:2019/12/09(月) 23:15:28.28 ID:UpuJMMdk.net
出題2はめんどくさかった
エレガントな回答カモン

497 :132人目の素数さん:2019/12/10(火) 07:51:38.48 ID:9+9M8wAb.net
■出題2

補題1
 4点以下は、すべて÷配置である。

補題2
 5点以上では無限配置も現れる。たとえば
・半直線AX上に2点以上、AY上に2点以上ある。(AX、AYは点Aで交叉する)
・凸4角形の4頂点と、その対角線(の延長)上にない1点。
・凸m角形 (m≧5)


さて、どうするか。

498 :132人目の素数さん:2019/12/14(土) 11:42:12.06 ID:Zsp8poRO.net
○ Pが凸4角形を含む場合(上記を除く)

 ・凸4角形の両対角線上にPの点X,Y (≠ 4角形の頂点)がある場合
  T(P)は対角線の交点Aを含む。
  半直線AX上に2点以上、半直線AY上に2点以上、Pの点(≠A)がある。
  → Pは無限配置。

 ・Pの点(≠ 4角形の頂点)は、凸4角形の一方の対角線上に限る場合。
  → Pは÷配置。

○ Pが凸m角形 (m≧4) を含まない場合
  Pの凸包は△で、その内部または辺上にPの点が2個以上ある。
  ・或る2点を結んだ直線Lが△の頂点を通らないとき
    Lに関して同じ側に△の2頂点がある。
    → Pは凸4角形を含む。
  ・どの2点を結んだ直線も△の頂点を通るとき
    ある頂点Aを2本の直線AX、AYが通る → Pは無限配置。
    1頂点を1直線が通るのみ → Pは÷配置。
    2頂点を2直線が通る → 下に限る。
    3頂点を3直線が通る → Pは手裏剣配置。

以上により、Pは÷配置、手裏剣配置、無限配置のいずれか。
∴ Pが有限配置ならば、Pは÷配置または手裏剣配置。(終)

たしかに面倒だな・・・・

499 :132人目の素数さん:2019/12/14(土) 11:51:07.66 ID:Zsp8poRO.net
逆の
 「Pが÷配置または手裏剣配置ならば、Pは有限配置」
は T(P) を考えればよい。

500 :132人目の素数さん:2019/12/14(土) 11:57:16.98 ID:Zsp8poRO.net
>>453-461

10月号出題2 (1)は「N≠0 に限る」と修正するようです。
(1月号解説)

501 :132人目の素数さん:2019/12/14(土) 14:57:52.65 ID:R6EPz61W.net
まずは5点のケースで有限配置と無限配置を分類
PがN+1点で有限配置のとき、任意のN点が取るべき配置を考える
こう書くとエレガントな解答に見えるが実際は細かい議論が必要でエレファント

502 :132人目の素数さん:2019/12/15(日) 00:45:56.80 ID:Nfo6ujPm.net
>>498

○ Pが△を含まない場合
Pのすべて点は一直線上に並ぶ。
 Pは÷配置。

503 :132人目の素数さん:2019/12/15(日) 00:55:02.75 ID:1xZAPqJd.net
証明自体は簡単なんだけどな。
ただ3レスくらいかかる証明しか持ってないので書く気しないよ。

504 :132人目の素数さん:2019/12/15(日) 15:36:43 ID:Nfo6ujPm.net
>>497
補題1
(略証)
 凸包が凸4角形のとき、対角線の1つをLとする。
 凸包が△のときは、頂点でない点と1頂点を含む直線をLとする。
 凸包が直線のときは、それをLとする。

補題2
(略証)
・Pが点Aと、半直線AX上の2点B1、C1と、半直線AY上の2点B2,C2を含むとき
 T(P)は B1C2 と B2C1 の交点Dを含む。
 TT(P)は B1B2 と AD の交点Eを含む。
 半直線AED上には2点E,Dがある。
 同様にしてAをとおる無数の半直線が生じる。
∴ Pは無限配置。

・Pが凸4角形の4頂点と、その対角線上にない点A'を含むとき
 T(P)は4角形の対角線の交点Dを含む。
 A'が4角形の外部にあるとき、TT(P) はA'Dと1辺の交点Aを含む。
 A’が4角形の内部または辺上にあるときは A=A' とする。
 両対角線により点Aと隔てられた頂点をB1、B2とする。
 TTT(P) はAB1と対角線の交点C1、AB2と対角線の交点C2を含む。
 半直線AC1B1 と 半直線AC2B2 が点Aで交叉する。
∴ Pは無限配置。

・Pが凸m角形 (m≧5) を含むとき
 一頂点Aから対角線 AB1、AB2を曳く。
 Aの隣の2頂点をむすぶ対角線 L~ を曳く。
 T(P)はそれらの交点C1,C2を含む。
 半直線AC1B1 と 半直線AC2B2 が頂点Aで交叉する。
 ∴ Pは無限配置。

505 :132人目の素数さん:2019/12/17(火) 19:14:20.23 ID:0PR0vNCM.net
はよ解答書けや

506 :132人目の素数さん:2019/12/17(火) 23:54:28.78 ID:/04vhOiY.net
大筋は

「Pが有限配置ならば、Pは÷配置または手裏剣配置」は
>497 (略証 >504) + >498 + >502

逆の
「Pが÷配置または手裏剣配置ならば、Pは有限配置」 は
>499

としよう。あとは任せた。

507 :132人目の素数さん:2019/12/19(木) 18:20:57.91 ID:WgMaiYlt.net
カッパー氏って、今何やってるのかな?

508 :132人目の素数さん:2019/12/20(金) 02:07:23.51 ID:yiLw1Jz8.net
0730
しろ@huwa_cororon 11月27日
苦節6ヶ月、初満点&一等賞です!
https://twitter.com/huwa_cororon/status/1199593474128896000
(deleted an unsolicited ad)

509 :132人目の素数さん:2019/12/21(土) 10:57:42.50 ID:78CNH7Nl.net
今月は手強そうやな

510 :132人目の素数さん:2020/01/04(土) 10:05:49.58 ID:SH03Jqfm.net
出題1だけど、一意性の証明は不要?

511 :132人目の素数さん:2020/01/04(土) 12:02:14.80 ID:BYJjlqaZ.net
要るでしょ。一例を示すだけなら簡単すぎるので。

512 :132人目の素数さん:2020/01/04(土) 12:13:45.33 ID:SH03Jqfm.net
そうなのか
となると例えばだけど、2回微分不可で題意を満たす関数の存在/非存在を議論しなければならないよねえ?

513 :132人目の素数さん:2020/01/04(土) 14:00:11.94 ID:Dm84v2tx.net
エレガントな解答を考えてください

514 :132人目の素数さん:2020/01/04(土) 17:17:56.44 ID:4MdsIuzY.net
>>510
出題1だけど、一意性は示せないと思う。
折れ、題意をみたす関数を少なくとも2つ見つけちゃったんだw

515 :132人目の素数さん:2020/01/04(土) 17:32:18.17 ID:hhufihOv.net
>>514
その2つの線形和を1つの関数と考えたときの一意性はどうよ

516 :132人目の素数さん:2020/01/04(土) 21:05:40.18 ID:nbZtfxS2.net
>>515
それ、>>514書いた後に気が付いたw

517 :132人目の素数さん:2020/01/05(日) 07:24:55.67 ID:fVS8vAuP.net
>>512
f '(・・・) をもう一度微分するでござるか?
それが可能かどうか?
しかし (・・・) が定数ならば明らかに0じゃ。

518 :132人目の素数さん:2020/01/05(日) 10:00:53.71 ID:bHdBPZOn.net
>>517
要するにC∞ではない関数が存在するのかどうか

519 :132人目の素数さん:2020/01/09(木) 22:13:00.86 ID:ZnFau0TA.net
締切ぞ過ぎにけり

520 :132人目の素数さん:2020/01/10(金) 00:05:42.17 ID:5f1mkQv1.net
今月の出題2は岡本先生
いかにも面白そう

出題1は一松先生
御年93という生ける伝説
同時代に生まれて解答を読んで頂ける幸せ

521 :132人目の素数さん:2020/01/10(金) 03:03:17.76 ID:TDh/4MHA.net
>>517
 f '(√(xy)) が定数ってことは xy=a (定数) ってことか。
つまり x,a を独立変数と考える。
 f(x) - f(a/x) = f '(√a)(x - a/x),
そこで a>0, a≠1 に対して
 h(x) = {[f(x) - f(a/x)]/(x-a/x) - f '(a)}/(1-a),
とおく。 (なぜか 1-a で割ってある.)
これをxで微分すると
 h '(x) = {[f '(x) + f '(a/x)(a/xx)]/(x-a/x) + [f(a/x)-f(x)](1+a/xx)/(x-a/x)^2}/(1-a)
   = 0  (←題意)
ここで変数を交代して x=1, a=u とおこう。
 h '(1) = {f '(1) + f '(u)・u}/(1-u)^2 + [f(u) - f(1)](1+u)/(1-u)^3
  = f '(1)/(1-u)^2 + f '(u)・u/(1-u)^2 + [f(u)-f(1)](d/du){u/(1-u)^2}
  = (d/du){f '(1)/(1-u) + [f(u)-f(1)]・u/(1-u)^2}
  = 0,
uで積分して
 f '(1)/(1-u) + [f(u)-f(1)]・u/(1-u)^2 = C,
 f(u) = f(1) - f '(1)(1-u)/u + C(1-u)^2 /u
  = A/u + B + Cu,    (u=1 も含めて)

522 :132人目の素数さん:2020/01/10(金) 23:57:21.52 ID:5f1mkQv1.net
手のひらで転がすような解答だな

523 :132人目の素数さん:2020/01/11(土) 01:58:06.13 ID:Peg94VlV.net
>>521
なんかの本か論文で読んだことあるわ
右辺で、幾何平均の代わりに算術平均を用いると確か二次関数が出てきて、
調和平均とか二乗平均平方根とかでも類似の結果が得られるんだよな
平均の形に限らない一般形の場合はオープンクエスチョンになってた気がする

524 :132人目の素数さん:2020/01/11(土) 03:46:19.36 ID:RkMH+jmj.net
>>512
 2回微分不可で題意を満たす関数

>>518
 C^∞ 級ではない関数

は無いんかいな?

525 :132人目の素数さん:2020/01/11(土) 04:41:10.23 ID:IwcKojG6.net
任意のaについて
f'(x)=(f(ax^2)-f(1/a))/(ax^2-1/a)
が言えてるんだからC^∞じゃないの?
任意のx>0についてその近傍Uと適当なaで右辺の分母がUで0にならないようなものを持って来れる。

526 :132人目の素数さん:2020/01/11(土) 08:15:08.77 ID:lgKUVeiw.net
>>523
おもしれ
他の平均だとどうなるのか知りたい

527 :132人目の素数さん:2020/01/11(土) 08:18:58.15 ID:lgKUVeiw.net
>>525
C∞なら級数を使う方法が簡単かな

528 :132人目の素数さん:2020/01/13(月) 03:27:51.34 ID:JM7pXrDp.net
2020年1月号 講評?

■出題1
 微分を含んだ関数方程式。
 C^∞ 級の f(x) = A/x + B + Cx だけらしい・・・・ >>521

■出題2
 H(i,j) ≧ 2ij +i +j +1
 までは行くんだが・・・・、等号かな?

529 :132人目の素数さん:2020/01/13(月) 09:11:09.64 ID:7u70ISAt.net
>>528
≦8は必ず現れる特定配置に注目すれば何とかカントカ
≦13の方が簡単な気が

530 :132人目の素数さん:2020/01/13(月) 20:09:50.55 ID:JM7pXrDp.net
>>485
9月号出題1 (解説:12月号)
「真っ二つに」が「等体積に」の意味だとすれば、最低30個が無傷で残るらしい。
30個 と解答した10人を含めて 34人が到達でござる。

(2020年2月号 訂正)

531 :132人目の素数さん:2020/01/14(火) 19:25:16 ID:Nvi2zejl.net
>>483
 拙者もそう思うでござるよ。
 2月号の解説では「ひじょうに効率が悪い」と言われたが
 割り算なしで同じ結果に到達できるなら・・・・
                      ぬるぽ

532 :132人目の素数さん:2020/01/18(土) 11:18:11.92 ID:F/Us7hOY.net
1月号出題2を解ききった猛者はいないのか

533 :132人目の素数さん:2020/01/19(日) 10:03:21 ID:K/so592d.net
↓の結論は H(i,j) = 2ij+i+j+1 らしい。

プログラム技術板 - お題スレ Part16
http://mevius.5ch.net/test/read.cgi/tech/1573948822/326-457

534 :132人目の素数さん:2020/01/19(日) 13:41:13 ID:IG3CW3R1.net
>>533
証明みつからなかった

535 :132人目の素数さん:2020/01/21(火) 22:46:22 ID:r+ocZZAM.net
出題1はもうすこし単純な計算でもいけた

雑に書くと
・条件を満たす関数の集合Vは線形空間
∵ 条件式の両辺はfに関して線形
・f∈V, t>0 ⇒f(tx), f(t/x) ∈V.
∵ 合成関数の微分するだけ
・V={f∈V|f(x)=f(1/x)} ⊕ {f∈V|f(x)=-f(1/x)}
∵ 共通部分が0の部分空間
 f(x)= (f(x)+f(1/x))/2 + (f(x)-f(1/x))/2.
・f∈V, f(x)=-f(1/x) ⇒ f(x)=A(x-1/x)
∵ 2f(x)=f(x)-f(1/x)=f'(1)(x-1/x)
・f∈V, f(x)=f(1/x) ⇒ f(x)=A(x+1/x)+B
∵ 両辺微分してxを掛けると, xf′(x)=-f′(1/x)1/x.
 f(t x)−f(1/t x)= f′(x)(t x-1/t x)よりxf′(x) ∈V.
 xf′(x) =A(x-1/x)の両辺をxで割って積分.
・f∈V ⇒ f(x)=Ax+B+C/x
∵ 係数を整理

536 :132人目の素数さん:2020/01/22(水) 02:09:15 ID:xZx9jgfS.net
【単純】
1 そのものだけで、混じりけがない。他の要素などが混入していない。
2 機能・構造・形式などが混み入っていない。
3 {e}とそれ自身以外には正規部分群、商群をもたない。

537 :132人目の素数さん:2020/01/24(金) 17:57:48 ID:rC18lOHu.net
そうか、出題1はC^∞まででたら後はそこまで難しくはないのか。
任意のa,b,xに対して
f(ax)-f(1/x)=f'(√a)(ax-1/x)
f(bx)-f(1/x)=f'(√b)(bx-1/x)
なので辺々ひいてxかけて三回微分して
(xf(ax)-xf(bx))'''=0。
∴ a^3x f'''(ax)+3a^2x f''(ax)
- (b^3x f'''(bx)+3b^2x f''(bx))=0。
x=1を代入して
a^3f'''(a)+3a^2f''(a):const
∴ (x^3 f''(x))':const。
∴ f(x)=Ax+B+C/x+Dlogx。
後はD=0を示せば良い。
今更ながらですが。

538 :132人目の素数さん:2020/01/24(金) 22:17:18 ID:QO+s+Rac.net
log??

539 :132人目の素数さん:2020/01/24(金) 22:17:36 ID:QO+s+Rac.net
あごめん無視してw

540 :132人目の素数さん:2020/01/26(日) 05:48:10.62 ID:pENss7Z1.net
f(x) は2回微分可能とします。
 ・・・・
辺々引いて
 f(ax) - f(bx) = (定数1)x - (定数2)/x,
xを掛けて微分する。
 {ax・f'(ax)-bx・f'(bx)} + {f(ax)-f(bx)} = 2(定数1)x
xで割って微分する。
 {aaf"(ax)-bbf"(bx)} + {af'(ax)-bf'(bx)}/x -{f(ax)-f(bx)}/xx = 0,
xxを掛ける。
 (ax)^2・f"(ax) +ax・f'(ax) -f(ax) = (bx)^2・f"(bx) +bx・f'(bx) -f(bx),
よって
 u^2・f"(u) + u・f'(u) - f(u) = -B (定数)
これから
 f(u) = A/u + B + Cu,
定数1,定数2を消すために2回微分したので、積分定数が2個出る。
他に定数Bもある。

541 :132人目の素数さん:2020/01/26(日) 19:01:52 ID:HSj8A8kk.net
>>537
よくよく考えたらa=0代入したらconst.は0確定だからD=0に決まってますな。

542 :132人目の素数さん:2020/02/05(水) 21:44:25 ID:IQtO/k+g.net
2^nの壁

543 :132人目の素数さん:2020/02/06(木) 22:17:38 ID:TEIP+ffP.net
「余力がある場合は」
これがなければ最後の1週間幸せに過ごせたのに

544 :132人目の素数さん:2020/02/06(木) 23:32:26 ID:URmc07Wk.net
一番最初に思いついた方法が2^nだった。
ついてたね。

545 :132人目の素数さん:2020/02/07(金) 00:11:34 ID:MoVOlU2O.net
>>544
やるなおぬし

そのコメントがさりげないヒントになった
うっしっし

546 :132人目の素数さん:2020/02/08(土) 23:17:46.36 ID:RHBQyrQ6.net
3月号が届いた
出題2の問題文長過ぎる

547 :132人目の素数さん:2020/02/09(日) 12:22:21 ID:fEhmVG0S.net
最初に思いついたのは2^{n-1}だった
nが大きくなればなるほどより短く評価できると思う
長さの最小値は分からない

548 :132人目の素数さん:2020/02/09(日) 16:50:30 ID:NRGMNCPO.net
そんなに簡単な問題だったのこれ?
しょぼん

549 :132人目の素数さん:2020/02/09(日) 21:05:25 ID:BHX2wTJj.net
あれ?これもう締め切りすぎたんじゃないの?
解答カモーン

550 :132人目の素数さん:2020/02/09(日) 21:46:48.26 ID:JijE+Tx4.net
2020年2月号 講評?

■出題1

垂足チェヴァ点Pの軌跡Γの問題。
チェヴァの定理より
 Pが垂足チェヴァ点 ⇔ f(P)=1
ここに 
 f(P) = (BD/DC)(CE/EA)(AF/FB),
ただし3点 D,E,F は問題文のように定める。

〔補題〕
 P(≠O) が垂足チェヴァ点ならば、外心Oに関してPと対称な点P~も垂足チェヴァ点である。

これが出ればあとはやさしい。

・問1
問題文から、△ABCの垂心Hは垂足チェヴァ点である。(証明略)
外心Oに関して垂心Hと対称な点はオイラー線上にあるが、これは補題により垂足チェヴァ点である。
(ド・ロンシャン点と云うらしい)

・問2
 各点の座標を A(0,2mc) B(-3c,-mc) C(3c,-mc) P(X,Y) などと置き、f(P)=1 をひたすら計算する・・・・

(補題の略証)
 まず △ABCの外心をOとすると、OA=OB=OC
 ΔOBC, ΔOCA, ΔOAB は2等辺三角形。
 外心Oから辺BCに引いた垂線の足Lは、辺BCの中点。
 次に P,O,P~ から直線BCに引いた垂線の足をD,L,D~ とする。
 中点連結定理から、DD~ の中点も L
 BD=D~C, BD~=DC
 (BD/DC)(BD~/D~C) = 1,
他の二辺についても同様だから掛け合わせて
 f(P)f(P~) = 1,
 f(P)=1 ⇔ f(P~)=1   (終)

551 :132人目の素数さん:2020/02/09(日) 23:16:02 ID:0rilIx6Y.net
|A|=3のとき
s(a_1)=123, s(a_2)=231, s(a_3)=312
と定めればよい.
(s(b)の定め方は行数の都合上ここでは省く)

|A|=4のとき
s(a_1)=414243, s(a_2)=424341, s(a_3)=434142, s(a_4)=4
と定めればよい.

以下|A|=nのときは一つ前の文字列の先頭と,文字と文字の間にnを付け加えてs(a_i)を帰納的に定める.
s(b_i)もn=3のときから始めて,この文字と文字の間に挟む定め方に倣う. ただし一工夫必要.
各文字列の長さはn>=3において3×2^{n-3}以下

方針はこんな感じ

552 :132人目の素数さん:2020/02/10(月) 06:59:34 ID:esjbRF9d.net
>>550
チェバの定理なんやけど.015-016

553 :132人目の素数さん:2020/02/10(月) 10:13:45 ID:1+8rzOtr.net
>>551
オレもサイズが1あげる時間に新しい文字挟んでいく論法でやったんだけど、それだとn×nではうまくいかなかった。
で話をm×nに一般化してm×1からスタートしていった。
この方法だとm×nからm×(n+1)に上がるとき
・m個の文字列の方は間に新しい文字Aをはさみ、新しい(n+1)個目を受け付けるほうは末尾にもAを追加する。
・n個の文字列の方は間に文字Aを挟む。
・(n+1)個目の文字列はA一文字とする。
コレでいけるんだけど>>551さんはいわばこのmの方も1あげるんですよね?
それはどうするんですか?

554 :132人目の素数さん:2020/02/10(月) 10:30:43 ID:1+8rzOtr.net
訂正
>>551
×・n個の文字列の方は間に文字Aを挟む。
◯・n個の文字列の方は間に文字Aを挟み、末尾にもAを追加。

m個の文字列の方もn個の文字列の方も長さ2^(n-1)になります。
ちなみにm,nに大きさの大小の指定はないので1x1から双方1ずつ上げていってもできますが、その場合2^(m+n-2)になるのでm×1からスタートするものより格段に長い文字列が必要になってしまう。

555 :132人目の素数さん:2020/02/10(月) 10:56:45 ID:1+8rzOtr.net
あ、問題文の文字設定覚え間違ってた。
問題文はX⊂A×BでAの要素数がnですね。
適当に読み替えてください。

556 :551:2020/02/10(月) 15:17:16 ID:y3pjTCCQ.net
>>553
おそらく私もあなたも本質的には同じ手法と思われます.
私はs(b_i)を「3つの場合に分けて」定めてn×nの場合を証明しましたがn×mでも当然証明可能ですし,
私の「場合分け」の部分が、あなたの「m×nにしてm×1からスタート」あるいは「末尾にも〜」にあたるのだと思います.

557 :132人目の素数さん:2020/02/10(月) 15:52:58.61 ID:1+8rzOtr.net
出題1はどのくらいまでなら実の代数幾何使っていいんだろうか?
三次曲線Γは各辺の垂直二等分線を漸近線にしていて、一次の因子を持つならそれはこの三本のいずれかに限り、その時三角形はその線を対称軸とする二等辺三角形である事も容易に示せてしまう。
すると例えば問2は例えば

二等辺三角形のときその対称軸LはΓの一次の因子でΓは
Γ=L∪Q
と因子分解される。
Qは2次の実代数的集合である。
コレは二本の漸近線を持つから双曲線であるか、二本の直線に因子分解されるが、もしLと異なる因子をΓが持てば元の三角形は二本の対称軸を持つことから正三角形となり仮定に反する。
故にQは双曲線である。

とかは認めてもらえないのかな?

558 :132人目の素数さん:2020/02/11(火) 03:11:56 ID:RFPoCgqI.net
フィールズ賞受賞者が3人揃って代数幾何学の大家であるこの国のこと(12月号解説)だから認められて良いと思うけど・・・・
御老公の解説(5月号)をもう暫く待とう哉。

559 :132人目の素数さん:2020/02/11(火) 05:05:00.78 ID:RFPoCgqI.net
>>519
 締切 過ぎにけり。
 締切ぞ 過ぎにける。
のどっちかだろ。それとも
 締切こそ 過ぎにけれ。
かな? (係り結びの法則)

560 :132人目の素数さん:2020/02/11(火) 21:40:04 ID:yio90v+X.net
鎌倉以前と室町以降

561 :132人目の素数さん:2020/02/12(水) 04:56:11.31 ID:uWBQqkSN.net
室町以降は終止形と連体形の区別が無くなった。。。

562 :132人目の素数さん:2020/02/15(土) 09:37:55 ID:jgju9Hu6.net
もう一度言う

問題文長過ぐる

563 :132人目の素数さん:2020/02/15(土) 20:04:13 ID:h/D6xsZJ.net
初めていう

丸パクリ感

564 :132人目の素数さん:2020/02/16(日) 00:00:57 ID:TJjLtE4v.net
敢えて言う

出題1おもろない

565 :132人目の素数さん:2020/02/16(日) 15:14:24 ID:TJjLtE4v.net
N=7なら最初から 7 と言えよと思わぬでもなし

566 :132人目の素数さん:2020/02/16(日) 18:02:03 ID:N9QZtxQk.net
>>564
〔出題1:追加問題〕
 k = α+β+γ とします。次も求めてください。
 α/β + β/γ + γ/α,
 β/α + γ/β + α/γ,
 (α/β)^3 + (β/γ)^3 + (γ/α)^3,
 (β/α)^3 + (γ/β)^3 + (α/γ)^3,

567 :132人目の素数さん:2020/02/16(日) 19:06:38 ID:TJjLtE4v.net
>>566
もう対称式は見たくない

568 :132人目の素数さん:2020/02/17(月) 13:37:51 ID:e8jOHOIZ.net
>>566 は本題とは関係ない脇道なので、よゐ子は気にしないでね。
 β/γ = - {2/7^(1/6)} sin(8π/7),
 γ/α = - {2/7^(1/6)} sin(2π/7),
 α/β = - {2/7^(1/6)} sin(4π/7),
から
 α/β + β/γ + γ/α = -7^(1/3),
 β/α + γ/β + α/γ = 0,
 (α/β)^2 + (β/γ)^2 + (γ/α)^2 = 7^(2/3),
 (β/α)^2 + (γ/β)^2 + (α/γ)^2 = 2・7^(1/3),
 (α/β)^3 + (β/γ)^3 + (γ/α)^3 = -4,
 (β/α)^3 + (γ/β)^3 + (α/γ)^3 = 3,
 ・・・・・

569 :132人目の素数さん:2020/02/18(火) 08:22:51 ID:4+VDmEHG.net
>>568
θ=2nπ/7 (nは7と素) ゆえ
sinθ, sin(2θ), sin(4θ) は
 s^3 - {(√7)/2}s^2 + (√7)/8 = 0,
の根である。
 sinθ・sin(2θ)・sin(4θ) = -(√7)/8,
 sin(8θ) = sinθ,
よって
cosθ/cos(2θ) = sin(2θ)^2 /{sinθ・sin(4θ)}
 = sin(2θ)^3 /{sinθ・sin(2θ)・sin(4θ)}
   = -(8/√7) sin(2θ)^3,
から α/β, β/γ, γ/α が出る。
それを使えば
 α/β + β/γ + γ/α = - {2/7^(1/6)} {sinθ + sin(2θ) + sin(4θ)}
   = -7^(1/3) = -1.91293118
 β/α+γ/β+α/γ = (4/7^(1/3)){sinθ・sin(2θ)+sin(2θ)sin(4θ)+sin(4θ)sinθ}
   = 0,

570 :132人目の素数さん:2020/02/18(火) 12:34:40 ID:743GGd0Z.net
>>569
おぬしさては出題2も解いたな?

571 :132人目の素数さん:2020/02/18(火) 13:35:22 ID:IjmEFha0.net
もうこの辺にしとこう

572 :132人目の素数さん:2020/02/24(月) 14:41:54 ID:rY3ttyPd.net
みんな、どうやって解いた?

573 :132人目の素数さん:2020/03/07(土) 23:52:42 ID:SDdWDHEg.net
いまだに出題2(2)の題意が分からない俺様が通りますよ

574 :132人目の素数さん:2020/03/08(日) 00:14:13 ID:aBXErkYA.net
出題2(1)もおれはよく分からん
解けるけどなんでこういう問題なのかよく分からん
なにか根本的なところで誤解があるのか

575 :132人目の素数さん:2020/03/08(日) 10:14:38 ID:wBJwcGf2.net
(2)は解けてない
nが大きければfreeになりやすい、ってのがよく分からず
何を誤解してるんだろう俺は

576 :132人目の素数さん:2020/03/08(日) 11:09:16.57 ID:wBJwcGf2.net
俺の考えは逆で、nが大きい方がfreeになりにくいんだが
たとえばn=1で{1,5,7}には(1,7,7;5)の組が存在するが、2点が同一なので重三freeとなる
しかしn=2では数字を組み替えることで、重三が構成できてしまう
どこが間違ってる?

577 :132人目の素数さん:2020/03/08(日) 11:43:15 ID:IvQBbPYs.net
それより出題2の文章は無用に長いくせに意味がハッキリしないな。

4点が異なっている限り

は4点x,y,z,wが

x≠y ∨ x≠z ∨x≠w ∨ y≠z ∨ y≠w ∨ z≠w

なのか

x≠y ∧ x≠z ∧ x≠w ∧ y≠z ∧ y≠w ∧ z≠w

なのかハッキリしない。
当然後者だとは思うんだけど"異なる"は本来引数2つのpredicateでそれを4つの引数に適用するんだったら、そこはハッキリさせないと。
"全て相異なる"って便利な言葉があるのにそれ使ってないなら本問は違う方の意味にとってるのかなと思ってしまう。

578 :132人目の素数さん:2020/03/08(日) 12:34:41 ID:wBJwcGf2.net
>>577
そうだねえ
しかし前者の意味に取ったところで(2)は依然ナンセンスだと思う

579 :132人目の素数さん:2020/03/08(日) 15:01:05 ID:wBJwcGf2.net
もちろんナンセンスってのは誤解してるだけなんだろうけど
あと数時間しかない
誰か誤解を解いてケロ

580 :132人目の素数さん:2020/03/08(日) 20:06:45 ID:wBJwcGf2.net
もう諦めたワイ

581 :132人目の素数さん:2020/03/09(月) 23:55:26 ID:1cVzEUXL.net
たれぞ出題2の2にスッキリ解答よろ

582 :132人目の素数さん:2020/03/10(火) 11:05:37.55 ID:Ct1vj+NA.net
出題1にスッキリ解答・・・・

 sinθ = cos(π/2 -θ) = - cos(π/2 +θ),
から
 sin(π/14) = - cos(4π/7),
-sin(3π/14) = - cos(2π/7),
 sin(5π/14) = - cos(8π/7),
として下記の動画を視聴すると
 k = {(3・7^(1/3) -5)/2}^(1/3) = 0.71751508   
 k^3 の最小多項式は (2t+5)^3 - 7・3^3,

http://studytube.info/videos/38864      24:12
http://www.youtube.com/watch?v=CRWK5-Dp0Yc 24:12
タマキ/環耀の数学

S. Ramanujan: "Question 524", J. Indian Math. Soc., p.190-191 (1914)
B. C. Berndt: "Ramanujan's notebooks: part IV", Springer-Verlag (1994)

583 :132人目の素数さん:2020/03/10(火) 18:59:26 ID:1kPMOxIk.net
やってることは分かるし高校の文系数学の知識で解けるってのは素晴らしいけど、流れや思考のプロセスが分からん
もっと素朴に(エレガントに)解ける

584 :132人目の素数さん:2020/03/10(火) 23:03:20 ID:YEA73B8n.net
スッキリじゃなくてもいいから出題2の2の解説よろ
問題の説明でもいいさね
さっぱり分からんのじゃ

585 :132人目の素数さん:2020/03/11(水) 12:36:15 ID:a2VXI31Q.net
PとQって等速直線運動なの?

586 :132人目の素数さん:2020/03/11(水) 13:13:07 ID:C0sIGim3.net
等速直線運動だと思うが
図がBD間の方が離れてるということは…

587 :132人目の素数さん:2020/03/11(水) 22:33:59 ID:2hJDzVzO.net
4月の出題者、最近出題したばかりな気が
気のせいかな

588 :132人目の素数さん:2020/03/12(木) 23:17:10 ID:mpirycu2.net
ぬるぽ氏に前月出題2の2の問題解説を頼みたいなんて、まさかねそんなこと言えない

589 :132人目の素数さん:2020/03/13(金) 06:57:36.79 ID:l20VjRfO.net
>>586
「一定の速度で」の意味だろうね。

「等速度」と云うが、PとQの速度が等しいワケぢゃないだろう。
「等速〜〜」 物理・力学・運動学でよく出てくるが・・・・誤訳か

590 :132人目の素数さん:2020/03/13(金) 07:34:36.59 ID:l20VjRfO.net
http://ejje.weblio.jp/content/等速直線運動
http://detail.chiebukuro.yahoo.co.jp/qa/question_detail/q137472283

uniform は 定速 だろうな。

591 :132人目の素数さん:2020/03/13(金) 20:40:03 ID:SLEyqjAe.net
制服じゃないの?

592 :132人目の素数さん:2020/03/15(日) 12:16:02 ID:fW6Y8kSv.net
解くだけなら簡単な「エレガントな形で書け」と要求してくる問題に興味が持てない某

593 :132人目の素数さん:2020/03/15(日) 18:00:19 ID:fW6Y8kSv.net
おい出題2!!!

オモロイやないか

594 :132人目の素数さん:2020/03/16(月) 23:28:11 ID:S0sVjxPf.net
そうだろそうだろ

595 :132人目の素数さん:2020/03/17(火) 08:28:21 ID:YLgNb34d.net
おまえら、どうやって解いたの?

596 :132人目の素数さん:2020/03/17(火) 13:26:44 ID:YLgNb34d.net
解答書いてね

597 :132人目の素数さん:2020/03/18(水) 00:04:41 ID:JVY7UXrj.net
先月問題2の2は誰も解説してくれない
たぶん、みんな俺と同じで問題の意図を汲み取れなかったと見た

598 :132人目の素数さん:2020/03/29(日) 02:43:33 ?2BP ID:X8OM/dxL.net
sssp://img.5ch.net/ico/nida.gif
数学上から要約でないとプログラミングとか結果の世界との整合性で嘘だよ数検今の所最年少とか違う視点でないとそんなとこ誰も要求してないし

599 :132人目の素数さん:2020/03/29(日) 02:44:11 ?2BP ID:X8OM/dxL.net
sssp://img.5ch.net/ico/nida.gif
と言うか近視の数学になってて近視観点は一々近視観点だとその言葉で集合も分かるから

600 :132人目の素数さん:2020/04/05(日) 13:18:56 ID:8LNbG9kY.net
一般のnの最少性は難問ぞかし

601 :132人目の素数さん:2020/04/08(水) 17:16:28 ID:+GhwHc0A.net
いや、簡単だろ

602 :132人目の素数さん:2020/04/08(水) 19:57:00 ID:RB38l11p.net
グラフ理論よくわからん
証明の書き方の作法がわからん

603 :132人目の素数さん:2020/04/08(水) 23:25:07 ID:Icgs18dA.net
>>601
拙者しょぼんでおじゃる

604 :132人目の素数さん:2020/04/09(木) 14:37:48 ID:FFP+Q2M6.net
小瓶のビーズの色がどのような組合わせであったとしても???
この文章の場合って色だけに着目するのか?それとも個数も?

605 :132人目の素数さん:2020/04/09(木) 18:02:32 ID:pbzhwLWN.net
ビーズの総数のみを指定します。(≧40)
何色あるのか、各々の色が何個あるか(次数列)は指定せず、
任意の組合わせが起こり得るとします。

606 :132人目の素数さん:2020/04/09(木) 23:01:56 ID:NHqOYdjo.net
もう翌月の問題に進んでるのかーい

607 :132人目の素数さん:2020/04/10(金) 01:50:57 ID:IAsBrfBV.net
10日でござる。
桜もコロナも満開でござる・・・・orz

2020年4月号 講評

■出題1
 P(t)、Q(t) は定速度で動く動点で、P(0)=A、Q(0)=D、P(1)=C、Q(1)=B とする。
 tがすべての実数に亘って動くとき、直線AQ(t)と 直線BP(t)の交点 X(t)となる
 点の集合を表わす式を求める問題。

 2次曲線になるのはウスウス分かってても、要領よくやらないと2次分数式と格闘するハメになる。
 小生は「外積」あるいは「有向面積」なるものを勝手に定義したが、泥沼流は不可避であった。

■出題2
(1)●の飽和性から、n点のときの本数が[3n/4]本以下なのは明らか。
  あとはこれを実現する例を挙げればよい。
 * 化学を習った人は●を3価の元素(窒素、リンなど)と思えばよい(?)
(2)それ以上は線が曳けなくなる本数の下限
  n=6 の場合は3本?
  一般のnの場合の最小性は難しそう >>600
 「6」「9」「P」「の」形3つと中心点の7点で「飽和化合物」になるから
 下限は[3(n+1)/7]本か?

今月は(今月も)難問であった…orz

608 :132人目の素数さん:2020/04/10(金) 05:33:08 ID:IAsBrfBV.net
出題1
 直線ADと直線BCの交点Eは軌跡Γの極で、ABは極線。
 Eを通る直線Lと直線ABとの交点をF、直線Lと軌跡Γとの交点をX,Yとすると
 {E,X,F,Y}は調和点列。
 点Aから見れば{E,X,F,Y} ≡ {D,Q,B,?}
 点Bから見れば{E,X,F,Y} ≡ {C,P,A,?}
なんてやるんだろうな・・・・と推測

609 :132人目の素数さん:2020/04/11(土) 11:18:29 ID:NNRlgNBR.net
出題1は一次変換を使うとよさげ

>下限は[3(n+1)/7]本か?
n=7*a+b (a∈N0, b∈{0,1,2,3,4,5,6})として3*a+floor(b/2)が最小と思ったが、
しばらくして[3(n+1)/7]とエレガントに書けることを知って落ち込んだ
で証明はどうする

610 :132人目の素数さん:2020/04/11(土) 11:23:26 ID:NNRlgNBR.net
>>607
>今月は(今月も)難問であった…orz

先月の出題2(2)、いまだによくわからん

611 :132人目の素数さん:2020/04/11(土) 14:41:46 ID:jVXfLHUH.net
(1)
シルヴェスターの慣性律を使えば分類できる(?)

(2)
3本の線で7点が飽和するから、それより大きくはないけど。


⦿  あるいは 「三つ巴」 >>607


それより小さい例がないのか、それが問題。。。

612 :132人目の素数さん:2020/04/12(日) 17:56:10.17 ID:5mGdhAyE.net
n=6も場合分けで面倒
一般のnはどうする

オイラーの公式
領域数と鳩ノ巣原理
これらを武器になんとかならんか
mod7での場合分けくらいは頑張るから

613 :132人目の素数さん:2020/04/13(月) 00:45:51 ID:PNtjkUIN.net
nが5で割り切れるなら



を n/5 組で 2n/5 本。
ただし、余りがあるときは、隔離線が要る・・・・

614 :132人目の素数さん:2020/04/13(月) 12:22:05 ID:PNtjkUIN.net
・2本/5点 >>613
をなるべく多くし、
・3本/7点 >>611
・1本/2点  (;)
も使うと下限は[2(n+2)/5]本か? (n≠1,3,8)

615 :132人目の素数さん:2020/04/14(火) 18:11:13.77 ID:30ihBTdn.net
おまえら、離散数学分かるのか?

616 :132人目の素数さん:2020/04/14(火) 23:02:46 ID:ZGzdde0B.net
オイラは分からん

617 :132人目の素数さん:2020/04/15(水) 23:05:18 ID:nzReXqSE.net
>>614
n=20を8本で済ますグラフ書ける?

618 :132人目の素数さん:2020/04/16(木) 00:33:52 ID:Fekx2b8P.net
 9


に目玉を入れた物を4組.... >>613

619 :132人目の素数さん:2020/04/16(木) 00:51:29 ID:Arl3WU5z.net
>>618
なる
奥深いなこの問題
証明しようにも、予想を超えるものがすぐ現れるw

620 :132人目の素数さん:2020/04/16(木) 03:18:17 ID:Fekx2b8P.net
同感でござる。
> ・各線はSの或る点からスタートし、Sの点をちょうど1つ通過し、Sの点で終わる。
これの意味がなかなか分かりづらい。
「ちょうど1つ通過し」と云うのがミソで、これが不可能ならOKだろうな。

621 :132人目の素数さん:2020/05/09(土) 16:10:37 ID:ckZVSFK4.net
6月号到着、ジュウサンフリーはみんな分からなかったようで安心した笑

レジェンド一人を除いては

622 :132人目の素数さん:2020/05/10(日) 13:34:38 ID:zIWxqOun.net
>>607
はや10日でござる。
桜はとうに散ってしまい、雑草が伸び盛り・・・・
今年はGWも家でゴロゴロ

2020年5月号の解答予想

□出題1  43個(以上)

□出題2  2 -(1/2)^(n-1),
    X = {1,2,4,・・・・,2^(n-1)}のとき。

合ってるかな?

623 :132人目の素数さん:2020/05/10(日) 13:45:40 ID:LJKY+5o/.net
>>622
同意

624 :132人目の素数さん:2020/05/10(日) 16:45:56 ID:hSQ8Tr/A.net
出題2だけど、総乗→logのルートを通らずに論証できた?

625 :132人目の素数さん:2020/05/11(月) 05:19:07 ID:aBpWM8d5.net
5月号 出題1
 ビーズが42個で次数列が(31,7,3,1)の場合は不可能だよね。

626 :132人目の素数さん:2020/05/11(月) 15:26:32 ID:W8fJM+2O.net
ピーターフランクルwwwwwwww

627 :132人目の素数さん:2020/05/11(月) 20:07:23 ID:9rslPAOW.net
この大道芸人は良問多し

628 :132人目の素数さん:2020/05/12(火) 02:42:55 ID:/ZQUNP4x.net
四桁ってマ?

629 :132人目の素数さん:2020/05/17(日) 11:48:50 ID:VBRHc1qm.net
出題2は作業だな
作業をエレガントにやれってことか

630 :132人目の素数さん:2020/05/17(日) 13:02:12.52 ID:VBRHc1qm.net
>>624
> 出題2だけど、総乗→logのルートを通らずに論証できた?

指数の肩に乗せて総乗→log→積分の方法
これ以外の方法はないのか?エレガントな5ちゃねらーずコメントをもとむ

示すべきは
(1)≦2-(1/2)^(n-1)(≡Smと書く)
(2)等号条件を満たす集合Xが存在

求める逆数和をS(X)、集合{2^(i-1)}をX~とおく

■命題1
「集合Xの任意の部分和が相異なるとき、逆数和Sの最大値はSである」
ダイレクトに示せないので迂回する

■補題1
「X~は題意を満たし、Smを与える」
証明容易

■命題2
「Y≠X~かつ部分和が相異なる集合YはS<=Smを与える」
これが示せれば証明終わり。難しいのでまた迂回する

■補題2
「部分和が0以上2^n-1を渡る集合はX~に限られる」
証明容易

■命題3
「部分和が相異なるが0以上2^n-1を渡らない集合Y≠X~の逆数和はS<=Smとなる」
これが示せれば証明終わり。簡単そうに見えるが・・・

■命題4
「部分和が0以上2^n-1を渡らない集合YにおいてS<Smならば、
同じ部分和を与える相異なる2個の部分集合∈2^Yが存在する」
命題2,3の代わりにこれを示してもよい

631 :132人目の素数さん:2020/05/17(日) 13:05:32 ID:VBRHc1qm.net
>>630
めちゃくちゃ間違えとる。正しくは下記

■命題1
「集合Xの任意の部分和が相異なるとき、逆数和Sの最大値はSmである」

■命題4
「部分和が0以上2^n-1を渡らない集合YにおいてS>Smならば、
同じ部分和を与える相異なる2個の部分集合∈2^Yが存在する」

632 :132人目の素数さん:2020/05/17(日) 13:22:46 ID:VBRHc1qm.net
■命題3
「部分和が相異なるが0以上2^n-1を渡らない集合Y≠X~の逆数和はS<=Smとなる」

これが比較的とっつきやすそう。とりあえずアタックしてみる
X,Yの要素を小さいほうからxi=2^(i-1), yiと書く

■小補題1
「部分和が相異なるY≠X~を考える。このときyi≠xiなる最小のiが存在し、yi>xiとなる」
証明容易

■小補題2
「あるi=i0が存在し、i≧i0⇒yi≧xiならばS≦Sm」
当たり前

■命題(迷題)5
「小補題1のiをi0とおく。このとき全てのi≧i0でyi≧xiとなる」
これを示せれば証明終わり。成り立つのか不明だがi≦i0+2までは真

633 :132人目の素数さん:2020/05/17(日) 14:16:35 ID:VBRHc1qm.net
>>576
> 俺の考えは逆で、nが大きい方がfreeになりにくいんだが
> たとえばn=1で{1,5,7}には(1,7,7;5)の組が存在するが、2点が同一なので重三freeとなる
> しかしn=2では数字を組み替えることで、重三が構成できてしまう
> どこが間違ってる?

解答編読んで自己レス
 {1, 2, 5, 6}^nよりも {7}^m X {1,2,5,6}^(n-m)としてnCm通りで組み替えたほうが要素は大きくなる
たしかにそんな気もする _| ̄|○
お前はアホだと真っ向から宣告された気がする
今日は天気がいいので散歩でもしてくる

634 :132人目の素数さん:2020/05/17(日) 15:14:21 ID:VBRHc1qm.net
>>632
>■命題(迷題)5
>「小補題1のiをi0とおく。このとき全てのi≧i0でyi≧xiとなる」
>これを示せれば証明終わり。成り立つのか不明だがi≦i0+2までは真

解けないなら迂回。困ったときは帰納法にすがってみる

■命題6
「#Y≦kで(1)S≦Sm、(2)等号条件を満たす集合はX~に限られる⇒#Y=k+1でも(1)、(2)が成り立つ」
簡単ではなさそうなので捻ってみる

■小補題3
「あるa∈{0, 1, ....}を取り、部分和が相異なる集合Yの全ての元を2^a以上に制限したとき、
逆数和が最大となるYの最小元はy1=2^aである」
自明(に見える)

■命題7
「#Y≦kでy1=1、S≧Smを満たし、部分和が相異なるYはY=X~に限られると仮定する。
このとき、あるa∈{0, 1, ....}を取り、#Y’=k、y1=2^a、
部分和が相異なるY’の逆数和が最大となるのはY={yi|yi=2^{a+(i-1)}}のときに限る」
小補題3が言えるなら命題7も言えそう
命題7が言えれば証明は終わりそう


はてさて

635 :132人目の素数さん:2020/05/17(日) 15:16:51 ID:VBRHc1qm.net
>>634
すまん書き直し。やれやれ

>>632
>■命題(迷題)5
>「小補題1のiをi0とおく。このとき全てのi≧i0でyi≧xiとなる」
>これを示せれば証明終わり。成り立つのか不明だがi≦i0+2までは真

解けないなら迂回。困ったときは帰納法にすがってみる

■命題6
「#Y≦kで(1)S≧Sm、(2)等号条件を満たす集合はX~に限られる⇒#Y=k+1でも(1)、(2)が成り立つ」
簡単ではなさそうなので捻ってみる

■小補題3
「あるa∈{0, 1, ....}を取り、部分和が相異なる集合Yの全ての元を2^a以上に制限したとき、
逆数和が最大となるYの最小元はy1=2^aである」
自明(に見える)

■命題7
「#Y≦kでy1=1、S≧Smを満たし、部分和が相異なるYはY=X~に限られると仮定する。
このとき、あるa∈{0, 1, ....}を取り、#Y’=k、y’1=2^a、
部分和が相異なるY’の逆数和が最大となるのはY’={yi|yi=2^{a+(i-1)}}のときに限る」
小補題3が言えるなら命題7も言えそう
命題7が言えれば証明は終わりそう


はてさて

636 :132人目の素数さん:2020/05/17(日) 18:44:06 ID:jv4DNZp5.net
2020年3月号
■出題2
>>633 のやり方だと
#A = 4^(n-m)・C[n,m]= 4^(n-m) n!/{m!(n-m)!},
nを固定してmを変えると
m = n/5 - 0.3 のあたりで最大となり
 #A 〜{5/√(8πn)} 5^n,
∴ c < 5 で成立。

{1,2,5,6,7}^n に似てるけど微妙に違う。
重心のi成分が「7」なら他の3点のi成分は「7」しかない。
「7」の個数を決めておけば、他の成分は「7」以外になり、
重三を回避できた・・・
「1」についても同様かも?

637 :132人目の素数さん:2020/05/17(日) 19:25:15 ID:VBRHc1qm.net
>>635
命題7は2秒で反例が見つかる糞命題だったw
小補題3が言えれば命題6の帰納法が前に進むんだろうか

638 :132人目の素数さん:2020/05/17(日) 23:26:38 ID:jv4DNZp5.net
>>636
#A = 4^(n-m)・C[n,m]= 4^(n-m)・n!/{Γ(m+1)Γ(n-m+1)},

∂(#A)/∂m =(#A){H(n-m)- H(m)- log(4)}
 ≒(#A){∫[m+1/2, n-m+1/2]1/t dt - log(4)}
 ≒(#A){log(n-m+1/2)- log(m+1/2)- log(4)}
 = 0
より
 log(n-m+1/2)- log(m+1/2)- log(4)= 0,
 n-m+1/2 = 4(m+1/2),
 m = n/5 - 0.3

639 :132人目の素数さん:2020/05/18(月) 22:32:46.56 ID:rgdtIx2/.net
ジュウサンフリーはマニアックな問題だったなw

640 :132人目の素数さん:2020/05/19(火) 09:33:12 ID:C7hbQ2t7.net
>>636
> ∴ c < 5 で成立。

#{1,2,5,6,7}^n = 5^n
の元を「7」の個数mで分類すると m=0,1,・・・・,n の(n+1)とおりあるが、
m=0 と m=n を1つにまとめれば nとおり。
#Aが最大のものは
 #A >(1/n)・5^n
である。(実際はもう少し大きい)

c < 5 とすると
δ = log(5/c) > 0,

n > (-logδ)/{(1-1/e)δ} に対しては

nδ - log(n)= nδ - log(nδ) + logδ
 > nδ - nδ/e + logδ     (*)
 = n{(1-1/e)δ}+ logδ
 > - logδ + logδ
 = 0,
(5/c)^n =(e^δ)^n > n,
 #A >(1/n)・5^n > c^n,
∴ 常に YES。

* log(x)= 1 + log(x/e)< x/e,

641 :132人目の素数さん:2020/05/23(土) 22:51:42 ID:BHMTcO5g.net
ピーター難しいナー

642 :132人目の素数さん:2020/05/29(金) 00:28:38 ID:NJ9bSeqT.net
難しすぎないか今回のぴーたーは

643 :132人目の素数さん:2020/05/31(日) 10:03:50.40 ID:FN96o+kA.net
むずかしすぎないか?3度目になるが

644 :132人目の素数さん:2020/06/01(月) 05:40:40.79 ID:q3J6Hp+/.net
PCで解いても法則性が良くわからんなあ。条件を1つ緩めると簡単なんだが。

645 :132人目の素数さん:2020/06/03(水) 00:38:40 ID:GHpoXm6c.net
久しぶりに手も足も出ない

646 :132人目の素数さん:2020/06/04(木) 15:25:35.73 ID:FAkFQ0aS.net
ピーターは入れ替えるか2で割ると2^n(n=4.5.6.7.8.9.10.11.12.13)になるものを選べばいいんじゃないのか?

647 :132人目の素数さん:2020/06/04(木) 16:45:23.16 ID:I6SNebx8.net
と思うじゃん?

648 :132人目の素数さん:2020/06/04(木) 18:51:18 ID:hFhLQxyG.net
いや思わんよ

649 :132人目の素数さん:2020/06/07(日) 14:37:04.83 ID:esBiw1WP.net
ノーヒントつらいなーw

650 :132人目の素数さん:2020/06/07(日) 21:21:48.19 ID:XudJtBbg.net
解けた人いるのコレ

651 :132人目の素数さん:2020/06/09(火) 03:10:11.63 ID:oCR5MqlE.net
Nothing really matters to me,
anyway the wind blows.

http://www.youtube.com/watch?v=sBspSJWRT2E 05:58

652 :132人目の素数さん:2020/06/11(木) 00:43:29.46 ID:9MYLZRNv.net
難しすぎてコメントゼロ

653 :132人目の素数さん:2020/06/11(木) 04:05:02.72 ID:U1wqDpVl.net
出題1、結果だけならプログラム組めば出せる
無論、そこにエレガントさはカケラもない。

1から2倍と桁入れ替えで到達可能な数をリストアップすると、以下を除く4桁の数(5558個)になった。
これらが1に到達不可能なことを確かめるのは難しくない。

・3の倍数
・全ての桁が奇数
・全ての桁が2か6
・1114, 1141, 1411, 4111, 1118, 1181, 1811, 8111, 2228, 2822, 2282, 8222, 4444, 8888

654 :132人目の素数さん:2020/06/11(木) 04:16:45.02 ID:1i56AYPZ.net
まず、3の倍数であることは(1)(2)で変わらない。

最大n回(2)が可能、で考察すると
0回:全ての桁が奇数
1回:全ての桁が2か6、1114, 1141, 1411, 4111, 1118, 1181, 1811, 1110, 1101, 1011
これは下2桁が4の倍数+2、2で割って千の位が奇数、2で割って十の位が奇数、を考えればよい。

2倍→桁を入れ替え→2で割る、という手順でここから抜けられない数を順次リストアップしていくと
2回:2228, 2822, 2282, 8222, 4444, 2220, 2202, 2022
3回:8888, 4440, 4404, 4044
4回:8880, 8808, 8088
5回:なし

となって、上記に含まれない数は手順を選べば無限回(2)が可能。
蜂の巣原理から、その手順のどこかで桁の組合せが同じ数に戻る。
……までは考察できたが、それが1に到達可能な数という保証ができないな。

655 :132人目の素数さん:2020/06/11(木) 04:33:35.28 ID:TkJPwCTf.net
読み直してみると、途中で3桁に落ちるパターンが考慮から漏れているかな。
3桁でも同じようなことを考えることはできそうだが。

656 :132人目の素数さん:2020/06/11(木) 09:39:36.05 ID:2VKGJNso.net
・3の倍数
 1002から9999まで  3000個
・全ての桁が奇数 {1,3,5,7,9}
 5^4 - 208 (3の倍数) = 625 - 208 = 417個
・全ての桁が {2,6}
 2^4 - 4 (2が3個) -1 (6666) = 11個
・{1114} {1118} {2228}, 4444, 8888 = 14個
・{1110} {2220} {4440} {8880} は3の倍数 = 0個

ここまで 3442個 (到達不能)

・全体で
1000から9999まで 9000個

・上記のリストにないものは
9000 - 3442 = 5558個

657 :132人目の素数さん:2020/06/11(木) 12:35:16.85 ID:FIRrnvGD.net
1/2倍だけではなく2倍の操作も許し、途中で5桁以上になっても良いとすると、1に到達不可能なのは元の数が3で割り切れることと同値なようだ。
(5桁まで膨らむ場合をPCで計算。実は5555のみが例外的に到達不能だったが、桁数を増やせば行けるんじゃないか。)

つまり、桁数を無視すると(1)(2)でつながるネットワークの頂点はmod3=1,2の集合ということになる。

あとはその中から2倍操作をしなくても1に到達するものを選ぶという話になるな。

658 :132人目の素数さん:2020/06/11(木) 12:46:44 ID:FIRrnvGD.net
>>657
5555も6桁使えば行けたわ。

5555 (×2^5)->177760->177706->88853->53888->26944->24496->12248->6124->1624->812->128->略

659 :132人目の素数さん:2020/06/11(木) 14:03:33.02 ID:8Ty+yyMM.net
いまどきは 蜂の巣原理 というのか

660 :132人目の素数さん:2020/06/11(木) 14:18:21.90 ID:TS8a5l3a.net
うん、鳩の巣原理だな
全桁奇数は回避できるが無限ループで1に到達できず、という可能性を排除できないけどなんかいえねーかなーと……

661 :132人目の素数さん:2020/06/11(木) 22:18:58 ID:JAzk3B8F.net
蜂のほうがイメージしやすくてgood

662 :132人目の素数さん:2020/06/12(金) 21:34:07.64 ID:MxsSZpyi.net
でも蜂の巣って一つの巣に何百何千の蜂が住んでいらっしゃるから
本来の鳩ノ巣原理のニュアンスとは違ってこなくないかい

663 :132人目の素数さん:2020/06/13(土) 03:41:34.30 ID:7U7UnxDr.net
今月号の出題2だが、(3)が一番簡単だろw

664 :132人目の素数さん:2020/06/13(土) 06:02:34.44 ID:R063DAmj.net
>>663
撤回するわw

665 :132人目の素数さん:2020/06/13(土) 07:13:32.00 ID:KjHAbDUS.net
>>662
蜂の巣は誰でも知ってるでしょ
でも鳩ノ巣を見たことあるか?

666 :132人目の素数さん:2020/06/13(土) 07:45:52.41 ID:+MT8Zxaq.net
pigeonhole principle

pigeonhole:
鳩がせっせと作る巣ではなくて、(区分けされた)巣箱
転じて、棚などの区画

鳩の巣
https://gaiju-seikatsukyukyusya.com/pigeon/information/post-72/

667 :132人目の素数さん:2020/06/13(土) 10:17:46.82 ID:KjHAbDUS.net
日本語の鳩ノ巣じゃまずいから引き出し論法にしたのかもしれんが、もっとまずいだろ

それに比べて蜂の巣はどうだ
穴と蜂の対応関係が絶妙にイメージしやすいではないか

668 :132人目の素数さん:2020/06/13(土) 10:19:03.33 ID:KjHAbDUS.net
正直ピーターは手も足も出ず、こんなコメントしかできないのが恥ずかしいわ

669 :132人目の素数さん:2020/06/13(土) 10:29:15.79 ID:KjHAbDUS.net
ちなみに俺のアプローチを言うと、4桁から3桁へ、3桁から2桁へ移る数のうち、1へ到達可能な数を調べ上げた

例えば2桁の数は58,61,64,85,92に限られるので、
÷2でこれらに移行できる3桁の数Tは上の5数の2倍。
同様にして、最終的に上の3桁5数に到達できる4桁の数Qも多少の根気で調べ上げることができる(1000以上1999以下)

あとはQに到達できる4桁の数を網羅すればよい
この網羅は全く容易でない
ゆえにこれらの努力が無駄であることが示された(証明終)

670 :132人目の素数さん:2020/06/13(土) 15:25:08.11 ID:PwdDNtuf.net
蜂の巣というたらあのでっかい塊のこと
巣と巣穴を混同してはいけない

だから蜂の巣穴論法というべき

671 :132人目の素数さん:2020/06/13(土) 19:05:04.99 ID:YB3Es/uG.net
規則性や保存量を一生懸命探したが見つからなかったな

672 :132人目の素数さん:2020/06/15(月) 17:19:30.01 ID:dl2LK2OC.net
今月の問1は例のアレか

673 :132人目の素数さん:2020/06/15(月) 21:42:59.32 ID:w5Hmykx3.net
>>672
なんなん

674 :132人目の素数さん:2020/06/16(火) 02:44:29 ID:tyACuXlX.net
ワークマンの女子高生問題か

675 :132人目の素数さん:2020/06/16(火) 15:11:23 ID:2YfX/iX0.net
ワークマンの作業着を着た女子高生に萌える

676 :132人目の素数さん:2020/06/17(水) 22:41:26.81 ID:MyAVUYgD.net
ワークマン?何の話?

677 :132人目の素数さん:2020/06/17(水) 23:47:40.09 ID:NyxAI4wb.net
どおくまんの男子大学生問題

678 :132人目の素数さん:2020/06/18(木) 08:14:46.02 ID:yuRO46b1.net
嗚呼!! 花の応援団(1975〜1979)
チト古杉....

679 :132人目の素数さん:2020/06/20(土) 17:08:47.77 ID:XzEjTdKU.net
出題1むずすぎるんだが
nは絞り込めても表が作れる気がせん

680 :132人目の素数さん:2020/06/20(土) 19:18:45.67 ID:jbwdAnTe.net
>>679
十分条件を求められているかというとハテナでございます

681 :132人目の素数さん:2020/06/21(日) 00:16:16.63 ID:Ah8GvzYZ.net
出題2は(3)が1番簡単な気がするんだが
読み違えてるのかな

682 :132人目の素数さん:2020/06/21(日) 00:17:23.72 ID:Ah8GvzYZ.net
ああ勘違いに気付いた

683 :132人目の素数さん:2020/06/21(日) 12:29:49 ID:Ah8GvzYZ.net
出題2はヒントが親切過ぎる

684 :132人目の素数さん:2020/06/21(日) 19:54:59.25 ID:onkrGecf.net
が、出題2(3)は骨がありそうだ。良問!

685 :132人目の素数さん:2020/06/22(月) 18:45:51.31 ID:TPfYUS8O.net
>>679 >>680
この話に心当たりがあったのでもうググって答え見た。
資料みて思うにコレは表の存在を自分で導出するのは無理だ。
なんせ表の存在だけで論文になってるレベル。
この表をいかに簡単に作れるかでひとつのテーマになってる。
(1)も(2)も存在しうるならこの2つしかないまでで良いんだろうと思う。

686 :132人目の素数さん:2020/06/22(月) 22:09:11.35 ID:nOEAAo4S.net
先月号の出題2が全然話題にならないのは
個々の住人には簡単すぎたから?

687 :132人目の素数さん:2020/06/22(月) 22:55:21.87 ID:dX9g2rrw.net
>>686
早々に解けたことは覚えてるけど全く記憶にない
簡単すぎたのか、ピーターに時間を費やしすぎたか

688 :132人目の素数さん:2020/06/22(月) 22:58:09.25 ID:dX9g2rrw.net
>>687
いま問題見て思い出した
大学受験みたいだなーと思った
こういう問題はツマラン

689 :132人目の素数さん:2020/06/22(月) 23:02:16.35 ID:ZgWK0qfa.net
>>685
与えられた問題を解くだけなら簡単そうだ
やはり出題2(3)だな今月の山場は

690 :132人目の素数さん:2020/06/23(火) 06:17:24.18 ID:EJLy4HwQ.net
先月の出題2は x^2+y^2=z^2 の一般整数解を知ってればつまんない問題ではないかな
交点の確認から外接条件、と誘導もしっかりしてて、あとは計算するだけに見える

691 :132人目の素数さん:2020/06/23(火) 17:29:54.58 ID:DYKTirLl.net
この雑誌って完全に数学科向け?
物理出身だが買ってみたいんだか

692 :132人目の素数さん:2020/06/23(火) 17:47:03.19 ID:lKA9/sJW.net
医学科出身だけど愛読してるお

693 :132人目の素数さん:2020/06/23(火) 19:25:40.67 ID:F5pXlGP9.net
出題2の(3)、多分解けた。
条件をみたす「島」を思い付くと意外と気持ち良い。

694 :132人目の素数さん:2020/06/23(火) 22:51:05.43 ID:3g7BJMko.net
中学生だけど、愛読してる。うふ

695 :132人目の素数さん:2020/06/24(水) 15:16:16.24 ID:W2vxdtjk.net
島の面積としてあり得る数は決定できますか

696 :132人目の素数さん:2020/06/24(水) 22:46:45 ID:LOdimE5b.net
>>693
早いじゃないか
島であること、有界でないこと、の2ステップだな
なんとなくイケる気がするんだが2トライして失敗
次は何を試そうかなあ(楽しい)

697 :132人目の素数さん:2020/06/24(水) 23:34:38 ID:sX2rVvuO.net
>>695
任意の大きさができるみたい。

698 :132人目の素数さん:2020/06/29(月) 22:16:42.93 ID:WDXLK/OL.net
2の(3)やはりむずかし

699 :132人目の素数さん:2020/06/30(火) 21:34:50 ID:ECJqkbpx.net
>>685
Steiner system S(5,8,24) の自己同型群は Mathieu群 M_24 で5重可移なんだね。
しかし一般に、どのようなパラメータに対して block design が存在するか
という問題は非常にむずかしい問題で、Combinatory Analysis のもっとも
重要な問題の一つらしい。。。

700 :132人目の素数さん:2020/06/30(火) 22:53:11.48 ID:S9bjrUT4.net
コラコラ

701 :132人目の素数さん:2020/07/04(土) 20:08:14.29 ID:TXq/8hPs.net
>>693
できたんか?ホンマにできたんか

702 :693:2020/07/05(日) 00:45:59.11 ID:izl2mYWJ.net
>>701
出来たよ。回答は今月8日の締め切り後に出すつもりだけど。
ヒント(といえるかはわからんが)として、明日ぐらいに回答に使った定理ぐらいは書いてもいいよ。
書いて欲しい人がいればの話だけど。

703 :132人目の素数さん:2020/07/05(日) 00:53:28.38 ID:277VsiZx.net
コラコラ

704 :132人目の素数さん:2020/07/05(日) 17:25:00.87 ID:uP/Fh04a.net
ヒントもあかん。締め切りまで我慢しなさい。

705 :132人目の素数さん:2020/07/06(月) 00:30:19.65 ID:HPDcrjtp.net
明日になった。(?)
・中国剰余定理は、主張をきちんと述べれば証明する必要はありません。
・ディリクレの算術級数定理も証明なしに用いて構いませんが、用いないで済めばその方が良しとします。
らしいぞ。

706 :132人目の素数さん:2020/07/06(月) 14:30:24.01 ID:rHgmpUJU.net
3では使わないよね

707 :132人目の素数さん:2020/07/06(月) 19:54:44.64 ID:CzRhbiNT.net
すまんが706は忘れてくれ
これはヒントだろうか?
否。断じて否である。

708 :132人目の素数さん:2020/07/06(月) 20:42:31 ID:kKKJVE+/.net
1箇所蓋が出来れば終わるんだが

709 :132人目の素数さん:2020/07/06(月) 21:20:08.76 ID:1woOkUsD.net
できた

710 :693:2020/07/09(木) 00:16:57.17 ID:kJ0rN7sT.net
出題2の(3)の回答
rを素数とする。rより大きな2r+2個の素数p(0),p(1),…,p(r),q(0),q(1),…,q(r)をとる。

正の整数aを以下の様に定める。

a≡0 (mod r)
rより小さな任意の素数sに対して
a≡1 (mod s)
a≡1 (mod p(0)p(1)…p(r) )
a≡-1 (mod q(0)q(1)…q(r) )

また正の整数bを以下のように定める。

b≡0 (mod a)
0≦i≦rをみたす任意の整数iに対して
b≡-i (mod p(i)q(i) )

明らかに、gcd(a,p(0)p(1)…p(r)q(0)q(1)…q(r) )=1
であることに注意すると、a,bは無数の値をとりうることがわかる。

ここで、r-1個の点(a,b+i)達は、「面積」r-1の「島」であることを示す。…※
(ただし、1≦i≦r-1)

711 :693:2020/07/09(木) 00:18:02.23 ID:kJ0rN7sT.net
出題2の(3)の回答続き

命題※の証明
3r+3個の点(a-1,b+i),(a,b+i),(a+1.b+i)達を考える。(ただし0≦i≦r)

a-1,b+iは、少なくとも公約数p(i)を持つので、点(a-1,b+i)はCの要素ではない。
またa+1,b+iは、少なくとも公約数q(i)を持つので、点(a+1,b+i)はCの要素ではない。
点(a,b+i)を考える (ただし0≦i≦r)
i=0,rのとき
a,b,b+rは、少なくとも公約数rをもつので、点(a,b),(a,b+r)はCの要素ではない。
1≦i≦r-1のとき
gcd(a,b+i)=gcd(a,i)=1
(∵aの素因数は、すべてr以上であり、かつiの素因数は、すべてrより小さいから。)
だから、点(a,b+i)はCの要素である。
したがって、命題※が正しいことがわかる。

素数rはいくらでも大きくとれるから、いくらでも大きな「面積」の「島」の存在が言えた。

712 :693:2020/07/09(木) 00:20:08.67 ID:kJ0rN7sT.net
出題2の(2)は>>710-711でr=2とおけばよい。

713 :132人目の素数さん:2020/07/09(木) 01:07:25 ID:XFAfLnLw.net
任意の大きさを作る解

補題
自然数nにたいし自然数a,b,k,eと素数p,q,ri,si (0≦i≦n+1)が

n<p<q、
(pq)^e ≡ 1 (mod ri)、(pq)^e ≡ -1 (mod si)、
pqk + ap ≡ -i (mod ri)、pqk + ap ≡ -i (mod si)、
(pqk + bq - 1) - (pqk + ap + 1) + 1 = n、

を満たす時、{((pq)^e, pqk + ap + i) | 1≦i≦n}は大きさnの島である。

∵) pqk + ap + i = pqk + bq - (n+1-i) は1≦i≦nであるとき、p,qのいずれとも互いに素であるから(pq)^eとも互いに素であるから((pq)^e, pqk + ap + i) は陸地である。
pqk + ap + 0はpの倍数、pqk + ap + n+1 = pqk + bqはqの倍数であるから陸地ではない。
0≦i≦n+1にたいしpqk + ap + i、(pq)^e-1は共にriの倍数であるから((pq)^e-1, pqk + ap + i)は陸地ではなく、pqk + ap + i、(pq)^e+1は共にsiの倍数であるから((pq)^e+1, pqk + ap + i)は陸地ではない。□

そこで補題の条件を満たす組みが存在する事を示せば良い。
eiをe1=3、e(i+1)=ei(pq)^eiと定めるとき、
((pq)^e(i+1)-1)/((pq)^ei-1) ≡ (pq)^ei ≡ 1 (mod (pq)^ei-1)、
((pq)^e(i+1)+1)/((pq)^ei+1) ≡ (pq)^ei ≡ -1 (mod (pq)^ei+1)
により相異なるi,jに対し((pq)^e(i+1)-1)/((pq)^ei-1)、((pq)^e(j+1)-1)/((pq)^ej-1)は互いに素である。
そこで((pq)^e(i+1)-1)/((pq)^ei-1)の素因子からri、((pq)^e(i+1)+1)/((pq)^ei+1)の素因子からsiを選べば良い。
この時第3式を満たすkは中国の剰余の定理によってとれる。
第4式はユークリッドの互除法によりとれる。

714 :132人目の素数さん:2020/07/09(木) 13:36:27 ID:Z0Pg/xy2.net
8月号の問題2は一見ラマヌジャン的な唐突さを感じるな

715 :132人目の素数さん:2020/07/09(木) 16:50:56 ID:2IXNWIKB.net
>>710
M = (rより小さな素数s すべての積)・p(0)p(1)・・・・p(r),
N = q(0)q(1)・・・・・q(r),
とおく。

仮定により MとNは互いに素だから、
 M u - N v = -1 をみたす整数 u,v がある。
(ユークリッドの互除法で得られる。)
 2Mu+1 = 2Nv-1,
一方、仮定により MNとrは互いに素だから
 k・(-MN) ≡ 1  (mod r)
となる整数 k がある。そこで
 a = (2Mu+1)(1+kMN) = (2Nv-1)(1+kMN),
とおく。
-------------------------
i ≦ r < p(i), q(i) より i と p(i)q(i) は互いに素。
 L(i) = aΠ[j≠i] p(j)q(j)
とおく。
仮定により L(i) と p(i)q(i) は互いに素だから
 k(i)L(i) ≡ -i  (mod p(i)q(i))
となる k(i) がある。そこで
 b = Σ[i=0,r] k(i)L(i) = aΣ[i=0,r] k(i)Π[j≠i] p(j)q(j).
とおく。

716 :132人目の素数さん:2020/07/10(金) 14:51:42.70 ID:F5aduSPW.net
作者土岡君か

717 :132人目の素数さん:2020/07/10(金) 15:17:59.62 ID:4MyAGV9b.net
7月号問1は本誌読んでるかの抜き打ちチェックだったな

718 :132人目の素数さん:2020/07/10(金) 15:28:00.61 ID:++P0OH09.net
カークマンの女学生問題そのものだよな。

719 :132人目の素数さん:2020/07/10(金) 20:40:26.35 ID:zO+7FxbR.net
数セミ増刊「数学100の問題」日本評論社 (1984)
 p.60-61 (山本幸一)

中村義作ほか:「数理パズル」中公新書 (1976)
 p.178-187

別冊 数理科学「群とその応用」サイエンス社 (1991)
 p.36-40 (永尾 汎)  p.150-158 (景山三平)

永尾 汎:「群とデザイン」岩波 数学選書 (1974)

720 :132人目の素数さん:2020/07/10(金) 21:44:07.30 ID:/JnRCRri.net
>>716
ご朋友でいらっしゃる?

721 :132人目の素数さん:2020/07/11(土) 00:11:06.63 ID:6WYuhTkh.net
問題が修正されてるな

722 :693:2020/07/11(土) 02:10:00 ID:Xb1OTyfv.net
折れも任意の自然数nに対して面積nの島を考えてみた。
(文字や考え方など、大分>>713氏をパクっているがw)

nを整数、p.qをn<p<qをみたす素数とする。
a,e,r(i),s(i)を下記のようにとる。(ただし、0≦i≦n+1)
a≡0 (mod p)
a+i≡0 (mod r(i)s(i) )
a+n+1≡0 (mod q)
eはある正の整数
r(i)達は(pq)^e-1を割り切る相異なる素数。
q(i)達は(pq)^e+1を割り切る相異なる素数。
」…♪
このとき、( (pq)^e,a+i)達は「面積」nの「島」である(ただし、1≦i≦n)。

3n+6個の点( (pq)^e-1,a+i)、( (pq)^e,a+i)、( (pq)^e+1,a+i)を考えると
(ただし、0≦i≦n+1)
上記のようにとれば、(pq)^e-1,a+iは少なくとも公約数r(i)を持ち
(pq)^e+1,a+iは少なくとも公約数s(i)を持つので、
( (pq)^e-1,a+i),( (pq)^e+1,a+i)達はCの要素ではない。
(pq)^e,aは公約数pを持ち、(pq)^e,a+n+1は公約数qを持つ。
aより大きな最小のpの倍数はa+p>a+nでありかつ
a+n+1より小さな最大のqの倍数はa+n+1-q<aだから
a+1,…,a+nはp.qいずれでも割り切れない。
したがって、1≦i≦nのとき( (pq)^e,a+i)はCの要素である。

723 :693:2020/07/11(土) 02:24:38.65 ID:Xb1OTyfv.net
あとは、♪が実際に構成可能であること。
n,p,qは明らか。
bをpqの倍数とするとき
gcd( b^(pq)士b^(pq-1)+b^(pq-2)+…+1,b干1)=gcd(pq,b干1)=1
b^(pq)士b^(pq-1)+b^(pq-2)+…+1≡1 (mod 2)より、
b^(pq)±1にはb±1にはない奇数の素因数が必ず存在する。
このことを踏まえると、以下のような構成が出来る。
e=(pq)^(n+2)とおく。すると素数r(i),s(i)達を下記のようにとれる。
r(i)はpq^((pq)^(i+1))-1を割り切るが、pq^((pq)^i)-1を割り切らない奇素数。
s(i)はpq^((pq)^(i+1))+1を割り切るが、pq^((pq)^i)+1を割り切らない奇素数。
上記のようにおくと、(pq)^e-1を割り切る相異なるn+2個の素数r(i)達と
(pq)^e+1を割り切る相異なるn+2個の素数s(i)達を構成できる。
またgcd((pq)^e+1,(pq)^e-1)=2だから、r(i)達とs(i)達も相異なることわかる。
以上より、aは中国剰余定理より存在が言える。

724 :132人目の素数さん:2020/07/11(土) 02:37:12 ID:Xb1OTyfv.net
あと個人的な推測だが、たぶん>>713氏の回答には表向きでは使われてないけど
Zsigmondyの定理が背景にあると思った。
折れも考えてみたけど、>>713と似たものになっちまったw

折れも>>713もZsigmondyの定理の使用を表向きには避けたため、
eの値は非常に大きなものになっている。
Zsigmondyの定理を用いれば、eの値は大分小さくなると思う。
興味がある人はチャレンジしてみるといいんじゃないかな。

Zsigmondyの定理のステートメント
https://en.wikipedia.org/wiki/Zsigmondy%27s_theorem

725 :132人目の素数さん:2020/07/11(土) 02:40:24 ID:Xb1OTyfv.net
>>724の補足だが
>折れも考えてみた
これはZsigmondyの定理を用いないeの構成方法

726 :132人目の素数さん:2020/07/11(土) 07:05:15.58 ID:ZnhtLn45.net
〔ジグモンディの定理〕
a>b は互いに素な整数、nは正の整数とする。(*)
このとき、a^n−b^n を割り切るが a^k−b^k (1≦k≦n−1) を割り切らないような素数pが存在する。
* 但し、以下の場合を除く。
 (n=1, a-b=1) (n=2, a+b=2^k) (n=6, a=2, b=1)

n→2n とすれば・・・・

〔和のジグモンディの定理〕
a>b は互いに素な整数、nは正の整数とする。(*)
このとき、a^n+b^n を割り切るが a^k+b^k (1≦k≦n−1) を割り切らないような素数pが存在する。
* 但し、以下の場合を除く。
 (n=3, a=2, b=1)

上記の数を "primitive prime divisor" と呼ぶらしい。

727 :132人目の素数さん:2020/07/11(土) 15:52:13 ID:4lYEolum.net
島の形だけ示した宝の地図を提示してその座標を特定する宝探しゲームができないかと思ったが同じ形の島が無限個あるからだめだった

728 :132人目の素数さん:2020/07/12(日) 08:37:42 ID:TfKcF3sP.net
色んな方法があるんだね
けどやはり算術級数定理を使うのが1番シンプルと思った

729 :132人目の素数さん:2020/07/12(日) 08:47:35 ID:lCCkuzVr.net
まぁそうだな。
オレは素数定理使えばいいと思うんだけど出題者が「使わない方がなお良い」って言ってるからなぁ。
素数定理使っていいなら>>713で先にqi,riとって後でp,qとってeなんか全く必要なくなるし。

730 :693:2020/07/13(月) 06:18:19 ID:r0ffJNeq.net
>>728
もし、良かったら解答書いてくれないかな。
折れ自身、まず算術級数定理を使おうとしたけど、どうしてもうまくいかずに挫折したんだ。
方針転換して、解答>>710-711を見つけたがw

731 :132人目の素数さん:2020/07/13(月) 12:32:27 ID:tR49RdlH.net
ヨコだけど>>713でまず任意に相異なる奇素数ri,si (0≦i≦n+1)を先にとっておいて、それから任意に別の奇素数p>nをとり、最後にq>pを算術級数の素数定理で
pq ≡ 1 (mod ri)、pq ≡ -1 (mod si)
ととればe=1である解を与える。

732 :132人目の素数さん:2020/07/13(月) 19:45:11.01 ID:qNTcwU4s.net
>>731
それそれ。中心のX座標を2素数の積の形にできる。

733 :132人目の素数さん:2020/07/13(月) 19:50:25.83 ID:qNTcwU4s.net
>>710
>rより小さな任意の素数sに対して
>a≡1 (mod s)

これで逃げられるならこのほうが賢いわな

734 :693:2020/07/14(火) 01:34:46.10 ID:L+AsbTGF.net
>>731
ありがとう

735 :132人目の素数さん:2020/07/15(水) 01:51:09 ID:5dJcX4Gp.net
先月号出題2の(3)の出題者の意図した解は多分下記だと思う。

>>713>>722と同様にまず素数r(i)とs(i)を決めてやって
上記の素数とは別にnより大きな素数pをとる。
最後にpq≡1 (mod r(0)…r(n+1) )、pq≡-1 (mod s(0)…s(n+1) )
みたす整数qを中国剰余定理より求める。
>>731氏がやった通り、算術級数定理よりqの値を素数にすることができる。
(たぶんこれが、出題者の意図した算術級数定理を用いた解答)

ただ実はqが素数である必要は実はなく、qがn以下の素数で割り切れなければ
題意を満たすことが言える(その際に>>693と似た議論を用いる)。
例えばn以下の任意の素数tに対してさらにq≡1 (mod t)をみたす。
ように正の整数qをとればよい。
(そしてこれが、たぶん出題者の意図した算術級数定理を用いない解答)

736 :132人目の素数さん:2020/07/15(水) 05:09:10.83 ID:5dJcX4Gp.net
× >>713>>722と同様に
〇 >>713>>722

737 :132人目の素数さん:2020/07/15(水) 05:12:22.69 ID:5dJcX4Gp.net
あと、>>735で「n以下の任意の素数t」と書きましたが、
勿論、tはr(i),s(i)以外の値をとりますw

738 :132人目の素数さん:2020/07/15(水) 16:52:23.27 ID:dm0aC4Wz.net
ネットの掲載おそくない?

739 :132人目の素数さん:2020/07/15(水) 20:40:55.97 ID:soJPT0o5.net
>>738
え?締め切り翌日には掲載されてたけど。

740 :132人目の素数さん:2020/07/15(水) 21:12:39 ID:5HGaLpkt.net
今月のエレ解、「数学」とか「エレガント」のタグがついてないね

741 :132人目の素数さん:2020/07/15(水) 21:27:24 ID:+VNqdAmi.net
http://www.web-nippyo.jp/elegant/
          ↑
   この間に /tag/ を挟むと出て来ないですね。

742 :132人目の素数さん:2020/07/15(水) 22:19:58 ID:dm0aC4Wz.net
見れたわありがとう

743 :132人目の素数さん:2020/07/21(火) 22:14:25.83 ID:+NGICN8G.net
正解者がとっても少ないの

744 :132人目の素数さん:2020/08/05(水) 12:11:25.71 ID:XCXdhvGW.net
>>714
個人的にはなかなか楽しめた問題だったわ。

745 :132人目の素数さん:2020/08/06(木) 19:52:51 ID:1/AtprVO.net
これならどうなるでしょう?

485132人目の素数さん2020/07/22(水) 09:39:00.16ID:h9mML7y2>>486>>487
今月のエレガントな問題を少し変更したやつ

[x]はx以下の最大の整数
[[x]]はx以上の最小の整数
logは自然対数とする

n≧2のとき、次は常に成り立ちますか?
[2^(1+1/n)/(2^(1/n)-1)]=[[2n/log2]]

746 :132人目の素数さん:2020/08/06(木) 20:25:33.99 ID:zAQTDrQb.net
>>745
解けるよ。解き方は元の問題とほぼ同じだからここには書かないけど。
ていうか、これどこのスレ?

747 :132人目の素数さん:2020/08/06(木) 20:42:30.44 ID:1/AtprVO.net
>>746
いや、これは元の問題と同じ方法では解けないと思うんだが何かを勘違いしてるか

https://rio2016.5ch.net/test/read.cgi/math/1594131967/485

748 :132人目の素数さん:2020/08/06(木) 21:08:53 ID:zAQTDrQb.net
>>747
うーん、本当に同じ手法で解けたんだけどな。俺何か勘違いしてるかなあ。ただ、かなり邪道な手法だが。
締切以降に書くわ。

749 :132人目の素数さん:2020/08/06(木) 21:14:17 ID:1/AtprVO.net
>>748
そうか、自分が勘違いしてる可能性もある
締切後、解答頼む

750 :132人目の素数さん:2020/08/07(金) 02:43:57 ID:n8IYzqJl.net
2^(1+1/n)/(2^(1/n) - 1) = 1 + (2^(1/n) + 1)/(2^(1/n) - 1) = 2 + 2/(2^(1/n) - 1)

751 :132人目の素数さん:2020/08/09(日) 07:42:58 ID:OYlLFGgh.net
泥臭いやり方以外に解く方法あるのかね?

752 :132人目の素数さん:2020/08/09(日) 08:18:55 ID:zV16c+KM.net
出題2解答
https://i.imgur.com/ngdKGdO.jpg

753 :132人目の素数さん:2020/08/09(日) 08:37:42 ID:OYlLFGgh.net
答えも見つけ方もネット見りゃ分かったけど、これしかアプローチが無いのはなんだかなあ

754 :132人目の素数さん:2020/08/09(日) 08:57:21.62 ID:zV16c+KM.net
>>745はこうか?
https://i.imgur.com/do74soJ.jpg

755 :132人目の素数さん:2020/08/09(日) 09:00:12 ID:DPfZDm/K.net
コレ無理数度で解けた人いる?
μ^(x)=inf{ μ | inf {(p/q - x)q^μ | p/q>x }>0 }
μ_(x)=inf{ μ | inf {(x - p/q)q^μ | p/q<x }>0 }
とでもおいて出題2は
μ^(2/(log(2)) > 2
が示されれば十分なのはすぐわかる
ところでμ(x)=max{μ^(x), μ_(x)}はirrationality measureと呼ばれてコレの上からの評価を作るテクニックや結果は色々見つかるんだけど下からの評価、つまり「この無理数はこんないい有理数近似を持ってる」事を示す方の論文はあんまり見つからない
例えば􏰈􏰖 μ(2/log(2))
≦1-(1+ln(3+sqrt(8)))/(1+ln(3-sqrt(8)))
みたいのは出てくるんだけど反対向きが見つからない

756 :132人目の素数さん:2020/08/09(日) 09:35:29 ID:/Fnf+4AP.net
泥臭いというかなんというか、

自然数kに対して
k<⌈2/(2^{1/n} -1)⌉
⇔ k < 2/(2^{1/n} -1)
(変形中略)
⇔ log(2)/2n < (log(k+2)-log(k))/2

k<⌊2n/log(2)⌋
⇔ k+1 ≦ 2n/log(2)

1/(k+1) < (log(k+2)-log(k))/2 < 1/k
だからどっかで差がつく、でよいのかな

757 :132人目の素数さん:2020/08/09(日) 10:42:16.87 ID:zV16c+KM.net
大雑把にいうとこんな感じ?
2n/log2-2/(2^(1/n)-1)が「2n/log2の小数部分」よりも小さければ等号は成り立たなくなる。
前者は無限遠でフーリエ変換すると1-log2/(6n)+O(1/n^3)、後者は0,1間の擬似乱数みたいなもの。
だから、あるnで等号にならない確率はlog2/(6n)。この確率はnの桁が1増えると大体1/10倍だけど、
その桁に含まれる数も10倍だから、桁ごとに等号が成り立たないnが発見される確率はさほど変わらず、
だいたいlog2/6(1+1/2+...+1/9)=0.3268。もう少しちゃんと計算すると
log2/6 sum(n=10^k〜10^(k+1)-1)1/nのk->∞を計算してlog2log6=0.266で、これ以上下がらない。
1つ桁を増やせば確率0.266で見つかるから、いずれどこかで見つかるよね。

まあ証明問題なら0点だけど、これを手掛かりに反例を1つ見つければ勝ちかな。

758 :132人目の素数さん:2020/08/09(日) 10:43:17.65 ID:zV16c+KM.net
>>757
間違えた
×フーリエ変換
○テイラー展開

759 :132人目の素数さん:2020/08/09(日) 10:58:14.27 ID:zV16c+KM.net
>>757
もう一つ訂正で、確率を足算しているから、見つかる確率ではなくその桁に等号が成り立たないnがいくつあるかの平均だな。

760 :132人目の素数さん:2020/08/09(日) 12:02:44.68 ID:WJnpo5Pt.net
俺の直感ではもともと不成立
なんだか成り立つように見えて、果てしなく遠い数でようやく反例が見つかる
へたな計算機では誤差が生じて反例の確認すらままならないときた
エレ解の問題としては異端児すぎる

761 :132人目の素数さん:2020/08/09(日) 12:37:15.79 ID:Ym6X0bb6.net
>>760
なんだよな
まぁ正則連分数表示を求めるアルゴリズム作って正則連分数近似の中から探すとかにすればややかっこよくなるだけで本質的に計算機で探すしかうまい手が見つからない
irratinality measureがらみなんだけどうえから評価でなくて下から評価であんまり研究が見つからない

762 :132人目の素数さん:2020/08/09(日) 16:27:13.36 ID:wgUVJ5Xl.net
>>752
n = 777451915729368;
2n/log(2) = 2243252046704767 - 0.0331107460500546… /n,
2/(2^(1/n) -1) = 2243252046704766 + 0.08241378404327… /n
 = coth(log(2)/2n) - 1,
辺々引いて
2/(2^(1/n) - 1) + 1 - 2n/log(2)
 = coth(log(2)/2n) - 2n/log(2)
 = (1/3)(log(2)/2n) - (1/45)(log(2)/2n)^3 + (2/945)(log(2)/2n)^5 - … (マクローリン)
 = 0.11552453009332422 /n - …

763 :132人目の素数さん:2020/08/09(日) 17:15:38 ID:wgUVJ5Xl.net
>>954
n = 140894092055857794;
2n/log(2) = 406534415799078269 - 0.09254321204766 /n,
2^(1+1/n)/(2^(1/n) -1) = 406534415799078270 + 0.022981318045663 /n
 = coth(log(2)/2n) + 1,
辺々引いて
2^(1+1/n)/(2^(1/n) -1) -1 - 2n/log(2)
 = coth(log(2)/2n) - 2n/log(2)
 = (1/3)(log(2)/2n) - (1/45)(log(2)/2n)^3 + (2/945)(log(2)/2n)^5 - …
 = 0.11552453009332422 /n - …

764 :132人目の素数さん:2020/08/10(月) 02:52:35 ID:RKSK+UXb.net
ディリクレのディオファントス近似定理
 |qα-p| < 1/N, 0<q≦N
を改良して
 |qα-p| < 0.11552453/q, 0<q≦N
とできればよいのだが、それはかなり難しいようだ。

α = 1/(2^{1/n} -1) はn次方程式 2x^n - (x+1)^n = 0 の根
つまり n次の代数的数。
これについては次の予想があるので、改良は容易でない。

〔予想〕(Serge Lang)
 αが3次以上の代数的数で、k>1 ならば、
  |qα - p| < 1/(q・log(q)^k),
 を満たす有理数 p/q は有限個しか存在しない。

765 :132人目の素数さん:2020/08/10(月) 16:14:45.55 ID:ue6XHQae.net
今月の問題を見てゲンナリしたワタクシ

766 :132人目の素数さん:2020/08/10(月) 17:10:25.60 ID:S/cc1a2M.net
8月号問2から続けて3問似た系統の問題だな。
解き方は全然違うだろうが。

767 :132人目の素数さん:2020/08/10(月) 23:28:21 ID:RKSK+UXb.net
きょう出題1に応募したら859番でした。
今月は出題2でがんばろう。

768 :132人目の素数さん:2020/08/11(火) 00:33:20.69 ID:7+rXuyzb.net
でもこういう奴の方がある意味難しい
問題を解けるかどうかなんて解けて当たり前レベル
いかに“面白く”解けるかが問われる

769 :132人目の素数さん:2020/08/11(火) 19:10:02 ID:u95MOfyC.net
確かに
時間をかければ誰でも示せるけどここではまさにエレガントな解答が求められているよね

770 :132人目の素数さん:2020/08/11(火) 21:46:40 ID:wR6KSB7W.net
敢えて論争を仕掛けて見るテスト

簡単な問題の解法をあれこれ工夫するのが好きか?
方法問わず難しい問題を解ききることに幸せを感じるか?

俺は後者。後者の問題こそ、思いもよらない解法が生まれやすい
「これしか解法ないだろ」と思って解くんだけど、みんな頭振り絞ってるせいか何通りも解法が出てきて仰天することが多々ある

771 :132人目の素数さん:2020/08/11(火) 21:56:17.42 ID:dlrqXygC.net
>>770
本音を言うとちょい物足りん

772 :132人目の素数さん:2020/08/11(火) 23:06:28.99 ID:mJLBy1sq.net
>>771
だよね

773 :132人目の素数さん:2020/08/12(水) 00:01:12.72 ID:qSeYF8jt.net
8月号の出題2、この辺
https://oeis.org/A129935
を見ても一番小さい例外は 777451915729368 みたいなんだけど、正しいのだろうか。
>>757の考え方で行くとその数まで例外が無い確率って1.7%ぐらいなんだけど、
見落としはないのかなあ。

774 :132人目の素数さん:2020/08/15(土) 03:00:41 ID:Bp8epmHH.net
>>773
検証プログラム作成時

1より大きい無理数αに対し正規連分数の打ち切り近似有理数列ri(α)を
r0(α)=[α]、
r(i+1)(α)=[α]+1/ri(1/(α-[α]))
で定める

補題
任意の奇数iに対しri(α)はαの上からの近似であり、分母がri(α)の分母以下である任意の上からの近似の中で最良である

2/(2^(1/n)-1)の天井と2n/(log2)の床が一致しないものが存在するならば、その最小を与えるものはある奇数iについてri(2/log2)の分母として得られる自然数に限られる
主張
奇数iに対し
ri(2/log2)=p/q、
r(i+1)(2/log2)=r/s、
r(i+2)(2/log2)=t/u
とおく
1) qが反例を与えるには(p/q-2/log2)q^2<log2/6が必要である。特に
(p/q-t/u)q^2<log2/6‥?
が必要である
2)qが反例を与えるには(p/q-2/log2)q^2<log2/6-(log2)^3/360/q^2なら十分である。特に
(p/q-r/s+1/q^4)q^2<log2/6‥?
なら十分である
定理
最小の反例はr39(2/log2)の分母である
n= 777451915729368
である
∵) r1(2/log2)=3/1であるがn=1は明らかに反例を与えない
i:3〜37である奇数に対し主張の?を満たすものはない
i=39のとき主張の?は満たされる

775 :132人目の素数さん:2020/08/15(土) 03:01:14 ID:Bp8epmHH.net
import Data.Ratio

-- 一般化連分数から有理数近似列を作成
fracs bs cs = id
$ map (head.fst)
$ iterate (\([[p,q],[p',q']],n) -> ([[(bs!!n)*p+(cs!!n)*p',(bs!!n)*q+(cs!!n)*q'],[p,q]],n+1))
$ ([[bs!!0,1],[1,0]],1)

rats bs cs = id
$ map (\[x,y] -> x%y)
$ fracs bs cs

-- 1/(log2) の一般化連分数の作成
bs = [1..]
cs = 0:(concat [[k^2,k^2] | k<-[1..]])

-- 有理数近似列から正規連分数の作成
makeRbs ratapxs = let
rcfsMain rs = let
apx1 = truncate $ rs !! 0
apx2 = truncate $ rs !! 1
newrs = map (recip.(+(-(fromInteger $ apx1)))) rs
in if apx1 == apx2
then (apx1:(rcfsMain newrs))
else rcfsMain $ tail rs
in rcfsMain ratapxs

-- 1/(log2) の正規連分数の作成
rbs = makeRbs $ drop 2 $ rats bs cs
rcs = repeat 1

-- 1/(log2) の正規連分数から有理数近似列を作成
rrats = map (*2) $ rats rbs rcs

-- 反例を与えるための必要条件 ind1<(log2)/6 と
-- 反例を与えるための十分条件 ind2<(log2)/6 の
-- 検証
tests = [ (k,ind1,ind2,n) |
k<-[1,3..],
let x = rrats !! k,
let y = rrats !! (k+1),
let z = rrats !! (k+2),
let n = denominator x,
let rn = fromInteger $ n,
let ind1 = fromRational $ (x-z)*rn^2,
let ind2 = fromRational $ (x-y+1/rn^4)*rn^2
]

main = do
mapM_ print $ take 20 $ tests

776 :132人目の素数さん:2020/08/15(土) 03:01:34.76 ID:Bp8epmHH.net
(1,0.1111111111111111,1.1428571428571428,1)
(3,0.28125,0.3074276462254604,9)
(5,0.2157303370786517,0.37955514726065,96)
(7,0.5604534005037783,0.778657718283889,445)
(9,0.1838388515860153,0.2907375803451737,794)
(11,1.0174722211315717,1.2252482805588467,4319)
(13,0.1773551664467339,0.19461297624934124,25469)
(15,0.6869730361965083,1.0463959311459805,287209)
(17,0.13674228263475482,0.18821662431664307,418079)
(19,0.7335370174413465,1.1584026182264542,3057423)
(21,1.0170018170613446,1.2245661504073762,8336111)
(23,0.19155865628150245,0.21960325986464296,24590254)
(25,1.4623220085717246,1.5494690343620314,385107953)
(27,0.2731271551989445,0.28278157095101175,6847196937)
(29,0.8922736690121698,1.6110001975243693,200557046245)
(31,1.1787446582422778,1.287114621424387,449541554817)
(33,0.210010870755119,0.23465044264678134,2669611999270)
(35,0.2248455561686892,0.40858063754648183,25423560120208)
(37,0.7271910471480607,0.7842187311800556,113071214541301)
(39,3.29262416686285e-2,3.524736840949706e-2,777451915729368)

777 :132人目の素数さん:2020/08/16(日) 02:41:58 ID:ocOa8mpd.net
>>774
コード間違ってた
1/log2正規連分数の打ち切り近似を2倍しても2/log2の正規連分数の打ち切り近似にはならん
コード直したらn=777451915729368はi=35での上から評価だった

import Data.Ratio

-- 一般化連分数から有理数近似列を作成
fracs bs cs = id
$ map (head.fst)
$ iterate (\([[p,q],[p',q']],n) -> ([[(bs!!n)*p+(cs!!n)*p',(bs!!n)*q+(cs!!n)*q'],[p,q]],n+1))
$ ([[bs!!0,1],[1,0]],1)

rats bs cs = id
$ map (\[x,y] -> x%y)
$ fracs bs cs

-- 2/(log2) の一般化連分数の作成
bs = 2:[2..]
cs = [0,2,1]++((concat [[k^2,k^2] | k<-[2..]]))

-- 有理数近似列から正規連分数の作成
makeRbs ratapxs = let
makeRbsMain rs = let
apx1 = truncate $ rs !! 0
apx2 = truncate $ rs !! 1
newrs = map (recip.(+(-(fromInteger $ apx1)))) rs
in if apx1 == apx2
then (apx1:(makeRbsMain newrs))
else makeRbsMain $ tail rs
in makeRbsMain ratapxs

-- 2/(log2) の正規連分数の作成
rbs = makeRbs $ drop 10 $ rats bs cs
rcs = repeat 1

-- 1/(log2) の正規連分数から有理数近似列を作成
rrats = rats rbs rcs
--rrats = map (*2) $ rats rbs rcs

-- 反例を与えるための必要条件 ind1<(log2)/6 と
-- 反例を与えるための十分条件 ind2<(log2)/6 の
-- 検証
tests = [ (k,ind1,ind2,n) |
k<-[1,3..],
let x = rrats !! k,
let y = rrats !! (k+1),
let z = rrats !! (k+2),
let n = denominator x,
let rn = fromInteger $ n,
let ind1 = fromRational $ (x-z)*rn^2,
let ind2 = fromRational $ (x-y+1/rn^4)*rn^2
]

main = do
mapM_ print $ take 18 $ tests

778 :132人目の素数さん:2020/08/16(日) 02:42:30 ID:ocOa8mpd.net
(1,0.1111111111111111,1.125,1)
(3,0.2571428571428571,0.3584995251661918,9)
(5,0.3645833333333333,0.5745868183338909,35)
(7,0.24181360201511334,0.2751801401822668,96)
(9,0.21822607876241706,0.22524981315418816,794)
(11,0.18275732576857484,0.19461297624934124,25469)
(13,0.15236871728780266,0.1584026182320683,418079)
(15,0.4463335352290383,0.8061415376018488,10975455)
(17,0.18006122736094865,0.21960325986464296,24590254)
(19,0.5385099766993894,0.5494690343620313,136566069)
(21,0.24775175562084492,0.3293483999092869,6847196937)
(23,0.36333931797114377,0.5706953927377529,27637329632)
(25,0.5300370170033072,0.6110001975243692,76064791959)
(27,0.6450755494730714,0.8217786553560967,574033809103)
(29,0.210010870755119,0.23465044264678134,2669611999270)
(31,0.2533250635740805,0.2900654933452463,25423560120208)
(33,0.5163505675436897,0.6960546827950139,200718868962394)
(35,3.29262416686285e-2,3.404729100026536e-2,777451915729368)

779 :132人目の素数さん:2020/08/17(月) 14:49:48.49 ID:U7f6nYy/.net
>>752 と一致したが・・・・
エレガントとは思えませぬ。。。

780 :132人目の素数さん:2020/08/17(月) 15:11:38.97 ID:A0lhg88c.net
>>779
存在するか?
なんだからそりゃ反例一個あげれば終わりでしょ?
>>774-778>>773へのレスだよ
ホントにn<777451915729368までに解が存在しないか一個ずつ計算したらいかに計算機でも辛い
連分数のテクニック使わないとn=777451915729368が最小の反例は計算機使っても苦しいと思う

781 :132人目の素数さん:2020/08/18(火) 07:54:22.81 ID:3xQcw/Vm.net
最良性が必要ってところが良くわからん

782 :132人目の素数さん:2020/08/20(木) 03:00:33.43 ID:hteSSTrX.net
きょう出題2に応募したら869番でした。
今月は出題3でがんばろう(?)

783 :132人目の素数さん:2020/08/22(土) 00:19:25.54 ID:PIye8TW8.net
そうなんだ

784 :132人目の素数さん:2020/08/26(水) 23:33:21.11 ID:E7gw9mcx.net
>>636
2020年3月号出題2
(別解)
n=7L かつ 成分の中に1〜7がL個ずつ現れるもの全体をAとする。
スターリングの公式から
 #A = (7L)!/(L!)^7 〜 k・(7^n)・n^(-3)・exp(-4/n),
 k = (√7)/(7/2π)^3 = 3.658504
ここで 0<c<7 とすると、じゅうぶん大きいnについて
 #A > c^n,
∴ c<7 で成立。

785 :132人目の素数さん:2020/08/26(水) 23:43:42.05 ID:E7gw9mcx.net
>>607 >>613-614
2020年4月号出題2

〔定理2〕
Sがn点のとき、これ以上線が曳けなくなるまでに
 min{ [2n/5], [3(n-1)/7] } 本の線が必要。
〔系〕
n≠1,3,8 の場合は [2n/5] 本の線が必要。

786 :132人目の素数さん:2020/08/27(木) 00:25:19.89 ID:yuNusFyR.net
>>622
2020年5月号出題2

〔補題〕
Xは問題の条件を満たす。{x_k}は単調増加としてよい。
 x_1<・・・・<x_k<・・・・<x_n,
このとき k=1,2,・・・・,n に対して
 x_1 + x_2 + ・・・・ + x_k ≧ 2^k - 1,
(略証)
 {x_1,x_2,・・・・,x_k} の部分集合は(空集合も含め)2^k 個ある。
要素の和は0以上の整数で、題意により互いに相異なる。
∴ 最大のものは上記のとおり。  (終)

 y_0 = 0
 y_k = Σ[j=1,k] (x_j - 2^{j-1}) = Σ[j=1,k] x_j - 2^k + 1,
とおけば、補題により
 y_k ≧ 0,      (*)
これを使って
(2 - 2^{1-n}) - Σ[k=1,n] 1/x_k = Σ[k=1,n] (2^{1-k} - 1/x_k)
 = Σ[k=1,n] (x_k - 2^{k-1})/(2^{k-1}・x_k)
 = Σ[k=1,n] (y_k - y_{k-1})/(2^{k-1}・x_k)
 = Σ[k=1,n-1] y_k・(2_{k+1} - x_k)/(2^k・x_k・x_{k+1}) + y_n/(2^{n-1}・x_n)
 ≧ 0,

(*) x_j - 2^{j-1} ≧ 0 は不成立。
  反例 n=4, X = {3,5,6,7}

787 :132人目の素数さん:2020/08/27(木) 00:52:54.35 ID:yuNusFyR.net
>>653-656
2020年6月号出題1

〔命題〕
nから始めて操作(1)(2)で1に辿り着かない条件は次のいずれか。
@ 3の倍数
A すべての桁が奇数
B 順不同で {1,1,1,4} または {1,1,1,8}
C A,Bを満たす数を、繰上りなしに何回か2倍して得られるもの

これらが1に辿り着かないことの確認は難しくない。
しかし逆は、次の補題を使っても難しい・・・・

〔補題〕
 @〜Cを満たさないnを、うまく桁を入れ替えてから2で割れば
 Aを満たさないようにできる。

788 :132人目の素数さん:2020/08/29(土) 02:08:57 ID:nEvr3uHf.net
>>785 訂正

×  [ … ]  これは floor(x)
○  ceiling(…)

789 :132人目の素数さん:2020/09/06(日) 15:52:46.57 ID:GtZ5Bfgy.net
>>787
工夫と根気を試されてたな

790 :132人目の素数さん:2020/09/06(日) 15:59:37.41 ID:GtZ5Bfgy.net
出題2の3だけど、自然な構成では本質的に平方数が顔を出す。如何に不自然にするかがポイントらしいが、何のためにこんなことしなきゃならんのか分からなくなってきて落ち込んでる

791 :132人目の素数さん:2020/09/06(日) 16:03:12.56 ID:GtZ5Bfgy.net
エレガントな問題を来月に期待しよう
時弘先生や岡本先生の問題がやってくる季節

792 :132人目の素数さん:2020/09/09(水) 23:07:45.05 ID:IR7822fG.net
それは楽しみだ

793 :132人目の素数さん:2020/09/10(木) 07:14:35.69 ID:xPQxlsn0.net
>>791
時弘先生と岩沢宏和だったな。
岩沢の確率本には、他の本が書かないようなことが載っていて、個人的にはかなり面白い。

794 :132人目の素数さん:2020/09/10(木) 23:59:35 ID:7NHP8bMP.net
>>782
きょう出題1に応募したら941番でした。
今月は出題2でがんばろう。

795 :132人目の素数さん:2020/09/11(金) 02:21:07.75 ID:KOZpx0Su.net
2020年9月号

■出題1
やり方は色々ありそうですが、ストレートに計算してもできる。
bの方が平方根の形だから、aaを計算する。
 p = (7√11 + 3√3)^{1/3},
 q = (7√11 - 3√3)^{1/3},
とおくと pq = 8 で,
 p と qq, q と pp が同類項にまとまるのがミソ。

aa = {7√11 + 8(p+q)}^2
 = 49・11 + 16(7√11)(p+q) + 64(p+q)^2
 = 16{(7√11)p+4qq} + 16{(7√11)q+4pp} + 49・11 + 128pq
 = 16p{7√11 + (1/2)q^3} +16q{7√11 + (1/2)p^3} + 539 + 1024
 = 24p(7√11 - √3) + 24q(7√11 + √3) + 1563
あとは3乗を計算する。
 = bb,
これと a>0, b>0 から a=b を示す。

出題:渋川元樹(略して シブキ?)

796 :132人目の素数さん:2020/09/11(金) 02:39:58.44 ID:KOZpx0Su.net
2020年9月号

■出題2 (1)
 A = √(N+1) + √(N - 1/2) + √(N - 1/2),
 B = √(N-1) + √(N + 1/2) + √(N + 1/2),
のとき
 3√N > A > B
を示す問題。(N = 333^2)

左側は (二乗平均) > (相加平均) で簡単。

右側は
 A - B = {√(N+1) - √(N-1)} - 2{√(N+1/2) - √(N-1/2)}
    = 2/{√(N+1) + √(N-1)} -2/{√(N+1/2) + √(N-1/2)}
    > 0,

∵ √(N+1/2) + √(N-1/2) > √(N+1) + √(N-1),
(略証)
 √(N+x) はxについて上に凸だから
  √(N+1/2) > (3/4)√(N+1) + (1/4)√(N-1),
  √(N-1/2) > (1/4)√(N+1) + (3/4)√(N+1),
  辺々たす。
または
 {√(N+1/2) + √(N-1/2)}^2 - {√(N+1) + √(N-1)}^2
 = 2{N + √(NN -1/4)} - 2{N + √(NN-1)}
 = 2{√(NN -1/4) - √(NN-1)} > 0,

あるいは
 g(x) = √(N+x) とおき
 A - B = {g(1) + 2g(-1/2)} - {2g(1/2) + g(-1)}
  = (1/4) g '''(r)        (平均値の定理)
  = (3/32)(N+r)^{-5/2}
  > 0,

797 :132人目の素数さん:2020/09/11(金) 03:01:43.50 ID:KOZpx0Su.net
2020年9月号

■出題2
(2) √2 + √z ≒ y
となる自然数 y, z を見つける問題。

 xx - 2yy = -1 ならば
 (xx +5 -4x)/2 = yy + 2 - (2√2)y - 2(x-y√2)
 = (y-√2)^2 + 2/(x+y√2),
∴ √2 + √{(xx +5 -4x)/2} = y + 1/{(x+y√2)(y-√2)} + … ≒ y,

 xx - 2yy = 1 ならば
 (xx +3 -4x)/2 = yy + 2 - (2√2)y - 2(x-y√2)
 = (y-√2)^2 - 2/(x+y√2),
∴ √2 + √{(xx +3 -4x)/2} = y - 1/{(x+y√2)(y-√2)} + … ≒ y,

なお「ペル方程式」
 xx - 2yy = (-1)^n
の解は
 x = {(1+√2)^n + (1-√2)^n}/2,
 y = {(1+√2)^n - (1-√2)^n}/(2√2),
で与えられる。

(3)
 工夫すれば色々できそう。

出題:阿賀岡

798 :132人目の素数さん:2020/09/12(土) 18:16:36.22 ID:n7twx+Wx.net
〔出題2〕
集団の人数やその友人関係は任意で、あらゆる可能性がある。
集団の人数とその友人関係、f(n) が一つ与えられたとき、
それに応じて
 「条件」をみたすように役員(定員なし)を選出する
ことが可能か?
ってことだろうな…

799 :132人目の素数さん:2020/09/13(日) 12:21:17.16 ID:2Sn4Bcu0.net
>>798
そうとしか読めんだろ。
役員0人は許されるのか?とは思ったが。

800 :132人目の素数さん:2020/09/19(土) 15:30:15.99 ID:70ypyh/K.net
fの定義域が正の整数だから役員0人はだめってことなんだろうな

801 :132人目の素数さん:2020/10/06(火) 12:24:18.39 ID:bkQwQMCs.net
むずかしい

802 :132人目の素数さん:2020/10/11(日) 15:37:14.38 ID:FIq2/+sl.net
出題2完答できた人いる?
出題1は時弘先生の割には簡単だったか?

803 :132人目の素数さん:2020/10/12(月) 20:50:21.15 ID:aUylGcxN.net
2020年10月号

■出題1
 たしかに易しめだが…
漸化式
 x ' = x(1-y) + yz,
と 0<y<1 から
 min{x,z} ≦ x ' ≦ Max{x,z}
はすぐ出る。(埼玉大学?)

この後のやり方は色々ありそうですが
線形問題に直すのもあり(?)
漸化式は
 ↑x ' = (1-b)↑x + b↑z,
 ↑y ' = (1-c)↑y + c↑x,
 ↑z ' = (1-a)↑z + a↑y,
↑x/a = ↑y/b = ↑z/c ならば、重心は変わらない。
 1-r ≦ {a,1-a,b,1-b,c,1-c} ≦ r,
なるrをとれば △XYZ は縮小率 r 以下で縮小する。
n→∞ で X。Y。Z。 の重心に収束する。
 出題:時弘哲治

804 :132人目の素数さん:2020/10/13(火) 21:13:34.02 ID:KZSrLTuA.net
>>803
スマートですな
絶対値の極限を考える方法もある
簡単過ぎもせず、エレガントな解答を競うにはうってつけの良問か

さて出題2
(1)はいいとして(2)は難しい
(3)に届かなかった人が多いと予想

805 :132人目の素数さん:2020/11/07(土) 20:23:16.93 ID:y1F8VxDV.net
締切ホルモンでラストスパート

806 :132人目の素数さん:2020/11/09(月) 15:13:17.19 ID:LkWWlvO9.net
反転幾何使うと何でもエレガントになる説

807 :132人目の素数さん:2020/11/11(水) 01:21:41.21 ID:rE2Lzr4n.net
2020年11月号

■出題1
 僊BC が △A。B。C。よりずっと大きい場合、
それをさらに相似拡大すると、不成立?
相似縮小も含むと解すべきか?

(補正案)
僊BC を正三角形でない任意の三角形とする。
このとき、平面Π上の正三角形△A。B。C。と 点P の組で、以下の条件
「 PA。, PB。, PC。の長さが辺BC, CA, AB の長さと順に等しい。」
をみたすものがただ二組(点PがΓの内側、点PがΓの外側、一組ずつ)存在する。

■出題2
 (i) 円周角の定理(とその逆)から出るらしい。
   P_ij 以外の交点(Q_ij)が の内部か外部か周上かで場合分け。
 (ii) お手上げ(反転幾何とか使うのかな?)

808 :132人目の素数さん:2020/11/11(水) 02:10:51.15 ID:rE2Lzr4n.net
11月号 出題1
 BC=a, CA=b, AB=c, 僊BC=S とおく。

・点PがΓの内側にある組
 2辺が b,c 挟角が∠A + 60°の三角形を描く。
 第二余弦定理より、対辺は L = √{(aa+bb+cc + (4√3)S)/2},
 同様の3つの凾フ頂角の和は ∠A + ∠B + ∠C + 180°= 360°
∴ これら3つの凾組合せると、一辺 L の△になる。

・点PがΓの外側にある組
 2辺が b,c 挟角が |60° - ∠A| の三角形を描く。
 第二余弦定理により、対辺は L' = √{(aa+bb+cc - (4√3)S)/2} > 0,
 同様にして、2つの凾フ頂角の和が、他の凾フ頂角。
∴ 上記2つの凾ゥら他の凾引くと、一辺 L' の△が残る。

唯一性はどう示すか・・・

809 :132人目の素数さん:2020/11/11(水) 02:40:01.79 ID:rE2Lzr4n.net
僊BCの各内角が120°より小さいとき、
フェルマー点をFとすれば
 L = AF + BF + CF = min{AQ+BQ+CQ | Q∈Π}
らしい。

文献
1.デボーヴ(M.Desboves)『平面幾何學 研究法』増補6版, 冨山房 (1929)
     吉田好九郎 譯 p.24
2.アメリカの数学オリンピック(USAMO 1974 [5])

3.数セミ増刊「数学の問題」第2集, 日本評論社 (1978)
 ●110 解説 図2

810 :132人目の素数さん:2020/11/12(木) 01:42:52.40 ID:UMkT78Ht.net
所与の3辺の比に対しPは正三角形の2辺で決まる2つのアポロニウス円の交点だから高々2点

811 :132人目の素数さん:2020/11/13(金) 17:51:16.89 ID:M5JR9HFw.net
チョト一般化してみた。

〔類題〕
僊BC を正三角形でない任意の三角形とする。
このとき、平面Π上の僊。B。C。と点Pの組で、以下の条件
「∠A。, ∠B。, ∠C。は所与の値。
 PA。, PB。, PC。の長さが辺BC, CA, AB の長さと順に等しい。」
をみたすものが二組存在する。

812 :132人目の素数さん:2020/11/13(金) 18:33:37.80 ID:M5JR9HFw.net
辺長が a'= BC sin(A。), b'= CA sin(B。), c'= AB sin(C。) である三角形の
頂角を α,β,γ 面積を S' = S(a',b',c') とする。

・点PがΓの内側にある組
 2辺が AB, BC 挟角が β + ∠B。の三角形を描く。
 第二余弦定理などより、対辺は 2R sin(B。)
ここに
 (2R)^2 = {sin(A。)cos(A。)BC^2 + sin(B。)cos(B。)CA^2 + sin(C。)cos(C。)AB^2 + 4S'}/{sin(A。)sin(B。)sin(C。)},
 同様の3つの凾フ頂角の和は 180°+ 180°= 360°
∴ 点Pの周りにこれらの凾並べれば、題意をみたす僊。B。C。になる。

・点PがΓの外側にある組
 2辺が AB, BC 挟角が |β - ∠B。| の三角形を描く。
 第二余弦定理などより、対辺は 2R* sin(B。),
ここに
 (2R*)^2 = {sin(A。)cos(A。)BC^2 + sin(B。)cos(B。)CA^2 + sin(C。)cos(C。)AB^2−4S'}/{sin(A。)sin(B。)sin(C。)},

 同様にして、2つの凾フ頂角の和が、他の凾フ頂角。
 点Pの周りにこれらの凾重ねれば、題意をみたす僊。B。C。が残る。

[面白スレ33.528-530]

813 :132人目の素数さん:2020/11/14(土) 03:23:42.84 ID:MWjdA7m9.net
出題文に「三次元への拡張など」とあるけど、どうするのかな?

(正4面体ではない) 任意の4面体ABCDから4つの距離a,b,c,dを取出す。(*)
このとき、正四面体 A。B。C。D。 と 点P の組で、以下の条件
「PA。, PB。, PC。, PD。 が a,b,c,d に順に等しい。」
をみたすようなものが何組ある・・・・とか

*) たとえば、頂点Aの対面BCDの周長をaとする(??)

814 :132人目の素数さん:2020/11/14(土) 11:31:59.47 ID:GBoMTaBy.net
>>813
それもアポロニウスの3円(球)を考えれば良さそう

815 :132人目の素数さん:2020/11/14(土) 21:30:06.74 ID:MWjdA7m9.net
>>812
面積で表わすなら
僊。B。C。= (1/2)(2R)^2 sin(A。)sin(B。)sin(C。)
 = (1/2){sin(A。)cos(A。)BC^2 + sin(B)cos(B。)CA^2 + sin(C。)cos(C。)AB^2 ± 4S'},
 S' = S(a' ,b', c')
かな

816 :132人目の素数さん:2020/11/19(木) 20:35:31.41 ID:Clp5hM1J.net
>>811
辺長が B'C'= BC・sin(A。), C'A'= CA・sin(B。), A'B'= AB・sin(C。) である三角形の
頂角を ∠A'=α, ∠B'=β, ∠C'=γ とする。

・点PがΓの内側にある組
 辺C'A' の外側に、内角 A。,B。,C。の三角形を貼り付け、C'A'D' とする。
 ∠D' = B。
 A'D' = CA・sin(C。), B'A' = AB・sin(C。), ∠B'A'D' = α+A。,
 B'C' = BC・sin(A。), C'D' = CA・sin(A。), ∠B'C'D' = γ+C。,

二辺が CA, AB で挟角が α+A。の三角形の対辺は x = B'D'/sin(C。),
二辺が AB, BC で挟角が β+B。の三角形の対辺は y,
二辺が BC, CA で挟角が γ+C。の三角形の対辺は z = B'D'/sin(A。),
 同様にして x/sin(A。) = y/sin(B。) = z/sin(C。) (= 2R)
 これら3つの凾フ頂角の和は 180°+ 180°= 360°
 点Pの周りにこれらの凾並べてできる、辺長 x,y,z の凾ヘ題意をみたす。

・点PがΓの外側にある組
 辺C'A' の内側に 内角 A。,B。,C。の三角形を貼り付け、C'A'E' とする。
 ∠E' = B。
 A'E' = CA・sin(C。), B'A' = AB・sin(C。), ∠B'A'E' = |α-A。|,
 B'C' = BC・sin(A。), C'E' = CA・sin(A。), ∠B'C'E' = |γ-C。|,

二辺が CA, AB で挟角が |α-A。| の三角形の対辺は x = B'E'/sin(C。),
二辺が AB, BC で挟角が |β-B。| の三角形の対辺は y,
二辺が BC, CA で挟角が |γ-C。| の三角形の対辺は z = B'E'/sin(A。),
 同様にして x/sin(A。) = y/sin(B。) = z/sin(C。) (= 2R*)
 2つの凾フ頂角の和が、他の凾フ頂角。
 点Pの周りにこれらの凾重ねて残る、辺長 x,y,z の凾ヘ題意をみたす。

817 :132人目の素数さん:2020/12/07(月) 21:12:51.62 ID:iTXaP4Tf.net
今更だけど10月号出題2の小問2と3解けた人いる?

818 :132人目の素数さん:2020/12/12(土) 22:33:04.01 ID:YkJVf6v7.net
思いもよらんかった

819 :132人目の素数さん:2021/01/08(金) 22:36:34.17 ID:1n03QYx6.net
必ず相異なる点に収束するってところがみたせねえ

820 :132人目の素数さん:2021/01/09(土) 09:57:05.36 ID:CHZnYHqh.net
締切過ぎた
今月はパズル感が強めだったな

821 :132人目の素数さん:2021/01/09(土) 20:47:31.03 ID:Gni2ACgE.net
「A。, B。, C。が平面上の相異なる3点としたとき・・・・」

だから ↑AB, ↑BC, ↑CA を不変にすれば

「点列 {A_n}, {B_n}, {C_n} は相異なる3点に収束する」んぢゃね?

一方で 「自明な漸化式を含まない」 とあるから、

ABCの重心は別の点(例えば原点)に近付く希ガス

822 :132人目の素数さん:2021/01/09(土) 21:04:02.72 ID:Gni2ACgE.net
・・・・というのは問2の僖EFの話でした。スマソ

問1の方は ↑AB を一定しておいて、

 C_{n+1} = 2B_n - A_n

とかやるのかな

823 :132人目の素数さん:2021/01/10(日) 17:06:28.31 ID:7Kh+SWdH.net
問2は三角形じゃなきゃだめかね?
全点x_i=0, y_i=i の相異なる6点から開始したらどうしようもなかろ?

824 :132人目の素数さん:2021/01/10(日) 20:19:56.31 ID:k4Y9uhcW.net
「下図を得るような漸化式を…」と言ってて、その図では凾ネんだが…

6点が共線の場合も含む、と広く解釈するか
D。, E。, F。は凾ニする、と限定するか

825 :132人目の素数さん:2021/01/10(日) 22:05:16.01 ID:7Kh+SWdH.net
一般には三角形になるんだけど、初期値によっては三角形にならないMがある。どんな初期値でも三角形になるか?823の例だと流石に無理だよね、って話

826 :132人目の素数さん:2021/01/13(水) 22:27:07.20 ID:G++RphGz.net
6点が共線の場合も含むと解釈すると、それを悪用していつでも共線化してしまう点列が問1と同様に作れてしまう
さすがにそれはなあと思う

827 :132人目の素数さん:2021/01/15(金) 00:41:35.87 ID:GhIdS4ix.net
>>826
問題を曖昧にして解答者の間口を拡げたとは考えられないか?考えられないかそうだよな
でも823は三角形にできんでしょ?こんな例はつまらんけど

828 :132人目の素数さん:2021/01/15(金) 00:49:03.78 ID:GhIdS4ix.net
ケーキカットは単純なロジックパズルで簡単だった
罠でもあったかな

829 :132人目の素数さん:2021/01/17(日) 05:08:41.58 ID:JaxqZKuI.net
2月号 出題2
 「ピタゴラス三角形」だと仰せなんだから
 上図の3行目は b=104, 4行目は c=157 と解するんだろうね。

(御老公はエクセルを使われないらしい…)

830 :132人目の素数さん:2021/01/17(日) 14:17:08.89 ID:xZfxvcfs.net
本当だ。間違ってるね。

831 :132人目の素数さん:2021/01/19(火) 00:52:17.75 ID:GWoFAzBM.net
「上の例のような8点の組」が (;(a,b)を交換した点も含む)必要は無い
に一票…

832 :132人目の素数さん:2021/01/20(水) 07:42:24.82 ID:joLMXWen.net
(a,b)交換しないなら a≒b もペル方程式も関係ないな…

833 :132人目の素数さん:2021/01/21(木) 07:42:11.57 ID:XywhSHYS.net
「ピタゴラス三角形の一般的表現」て複素数の2乗みたいな式?

「レオナルド・ピサノの恒等式」とかいう

834 :132人目の素数さん:2021/01/25(月) 05:25:10.22 ID:5gaa8JLe.net
今月号の出題1は、10進数に限らず、一般化できそうだな。

835 :132人目の素数さん:2021/01/31(日) 23:17:48.78 ID:9uj5p/5F.net
出題2の円の中心は原点以外だよな?

836 :132人目の素数さん:2021/02/01(月) 13:56:17.53 ID:jjXu+Br4.net
う〜む
出題文には「同一円周上に・・・・」とあるだけで、中心の位置には言及してないが。

たとえば
 (a,b) = (63,16) (60,25) (56,33) (52,39) (39,52) (33,56) (25,60) (16,63)
の8点は 原点を中心とする円周 (半径 c=65) 上にあるから
「正確に同一円周上には乗っていません」という条件に反してて
ダメだろうな....orz

837 :132人目の素数さん:2021/02/01(月) 17:37:53.67 ID:m8cw8mpJ.net
「上の例のような」という条件に反してるんじゃない?

838 :132人目の素数さん:2021/02/01(月) 22:38:49.19 ID:lwI9Z+xK.net
原点でいいなら簡単過ぎるし、2つの図は明らかに原点中心じゃないからな

839 :132人目の素数さん:2021/02/06(土) 18:38:21.05 ID:180aEO+d.net
そして解けない
もう原点中心でいいかなって

840 :出題1:2021/02/09(火) 04:42:51.84 ID:5f0PQTaM.net
一般化して、h進法で考える。
φ(t)をオイラーのトーティエント関数、
tが素数pのs乗で割り切れ、かつs+1乗で割り切れないとき
s=v(p:t)と書くことにする。
kはn^mをh進数で表記したときの桁数である。
log(x)は底がhの対数である。

任意の正の整数mに対して、
n^M≡n^m (mod h^k) M>mをみたすMが存在するための、n,hの条件を考える。

hを以下のように書く
h=(nの素因数からなる積)・(nの素因数以外の素数からなる積)。
ここで、(nの素因数からなる積)=a、(nの素因数以外の素数からなる積)=bと置く。
Nを正の整数、とすると、
フェルマー・オイラーの定理より、n^{m+φ(b^k)・N}≡n^m (mod b^k)
がいえる。

a=1のとき
M=m+φ(b^k)・Nとおくと、n^M≡n^m (mod h^k)がいえる。

以降、a>1とする。

aの任意の素因数pに対して、v(p:n^m)≧v(p:a^k)となるとき
明らかに、n^{m+φ(b^k)・N}≡n^m≡0 (mod a^k)
がいえる。
よって、M=m+φ(b^k)・Nとおくと、n^M≡n^m (mod h^k)がいえる。

aのある素因数p'に対して、v(p':n^m)<v(p':a^k)となるとき
v(p':n^M),v(p':a^k)>v(p':n^m)より、n^Mとn^mは、mod a^kで等しくない。
(ただし、Mはmより大きな整数とする。)
したがって、n^M≡n^m (mod h^k)とはいえない。

841 :出題1:2021/02/09(火) 04:50:47.18 ID:5f0PQTaM.net
以上を踏まえて、以下のようになる。

n<hのとき
aの任意の素因数pに対して、v(p:n)≧v(p:a)となるとき
mlog(n)=log(n^m)≧log(h^{k-1})=(k-1)log(h)より
m>(k-1){log(h)/log(n)}≧k-1よりm≧k-1+1=kがいえるから、
aの任意の素因数pに対して、
v(p:n^m)=m・v(p:n)≧k・v(p:a)=v(p:a^k)がいえるから、
>>840より、任意の正の整数mに対して、n^M≡n^m (mod h^k)をみたす
整数M>mの存在ががいえる。

aのある素因数p'に対して、v(p':n)<v(p':a)となるとき
m=1のとき、任意の2以上の整数Mに対して、n^Mとnはmod h^kで合同にならない。
したがって、n^M≡n^m (mod h^k)とはいえない。

n≧hのとき
aのある素因数p'に対して、v(p':n)≦v(p':a)となるとき
klog(h)=log(h^k)>log(n^m)=mlog(n)より
k>m{log(n)/log(h)}≧mよりk>mがいえるから、
aのある素因数p'に対して、
v(p':a^k)=k・v(p':a)>m・v(p':n)=v(p':n^m)がいえるから、
>>840より、n^M≡n^m (mod h^k)とはいえない。

842 :出題1:2021/02/09(火) 06:47:02.97 ID:5f0PQTaM.net
n≧h、a>1かつ、aの任意の素因数pに対して、v(p:n)>v(p:a)となるとき
この場合は、上記のような一般論を展開するのは難しいです。

ただ、hがsquare freeの場合以下のように議論できます。

h^(s-1)≦n<h^sをみたす整数sをとる。
aの任意の素因数pに対して、v(p:n)≧sとなるとき
h^(k-1)≦n^m<h^(sm)よりk=k-1+1≦sm
がいえるから、
v(p:n^m)=m・v(p:n)≧sm、v(p:h^k)=kv(p:h)=kより
aの任意の素因数pに対して、v(p:h^k)≧v(p:h^k)がいえるので
>>840より、任意の正の整数mに対して、n^M≡n^m (mod h^k)をみたす
整数M>mの存在ががいえる。

aのある素因数p'に対して、v(p':n)<sとなるとき
m=1のとき、任意の2以上の整数Mに対して、n^Mとnはmod h^sで合同にならない。
したがって、n^M≡n^m (mod h^k)とはいえない。

843 :132人目の素数さん:2021/02/09(火) 23:59:34.65 ID:gjcCpXes.net
出題2はまぁヒントというか縛りというかの意味がわかれば一瞬やな
1個目の例
中心 119+120i = (12+5i)^2
4点 153+104i = (13+4i)^2
   133+156i = (13+6i)^2
   85+132i = (11+6i)^2
   105+88i = (11+4i)^2
2個目の例
中心 240+238i = (17+7i)^2
4点 225+272i = (17+8i)^2
   253+204i = (17+6i)^2
   207+244i = (16+7i)^2
   275+252i = (18+7i)^2

どちらも中心α^2で4点が(α+ρ)^2, (α+ρi)^2, (α-ρ)^2, (α-ρi)^2, の形
このタイプで例1はρ=1+i、例2はρ=1
ρとして1+2iとか使っていいならρ=2+iも使えば簡単に8点作れるけど、多分禁止なんでしょう
となると例のようにy=xについて取り替えるしかない
それが許されるのは中心α^2が実部=虚部に近くないとダメ
一例目ではy=x+1, 二例目ではy=x-2
つまりα=x+yiとおいてα^2=x^2-y^2+2xyiが実部=虚部+k上にあるのは(x^2-y^2)-2xy = kの時だけどコレはペル方程式の形してるから基本解×基本単数^(2n)の形の解を無限に持つ

844 :132人目の素数さん:2021/02/10(水) 01:22:49.80 ID:V7Ph0vhz.net
なるほど。4点ずつ同一円周に近づけて x⇔y ですか。
でも 円の中心は |x-y| ≧1 ですから、
2つの円の中心は √2 以上離れてますよね。(傾き-1)
そうすると 円周もそのぐらい離れるような・・・・

4点ずつが同一円周にかなり近い、ということが却って、
8点が同一円周に近づくのを難しくする希ガス。

>>833
そのとおり。

845 :132人目の素数さん:2021/02/10(水) 07:39:28.87 ID:mdwU9sHK.net
>>844
まぁとは言え例示されてる例がその方法で作ってる8点で「このようにして作れ」なんだからしょうがないと思いますけどねぇ?
もちろんそれ以外の方法で実際に「同一円周上にない8点と中心“もどき”で“半径もどき”の差が0に近づく例」を見つけられればいいですけど
多分一松先生の気持ちとしてはy=x±constに件の格子点が無限にある事とかも気づいて下さいねっぽいのに、それをガン無視するのもどうなんだろうという気もする

846 :132人目の素数さん:2021/02/10(水) 15:25:12.43 ID:V7Ph0vhz.net
その方法で「8点の組が、いくらでも正確に同一円周上の点列に近づく」例が見つかるのかなぁ。

847 :132人目の素数さん:2021/02/11(木) 20:46:23.10 ID:2AoFT+Yp.net
P((α+ρ)^2)    |α| >> |ρ|
とし、中心を
Q((1+ε)α^2),  ε>0 (あとで決める)
とすると
|PQ|^2 = |(α+ρ)^2 - (1+ε)α^2|^2
 = |2αρ + (ρ^2 - εα^2)|^2
 = |2αρ|^2 + 2(|ρ|^2 - ε|α|^2)(αρ'+α'ρ) + |ρ^2 - εα^2|^2
ここで ε = |ρ/α|^2 とおくと中項が消えて
 = |2αρ|^2 + |ρ^2 - εα^2|^2
 ≦ |2αρ|^2 + (|ρ|^2 + ε|α|^2)^2
 = |2αρ|^2 + 4|ρ|^4,

∴ |2αρ| ≦ |PQ| ≦ |2αρ| + |ρ^3/α|,

|ρ| を固定して |α| を大きくすれば、
いくらでも正確に同一円周上の点列に近づく。

848 :132人目の素数さん:2021/02/12(金) 00:12:11.80 ID:kq9lG1q/.net
OPが最大になるのは ρ/α>0 のとき
 P: (1+√ε)^2 α^2,   ・・・・ 遠日点
OPが最小になるのは ρ/α<0 のとき
 P:  (1-√ε)^2 α^2   ・・・・ 近日点
その中点
 Q: (1+ε) α^2.

849 :132人目の素数さん:2021/02/13(土) 01:06:26.00 ID:rcfUzmW5.net
その2点の距離を直径2Rとすると
 R = (2√ε)|α|^2 = |2αρ|
∴ その2点は近地点でござる。

850 :132人目の素数さん:2021/02/13(土) 23:06:41.87 ID:rcfUzmW5.net
αとρの偏角の差δを使えば…
 δ = arg(ρ/α),
 ρ = α (√ε) e^{iδ},
と表わせるから
 P (α+ρ)^2 = {α (1 + (√ε) e^{iδ})}^2,
 Q (1+ε)α^2,

|PQ|^2 = |2αρ|^2 + |ρ^2 - εα^2|^2
  = |2αρ|^2 + |(εα^2) (e^{2iδ} - 1)|^2
  = |2αρ|^2 + |ρ|^4・| e^{iδ} - e^{-iδ} |^2
  = |2αρ|^2 + |ρ|^4・| 2i sinδ |^2
  = |2αρ|^2 + |ρ|^4・4 (sinδ)^2,

|2αρ| ≦ PQ ≦ |2αρ| + (|ρ|^3/|α|) (sinδ)^2,

851 :132人目の素数さん:2021/02/15(月) 03:06:49.52 ID:fbJrP/KA.net
> ρとして 1+2i とか使っていいなら ρ=2+i も使えば簡単に8点作れるけど、

他にもあるようです。
 ρ = ±33±4i, ±32±9i, ±31±12i, ±24±23i, ±23±24i, ±12±31i, ±9±32i, ±4±33i,
の32点 (|ρ|=√1105)
 ρ = ±65, ±63±16i, ±60±25i, ±56±33i, ±52±39i, ±39±52i, ±33±56i, ±25±60i, ±16±63i, ±65i,
の36点 (|ρ|=65) など

…と言っているうちにもう15日
3月号の問題に取り組まねば

852 :132人目の素数さん:2021/03/01(月) 21:40:46.59 ID:USy5nfjJ.net
出題2のほにゃほにゃをきちんと示せるかがポイントだな

853 :132人目の素数さん:2021/03/02(火) 18:47:19.00 ID:9tcN6NOO.net
出題1の(2)だが、実際に9個八面体を造る奴が、かなりいると思う。

854 :132人目の素数さん:2021/03/02(火) 18:50:21.28 ID:9tcN6NOO.net
八面体9個じゃなくて、八面体6個と十二面体3個だったw

855 :132人目の素数さん:2021/03/02(火) 19:30:46.38 ID:utk96U0n.net
cadで描こうとした某

856 :132人目の素数さん:2021/03/02(火) 20:20:38.05 ID:9xNo49zP.net
1の(2)は「(1)より明らか」じゃ駄目なのか?

857 :132人目の素数さん:2021/03/02(火) 23:35:42.98 ID:efiNz9aS.net
だれか折り紙で立体作る方法考えてくれ

858 :132人目の素数さん:2021/03/03(水) 02:31:01.10 ID:BYgBOF1p.net
>>857
ユニット折り紙でもマスターしたら?

859 :132人目の素数さん:2021/03/03(水) 12:03:20.13 ID:D59DkOi0.net
>>857
その方が問題解くより難しい

860 :132人目の素数さん:2021/03/04(木) 03:20:16.42 ID:cVC4XyuV.net
蛇足だけど。
「八面体」と、十二面体の半分である「正六角錐」を敷き詰めて
厚さ (√3)/2 の板を充填できる。 2個:1個
同じことだが
この八面体を小さい△で貼り合わせた「鼓形」と「十二面体」を
敷き詰めて 厚さ√3 の板を充填できる。
いわゆる 空間充填多面体。
(2) 一辺の長さが2の立方体にキレイに収まる物だけ残せば…

861 :132人目の素数さん:2021/03/07(日) 10:01:30.84 ID:gGJ1BgVW.net
出題2を正確に論じ切るの難しいんだが

862 :132人目の素数さん:2021/03/07(日) 10:36:23.10 ID:SHnRXOIk.net
>>861
バカめ
オレなんか1ミリもわからんわワッハッハ
orz

863 :132人目の素数さん:2021/03/09(火) 20:21:44.57 ID:Yzn2wvw2.net
4月号の問題きてるね

864 :132人目の素数さん:2021/03/09(火) 20:48:35.80 ID:atvTbRm0.net
3月号の問1は史上最も簡単だったのでは

865 :132人目の素数さん:2021/03/09(火) 21:55:56.09 ID:e9fDNuY/.net
3月号出題2はお茶を濁す解答になってしまった。変数変換後の領域(像)をどこまで正確に論じるか

866 :132人目の素数さん:2021/03/09(火) 21:59:14.72 ID:e9fDNuY/.net
4月号出題2はおもしろそう

867 :132人目の素数さん:2021/03/10(水) 13:39:00.18 ID:PeuQmY3+.net
3月号出題2の解説 (屮゜Д゜)屮カモーン

868 :132人目の素数さん:2021/03/10(水) 18:46:36.62 ID:PeuQmY3+.net
sin,cosの代わりにsinh,cosh使うだけか

869 :132人目の素数さん:2021/03/10(水) 21:54:07.36 ID:dMP4wwTf.net
うむ。
 x = sinhξ/coshη, y = sinhη/coshζ, z = sinhζ/coshξ,
等とおけば、D_2 の定義式は
 0 ≦ sinhξ ≦ coshη,
 0 ≦ sinhη ≦ coshζ, 
 0 ≦ sinhζ ≦ coshξ,
のようなものかな?

問題はこれをどこまで正確に論じるか?   >>865
 
半直線 ξ=η=ζ≧0 を含む希ガス…

870 :132人目の素数さん:2021/03/10(水) 21:54:07.36 ID:dMP4wwTf.net
うむ。
 x = sinhξ/coshη, y = sinhη/coshζ, z = sinhζ/coshξ,
等とおけば、D_2 の定義式は
 0 ≦ sinhξ ≦ coshη,
 0 ≦ sinhη ≦ coshζ, 
 0 ≦ sinhζ ≦ coshξ,
のようなものかな?

問題はこれをどこまで正確に論じるか?   >>865
 
半直線 ξ=η=ζ≧0 を含む希ガス…

871 :132人目の素数さん:2021/03/10(水) 22:07:18.33 ID:PeuQmY3+.net
そうそう
領域は無限に大きくなる
けど体積は有限

872 :132人目の素数さん:2021/03/10(水) 22:59:50.48 ID:x9yOJlKS.net
元のsin,cosの方は
0≦α≦asin(cos(β)) = π/2-β
0≦β≦asin(cos(β)) = π/2-γ
0≦α≦asin(cos(γ)) = π/2-α
で5点O(0,0,0), A(π/2,0,0), B(0,π/2,0), C(0,0,π/2),
D(π/4,π/4,π/4)の凸包、体積は三角錐OABCの3/2倍で
3/2×1/6×(π/2)^3=π^3/32
hyperbolic版はasinh(cosh(x))がasinh(cos(x))≧xにより領域はx=y=z>0を含む無限領域になる
 

873 :132人目の素数さん:2021/03/10(水) 23:36:21.01 ID:DmnTyu8T.net
出題1の解答
https://www.i.h.kyoto-u.ac.jp/users/tsuiki/3H6T/index.html

874 :132人目の素数さん:2021/03/11(木) 17:18:59.70 ID:i8irJ9V7.net
n=2の場合でsinh(x),cosh(x)使うと結局Σ1/(2n-1)^2の別計算になるんだな

integrate [0infty] (asinh(cosh(x)-x)dx
=
integrate [1,infty] (log(sqrt(t^2+1)+t) - log (sqrt(t^2-1)+t))/sqrt(t^2-1)dt
=
π^2/16
=0.616850275068...

integrate [1,infty] (log(sqrt(t^2+1)+t) - log (sqrt(t^2-1)+t))/sqrt(t^2-1)dt

875 :132人目の素数さん:2021/03/11(木) 17:33:32.53 ID:i8irJ9V7.net
間違えた
コッチ
https://www.wolframalpha.com/input/?i=integrate+%5B1%2Cinfty%5D+%28log%28sqrt%28t%5E2%2B1%29%2Bt%29+-+log+%28sqrt%28t%5E2-1%29%2Bt%29%29%2Fsqrt%28t%5E2-1%29dt&lang=ja

876 :132人目の素数さん:2021/03/12(金) 23:37:57.49 ID:qFD/RlMf.net
>>870
突起部を平面 x+y+z = 3a で切った断面を考える。
中心 (a,a,a) から最も遠い点は 2曲面の交線上にあり、
 (a-2e^(-2a), a, a+2e^(-2a)) とその rotation で、
 中心からの距離は 2(√2)e^(-2a)
と近似される。
さらに断面を正三角形と仮定すると面積は
 S = 6(√3)e^(-4a),
この平面と O(0,0,0) の距離は h = (x+y+z)/√3 = a√3 だから
 S(h) = 6(√3)e^(-4h/√3),
突起部 (x+y+z>2) の体積は
 ∫[2/√3, ∞] S(h) dh ≒ (9/2)e^(-8/3) = 0.31267553
ぐらいかな?

877 :132人目の素数さん:2021/03/13(土) 06:35:51.25 ID:D5MHIZN9.net
0 < x+y+z < log(1+√2) = b = 0.881373587 の部分は
 S(h) = (3/2)(√3)h^2,
 V = ∫[0,b] S(h) dh = (1/2)(√3)b^3 = 0.11411135

878 :132人目の素数さん:2021/03/13(土) 08:42:39.54 ID:D5MHIZN9.net
訂正スマソ
 V = ∫[0,b/√3] S(h) dh = (1/6)b^3 = 0.11411135


S(2/√3) = 0.73308 > 0.72209 = 6(√3)e^(-8/3),
S(√3) = 0.18893 < 0.19034 = 6(√3)e^(-4),

r(2/√3) = 0.83890 > 0.745565 = 2(√2)e^(-4/3),
r(√3) = 0.39544 > 0.382785 = 2(√2)e^(-2),

879 :132人目の素数さん:2021/03/13(土) 15:24:05.70 ID:Twk+dGtm.net
今月号の1番は4直線と正方形の配置は問題文の例の図に決め打ちしていいんかな?
単純に
「4直線が与えられているとき、それぞれから一点ずつを選んで正方形を作図せよ」
と読み替えてしまうと配置の可能性がかなり出てきて、それぞれによって違う作図法が必要になる希ガス
説明の仕方工夫してキレイにまとめる事も要求されてるのかな?
正直方程式立ててやっちゃう方が短いけど、それはエレガントと言えない気もするし

880 :132人目の素数さん:2021/03/13(土) 16:55:13.94 ID:NoAyNlDk.net
>>879
色々見つけたならどれか1個示せばよいかと。
余裕あるなら網羅すればよいかと
おれはいつも最低要求レベルの答案で満足してる

881 :132人目の素数さん:2021/03/13(土) 17:54:46.23 ID:r85d/wY8.net
>>880
イヤイヤ、方法が色々あるんじゃなくて4点と言っても正方形と4直線の関わり方でいくつかケースがあって、「このケースならこの方法で作図できて、このケースならこの方法で作図できて‥」といっぱいケースがあるんだよ
あんまり細かいこと書くとヒントになるから描けないけどオレの方法だと円書いて点取ってそこから幾ばくか離れたところに点取るとき右なのか左なのかとかケースによって変わってきて微妙に作図方法が変わる
2、3例しかないならまだしもいっぱい配置がありうるんだよ
どうしたもんかなと思って
見つかる四角形が問題文中に例示されてる配置に決まってるならそんな心配いらないんだけど

882 :132人目の素数さん:2021/03/13(土) 18:04:46.85 ID:4CZl6pQq.net
そら問題文の図だけじゃなくて一般的に論じるのがベターだろ
極端な話、例えば4直線が互いに平行に等間隔で並んでたりしたら自明なわけで

883 :132人目の素数さん:2021/03/13(土) 19:12:11.81 ID:Twk+dGtm.net
>>882
そうだわな
となるととてもエレガントとは言えんのかな
再考

884 :132人目の素数さん:2021/03/14(日) 07:12:55.57 ID:OBNK+OoH.net
進みが速いなあ
いまは中休みの時期だろ
そして1週間前になって焦る

885 :132人目の素数さん:2021/03/20(土) 01:00:04.50 ID:0BqbVchW.net
4月号だし簡単だろと思ってたら意外とむずいんだが?

886 :132人目の素数さん:2021/03/20(土) 22:49:36.50 ID:/7hHX3Cf.net
5月号じゃない?簡単なのは。新入生が買うのはそれだから。

887 :132人目の素数さん:2021/03/21(日) 10:29:04.30 ID:dgPR3iTS.net
>>877-878

0<x+y+z<b が 0.114
b<x+y+z<2 が 0.627
2<x+y+z が 0.311
合わせて 1.052

理論値 (7/8)ζ(3) = 1.0518 に近い…

888 :132人目の素数さん:2021/03/21(日) 14:15:40.10 ID:dgPR3iTS.net
ζ(3) = Σ[n=1,∞] 1/n^3 を「アペリーの定数」
それが無理数であることを「アペリーの定理」と云うらしい。

三井孝美「数論の最近の話題から」
 数学セミナー, vol.18, no.12 (1979/Dec)
 数セミ増刊「数の世界」日本評論社 (1982) p.142-150

889 :132人目の素数さん:2021/03/23(火) 02:22:36.14 ID:vqPaE4VE.net
作図できたけどgeogebraが真っ黒になった

890 :132人目の素数さん:2021/04/05(月) 00:25:21.93 ID:Rx9o0dRV.net
2問とも良問、すばらしき春の日かな

891 :132人目の素数さん:2021/04/09(金) 12:21:42.61 ID:MdEONQVB.net
5月号は既視感あるな

892 :132人目の素数さん:2021/04/09(金) 23:21:30.25 ID:qtjVxAQC.net
2021年4月号
■出題2
単一閉回路 (ジョルダン閉回路) を「ループ」と呼ぶなら、
奇数本の電灯がオンであるようなループが無いこと
かな?

893 :132人目の素数さん:2021/04/10(土) 10:15:00.64 ID:C7VQuk94.net
>>892
うむ。十分性の議論がきちんとできたかどうかがポイントかな

894 :132人目の素数さん:2021/04/10(土) 10:18:16.12 ID:C7VQuk94.net
問1も面白かったな
おれは中心を割り出す方法を採った

895 :132人目の素数さん:2021/04/11(日) 10:11:40.33 ID:Wdzgq4aa.net
一松先生大盤振る舞い、全員正解にしちゃったw

896 :132人目の素数さん:2021/04/11(日) 10:15:13.71 ID:B/ZwQ0zG.net
2/8〆のやつ?
まぁあれは問題文微妙だったからなぁwww

897 :132人目の素数さん:2021/04/11(日) 23:09:19.30 ID:XpMtbDgW.net
誰か出題1の答えあげてよ
オレの見つけた方法だと4本の直線のどこに頂点かわくるかの配置でめちゃめちゃ可能性があってとてもエレ解とは言えない
作図方法そのものじゃなくて作図可能である事だけ示すなら少しは楽になるけど

898 :132人目の素数さん:2021/04/12(月) 03:21:10.22 ID:gq7I01ph.net
出題1はこれを参考にした
https://www.nikkei-science.com/page/magazine/alice/201802/question.html

899 :132人目の素数さん:2021/04/12(月) 12:16:30.51 ID:scUzzqi2.net
まず4直線ではなく3直線を通る正方形を考える。これは無数にある

異なる2つの3直線の組に対して同じことをやると、通常1通りに絞られることが分かる

で、その存在が保証されていることから作図可能とわかる

900 :132人目の素数さん:2021/04/12(月) 12:21:25.53 ID:scUzzqi2.net
最初のステップ、3直線を通る正方形については、少なくとも頂点の1つが直線の交点となる極端なケースでは作図可能
この事実をうまく使えば作図可能な正方形の軌跡が得られ、次のステップに進める

901 :132人目の素数さん:2021/04/13(火) 00:50:45.30 ID:SfYSo0zq.net
オレがつけた作図可能の証明

4本中3本が平行な時には議論は容易ゆえそうでないとする
この時4本の直線k,l,m,nのうちk,lとm,nは交点を持つとして良い
実際、どの2本も平行なら当然で、一組平行である時k,mは平行として良い
この時仮定によりkとl、mとnは平行ではない
kとlの交点をX、mとnの交点をYとする
まずXとYが一致しない時を考える
適当に座標軸を固定してk〜nの偏角をθk〜θnとする
k,l,m,n上の点KLMNで正方形の頂点をなす物のうち、まずKLが対角線となる場合を考える
Kt(cost, sint)を通りkに平行な直線をk(t)、
Lt(-cost, -sint)を通りlに平行な直線をl(t)、
Mt(sint, -cost)を通りmに平行な直線をm(t)、
Nt(-sint, cost)を通りnに平行な直線をn(t)とする
k(t)、l(t)の交点をX(t)、m(t)、n(t)の交点をY(t)とすればX(t),Y(t)の座標はcos(t),sin(t)の線形結合となる
よってXYベクトルとX(t)Y(t)ベクトルが平行となる条件はtan(t)=の形を取る
その方程式を解いて得られる(cos(t),sin(t))の解は作図可能である
コレを用いて得られるtを用いてk(t)〜n(t)を用いて得られる図形は元の図形と相似で相似の中心も容易に作図できる
この相似変換でKt〜Ntに対応するk〜n上の点が求める点である
X=Yとなる場合やKLMNの順番が異なる場合も同様である

902 :132人目の素数さん:2021/04/13(火) 01:03:18.40 ID:4sZyeNlo.net
作図可能性を言いたいだけなら線形代数でよくないか?
90°の回転行列をRとしてAB=R BC, BC=R CDが条件だから、加減乗除だけで解けてもちろん作図可能

903 :132人目の素数さん:2021/04/13(火) 01:45:33.62 ID:SfYSo0zq.net
回転の中点点を(X,Y)として4本の直線k,l,m,nを(X,Y)を中心に0,R,2R,3R回転して得られる直線の方程式はX,Yの一次式で得られる
それが全て同一の点をとおる事、すなわちコレら4本の直線の係数のなす4行3列の行列のrankが2以下がX,Yのなすべき条件
その解が二次以下の方程式の繰り返しで得られる事の証明ができればそれでもいい

904 :132人目の素数さん:2021/04/13(火) 02:13:56.01 ID:SfYSo0zq.net
あれ?
その証明そんなに難しくないな

905 :132人目の素数さん:2021/04/13(火) 02:54:23.10 ID:AEPZEFT7.net
コレでよかったのか
なるほど

直線kを点P(X,Y)中心に+90°回した直線をkP、
直線lを点P(X,Y)中心に-90°回した直線をlPとして
kPとlPの交点が直線m上であるという条件を満たすPの軌跡aは直線になる
よってこの軌跡上の2点を作図できれば直線aが作図できる
ここでm上の点XをひとつとりXを通るk,lに垂直な直線k',l'をとる
このk'とkの2等分線の片方の上の任意の点を中心とする+90°回転によってkはk'にうつる
同様にl'とlの2等分線の片方の上の任意の点を中心とする-90°回転によってlはl'にうつる
よってこのふたつの2等分線の交点は直線a上にある
同様の構成をm上の別の点で行えばa上のもう一つの点を作図できる
コレでaが作図できる
同様にしてPを中心にkを-90°回転させた直線とlを+90°回転させた直線の交点がn上にあるようなPの軌跡のなす直線bも作図できる
このa,bの交点が求める正方形の中心である

906 :132人目の素数さん:2021/04/28(水) 17:57:53.96 ID:88VQNkKd.net
5月号なら簡単だろと思ってたらこれもむずいんだが?

907 :132人目の素数さん:2021/04/28(水) 21:57:32.54 ID:zYonR4F/.net
簡単そうと思って手を付けずにいたんだが難しい、だと?
困るじゃないか

908 :132人目の素数さん:2021/05/04(火) 13:27:48.37 ID:ItoSURf2.net
出題1簡単過ぎないか?
解釈間違ってるのかも

909 :132人目の素数さん:2021/05/05(水) 07:43:38.83 ID:7b/pfnNf.net
2番は?

910 :132人目の素数さん:2021/05/05(水) 12:34:36.13 ID:jJ+lV1kt.net
>>908
失敬やっぱり難しかった

911 :132人目の素数さん:2021/05/05(水) 13:57:44.13 ID:vTnGlv4q.net
だよな
てか全く初頭的な証明が分からん
ガロア理論とか使えば一発だけど

912 :132人目の素数さん:2021/05/05(水) 15:10:37.47 ID:jJ+lV1kt.net
>>911
問1クリアできたっぽい

913 :132人目の素数さん:2021/05/08(土) 10:32:48.34 ID:zmPJMSXR.net
2が解けないなー

914 :132人目の素数さん:2021/05/08(土) 14:15:01.46 ID:izXhYIuc.net
出題2もガロア理論使っていいなら一瞬なんだけど

915 :132人目の素数さん:2021/05/08(土) 17:56:18.49 ID:zmPJMSXR.net
>>914
問2はガロア拡大が活躍する典型問題
ガロアを使う解答はエレガントだけどエレ解としてはエレガントでないという矛盾

916 :132人目の素数さん:2021/05/08(土) 19:33:37.13 ID:rzut8Ezb.net
今月号の出題1だが、ガロア理論の「ガ」の字もでてこなかったんだけど・・・。
とりあえず締め切り以降に、回答らしきものを出す予定。

917 :132人目の素数さん:2021/05/08(土) 20:10:14.08 ID:zmPJMSXR.net
>>916
何故締切以降?

918 :132人目の素数さん:2021/05/08(土) 20:11:09.52 ID:zmPJMSXR.net
>>917
ああめんご5chに投稿するってことね

919 :132人目の素数さん:2021/05/08(土) 21:45:55.62 ID:PfNBiEsz.net
有理数は無い

920 :132人目の素数さん:2021/05/08(土) 21:57:35.96 ID:izXhYIuc.net
ガロア理論使わない解答がもちろん想定解だろう
まぁオレはガロア理論使った回答できたので満足してるけど

921 :132人目の素数さん:2021/05/09(日) 00:05:35.46 ID:B9RgXt6+.net
0でない整数mが素数pのu乗で割り切れ、かつu+1乗で割り切れないとき
u=v(p:m)と書くことにする。

ab=0あるいはgcd(a,b)>1のときは明らか。
ab≠0かつgcd(a,b)=1のときを考える。

a<0のとき、a・x^n+b・y^nの代わりに、(-a)・x^n+(-b)・y^nを考えれば同様に示せるので
a>0と仮定しても一般性を失わない。

ある素数pに対して、v(p:ab)≧2となるとき
v(p:a・x^n+b・y^n)=0あるいはv(p:a・x^n+b・y^n)≧2だから
v(p:m)=1となるような整数mは題意をみたす。

n≧3でかつ、ある素数pに対して、v(p:ab)=1となるとき
v(p:a・x^n+b・y^n)=0,1あるいはv(p:a・x^n+b・y^n)≧n≧3だから
v(p:m)=2となるような整数mは題意をみたす。

よって、「n≧2かつa=1,b=±1」または「n=2かつ-abはsquare free」
の場合のみ示せばよいことがわかる。

922 :132人目の素数さん:2021/05/09(日) 00:06:07.17 ID:B9RgXt6+.net
ここで補題を二つ上げる。

補題1
s,tを整数、pを素数とする。
そのとき、v(p:s^p-t^p)=0あるいは、v(p:s^p-t^p)≧2がいえる。

補題1の証明
s^p-t^pがpで割り切れないときは明らか
s^p-t^pがpで割り切れるとき
s≡s^p≡t^p≡t (mod p)がいえるから
s^(p-1)+s^(p-2)t+…+t^(p-1)≡pt^(p-1)≡0 (mod p)より
s^p-t^p=(s-t){s^(p-1)+s^(p-2)t+…+t^(p-1)}≡0 (mod p^2) ■

補題2
s,tを整数、wを平方数ではない整数かつpをwが平方非剰余となるような素数とする。
そのとき、v(p:s^2-w・t^2)=0あるいは、v(p:s^2-w・t^2)≧2がいえる。

補題2の証明
s^2-w・t^2がpで割り切れないときは明らか
s^2-w・t^2がpで割り切れるとき
tがpで割り切れないと仮定するとw≡(s/t)^2 (mod p)となり
wがpの平方非剰余であることに反する。
よって、tはpで割り切れる。したがってsもpで割り切れる。
以上より、v(p:s^2-w・t^2)≧2がいえる。 ■

923 :132人目の素数さん:2021/05/09(日) 00:07:35.00 ID:B9RgXt6+.net
a=1,b=-1のとき
nの素因数pをとると、補題1よりv(p:x^n-y^n)=0あるいは、v(p:x^n-y^n)≧2がいえる
から、v(p:m)=1となるような整数mは題意をみたす。

nは奇数かつa=1,b=1のとき
nの素因数pをとると、補題1よりv(p:x^n+y^n)=0あるいは、v(p:x^n+y^n)≧2がいえる
から、v(p:m)=1となるような整数mは題意をみたす。

nは偶数かつa=1,b=1のとき
x^n+y^n≡0,1,2 (mod 4)より、m≡3 (mod 4)となるような整数mは題意をみたす。

n=2かつ-abはsquare freeとなるとき、
-abが平方非剰余となるような素数pをとる。
v(p:m)=1となるような整数mに対して、m=a・(x_0)^2+b・(y_0)^2
をみたす整数x_0、y_0が存在すると仮定すると
am=(a・x_0)^2-(-ab)・(y_0)^2だから、
補題2より、v(p:(a・x_0)^2-(-ab)・(y_0)^2 )=0あるいはv(p:(a・x_0)^2-(-ab)・(y_0)^2 )≧2
がいえるが、v(p:am)=1だから不合理。よってv(p:m)=1となるような整数mは題意をみたす。

924 :132人目の素数さん:2021/05/09(日) 00:08:16.38 ID:B9RgXt6+.net
平方数ではない整数wに対してwが平方非剰余となるような素数pが必ず存在することの証明

w=-1のときp≡3 (mod 4)

v(2:w)が奇数のとき、
「p≡5 (mod 8)
wの2以外の素因数qに対してp≡1 (mod q)」…☆
とおくと、中国剰余定理とディリクレの算術級数定理より
☆をみたす素数pは必ず存在する。

上記のいずれでもないとき
v(q?0:w)が奇数となるような素因数q_0と、q_0の平方非剰余r_0をとる。
このとき
p≡r_0 (mod q_0)
p≡1 (mod 8)
wのq_0以外の素因数qに対して
p≡1 (mod q)」…★
とおくと、中国剰余定理とディリクレの算術級数定理より、
★をみたす素数pは必ず存在する。

いずれにせよwはpの平方非剰余になる。

925 :132人目の素数さん:2021/05/09(日) 00:11:44.00 ID:B9RgXt6+.net
>>924
×v(q?0:w)
〇v(q_0:w)

926 :132人目の素数さん:2021/05/09(日) 01:08:15.24 ID:v3AkR5Oi.net
奇素数ベキのa=1,b=-1がネックだったんだがv(p:m)≠1か‥気づかなかったな

927 :132人目の素数さん:2021/05/09(日) 01:46:39.27 ID:LEPemAHr.net
出題1
n=2のときは容易である
nは奇素数として良い
またa,bは互いに素として良い
f(t)=at^n+bとおく
ζ=exp(2πi)、α=|b/a|^(1/n)とする
(i) αが有理数のとき
このとき|a|^(1/n),|b|^(1/n)ともに整数であるからa=b=1として良い
素数pをQ(ζ)/QにおけるpのFrobenius共役類Fが単位元でないものを取る、すなわちp≡1 ( mod n )でないものを取る
p=x^n+y^nであるとすればx+y=±pまたはx+y=±1である
前者のときx^(n-1)+‥+y^(n-1)=±1となるが左辺はx>yのときは前から2つずつ出すと(n-1)/2個の正の整数のの和とy.^(n-1)の和であるからコレが±1となることはない
後者のときx^(n-1)+‥+y^n=pとなるが、このときζの最小多項式がmod pで有理解を持つ事になりFが単位元の属する類である事に反する
いずれにせよ矛盾するので(i)は起こり得ない
(ii)αが無理数のとき
このときK=Q(α,ζ)とおけばGal(K/Q)の元σでσ(α)=αζとなるものがとれる
素数pてそのFrobenius共役類Fがσの属する類とすれば方程式ax^n+by^n=pが整数解を持つとすればyはpの倍数足り得ず(x/y)のるいはZ/pZにおいてt^n+b/a=0の解となるが、コレはσがα,αζ,‥に不動点なしに作用する事に反する

928 :132人目の素数さん:2021/05/09(日) 09:38:59.26 ID:3/uQnkqI.net
出題2のガロア以外の解答は?

929 :132人目の素数さん:2021/05/09(日) 15:22:58.28 ID:KICdmF8d.net
生姜ねゑな。。。

2021年5月号
■出題2
まづ 1/(2^(1/3)) = q とおく。

cos(πθ) = q となる有理数θ = m/n が存在したと仮定する。
 nは自然数、mは整数。
このとき cos(nπθ) = cos(mπ) = (-1)^m  … (*)

ところで、和積公式
 cos((n+1)φ) = (2cosφ)cos(nφ) - cos((n-1)φ),
から
 2cos(nφ) は 2cosφ = 2x の 整数係数の多項式で表わせる。
ことが分かる。(nについての帰納法で)
 cos(nφ) = T_n(cosφ),
n次の第一種チェビシェフ多項式と云うらしい。

T_n(x) を 2x^3 -1 で割って
 T_n(x) = (2x^3 -1)Q(x) + a・xx + b・x + c,
とおく。 a_n, b_n, c_n は整係数。

2q^3 -1 = 0 より
 T_n(q) = a・qq + b・q + c,
これらにより、(*) は
  a・qq + b・q + c = (-1)^m,
となるが、{qq,q,1} はQ上1次独立だから (証略)
 a_n = b_n = 0, c_n = ±1,
となる自然数nが存在する。  (続く)

930 :132人目の素数さん:2021/05/09(日) 15:29:26.73 ID:KICdmF8d.net
ところで 2(qω)^3 - 1 = 0 からも
 T_n(qω) = a(qω)^2 + b(qω) + c,
これの挙動を調べてみる。
 |λ| = √(2q(q+1)+1) = 1.9614591767
 φ = π(1+θ)/2 = 1.897760298045
とおくと
 cos(φ +iLog|λ|) = qω,
 T_n(qω) = cos(nφ +in・Log|λ|)
  = cos(nφ)cosh(n・Log|λ|) +isin(nφ)sinh(n・Log|λ|),

|T_n(qω)|^2 = cos(nφ)^2 + sinh(n・Log|λ|)^2
   = cos(nφ)^2 + [(|λ|^n - |λ|^{-n})/2]^2,
∴ |T_n(qω)| ≧ (|λ|^n - |λ|^{-n})/2
∴ nとともに 等比数列的に増大する。
a, b, c の少なくとも一つは1より大きくなり、
 a_n = b_n = 0, c_n = ±1
と矛盾する。 ■

ガロア理論使ってないけど、ちっともエレガントぢゃねゑ。。。

931 :132人目の素数さん:2021/05/09(日) 15:40:00.49 ID:KICdmF8d.net
ω = (-1 + (√3)i)/2 は1の3乗根ですた。スマソ

932 :132人目の素数さん:2021/05/09(日) 15:54:33.05 ID:FwjTy9yc.net
改めてガロアの遺した業績の偉大さがわかるスレ

933 :132人目の素数さん:2021/05/09(日) 16:01:20.64 ID:FwjTy9yc.net
解けるのは解けるけど答えだけ見るとすごいシンプルな式になる
もっと鮮やかな導出がありそうだな

934 :132人目の素数さん:2021/05/09(日) 16:01:56.37 ID:FwjTy9yc.net
>>934
誤爆スマソ

935 :132人目の素数さん:2021/05/09(日) 16:26:44.53 ID:3/uQnkqI.net
常連の剛腕キタコレ

936 :132人目の素数さん:2021/05/09(日) 16:51:03.44 ID:jYmv+ON6.net
>>927
> (n-1)/2個の正の整数のの和とy.^(n-1)の和であるからコレが±1となることはない

なんで?

937 :132人目の素数さん:2021/05/09(日) 17:07:28.13 ID:FwjTy9yc.net
>>936
x^(n-1)-x^(n-2)y+x^(n-3)y^2+‥+y^(n-1)はxとyが異符号の場合には上記の和はぜんぶ同符号なのでもちろん±1になることはない
x>yの場合には前から2つずつくみあわせて一般項は
x^(2i)y^j - x^(2i-1)y^(j+1)
で必ず正
それに最後の項y^(n-1)も正なので総和は1より大きい
両方負の場合も同じ

938 :132人目の素数さん:2021/05/09(日) 17:45:27.75 ID:FwjTy9yc.net
あぁ、凸不等式なら一発か
ft)=|x|^t|y|^(n-1-t)とおいて凸関数
|x^(n-1)-x^(n-2)y+‥|
≧|x^(n-1)|-|x^(n-2)y|+‥
=(1/2)( |x|^(n-1) + |y|^(n-1) + Σ(f(2i-1) - f(2i+1)-2f(2i)) )
でカツコ内の第3項は凸不等式より正
第一項、第二項は正の整数だから共に1以上
よって和は1以上だがx≠yより1にはならない

939 :132人目の素数さん:2021/05/09(日) 21:00:03.34 ID:KICdmF8d.net
>>930
 |λ| = √{q/(1-q)} = 1.9614591767
 -cosφ = sin(πθ/2) = √{(1-q)/2} = 0.3211693276
 sinφ = cos(πθ/2) = √{(1-q)/2} (√3)/(2q-1) = 0.9470217859

|a|, |b|, |c| の少なくとも一つは1より大きくなり…

940 :132人目の素数さん:2021/05/10(月) 00:23:08.17 ID:i71w++EK.net
>>929
漸化式
 a_{n+1} = 2b_n - a_{n-1},
 b_{n+1} = 2c_n - b_{n-1},
 c_{n+1} = a_n - c_{n-1},

 u_{n+6} + 3u_{n+4} - 4u_{n+3} + 3u_{n+2} + u_n = 0,
特性多項式
 (tt+1)^3 - 4t^3
 = (tt - 2qt + 1){tt + 2qqt + q/(1-q)}{tt + 2q(1-q)t +(1-q)/q}
 = (t - e^{iπθ}) (t - e^{-iπθ}) (t - λ) (t - λ') (t - 1/λ) (t - 1/λ')

941 :132人目の素数さん:2021/05/10(月) 23:43:29.27 ID:aNmZ1Qa+.net
6月号問1の(2)なにこれ

942 :132人目の素数さん:2021/05/10(月) 23:56:53.84 ID:B8kGmpeH.net
受験数学ではありえないテーマやな
条件と必要十分になるθの条件を好きになんでも選べって事やろ
もちろん合ってる合ってないなら問題文の条件そのままコピペしても必要十分
正しいか正しいかではない
エディタを1番唸らしたものの勝ち

943 :132人目の素数さん:2021/05/11(火) 00:01:14.29 ID:nf4TeMQU.net
あ、もしかしたら簡単すぎるやんって意味?
それはそうかも

944 :132人目の素数さん:2021/05/11(火) 16:03:18.46 ID:V1E4ia2/.net
やっぱり簡単すぎるよなあ
ただしθは整数度、が抜けてるとかだろうか
ぶっ飛んだ言い換えをして審査員特別賞狙いにいくのは面白いかもと思った

945 :132人目の素数さん:2021/05/11(火) 23:34:28.62 ID:V7aKcUts.net
この問題を言い換えると、取りうるθを全部求めよってことだと思うけど

946 :132人目の素数さん:2021/05/20(木) 12:04:28.47 ID:P4oltw8K.net
5月号2番のガロア理論解答は?

947 :132人目の素数さん:2021/05/20(木) 13:17:32.35 ID:02+d2nOa.net
解1
m,nを互いに素である自然数とする時
[Q[cos(2πm/n):Q]
=φ(n)/2 (n≧3)
=1 (φはeulerのトーシェント関数)
φ(n)/2=3 となるのはn=7,14のみ
何も(1/2)^(1/3)を含まない

解2
任意の有理数θに対しQ(cos(πθ))/Qはガロア拡大
しかしQ((1/2)^(1/3))/Qは(1/2)^(1/3)の共役元(1/2)^(1/3)exp(2πi/3)を含まないのでガロア拡大ではない

948 :132人目の素数さん:2021/05/30(日) 20:34:22.17 ID:+7Z4asMt.net
今月は穏やかだな

949 :132人目の素数さん:2021/06/04(金) 10:12:54.79 ID:1f/LdWNr.net
こういうエレガントがどうこう言ってる限りダメだな
どんな問題でも最初解く時は力技だよ
エレガント云々言ってることは
新しい問題に取り組め無くなることと事実上同じでっせ

950 :132人目の素数さん:2021/06/04(金) 10:14:08.51 ID:1f/LdWNr.net
エレガントで無くていいから
まず解けな
注目する点を間違ってるぞ





 

951 :132人目の素数さん:2021/06/07(月) 11:26:46.84 ID:7mnf7YYR.net
7月号の問題はまだ?

952 :132人目の素数さん:2021/06/07(月) 21:56:25.53 ID:lGSE8Vpn.net
締切まだですから

953 :132人目の素数さん:2021/06/09(水) 10:16:12.64 ID:W7KpASHn.net
エレガントで無くていいから
まず解けな
注目する点を間違ってるぞ
 




 

954 :132人目の素数さん:2021/06/09(水) 12:14:47.41 ID:muX9ql7V.net
7月の問題キタ
文が長め

955 :132人目の素数さん:2021/06/10(木) 09:12:11.83 ID:wOdXYdYP.net
2021年6月号

■出題1
(1) は あと回し。
(2)
O以外の格子点
 A(a1,a2) B(b1,b2)
に対して
 C(c1,0) c1 = (a1)^2 + (a2)^2 = OA^2,
 D(d1,d2) d1 = a1b1+a2b2, d2=a1b2-a2b1,
とおくと、3平方の定理より
 OC/OA = OD/OB = CD/AB = OA,
∴ 僊OB ∽ 僂OD   (相似)
∴ θ = ∠AOB = ∠COD = ∠D'OD,
D'(d1,0) はDからx軸に下した垂線の足。
∴ θ=∠AOB ならば、θを内角とする直角三角形の
直辺の比 (つまり tangent) は整数比をなす。
逆は明らか。

(1) 上手いやり方が見つからない…
格子点 A(x,0) B(x,y) (y≠0)
が ∠AOB=θ を満たすとする。さらに
 C(xx-yy, 2xy)
 D(x(xx-3yy), (3xx-yy)y)
とおく。
 僊OB ∽ 傳OC ∽ 僂OD,
 ∠AOB = ∠BOC = ∠COD,
 ∠AOD = ∠AOB + ∠BOC + ∠COD = 3θ,
θ=60° とすると Dはx軸上にあるから
 (3xx-yy)y = 0 かつ y≠0
∴ 3xx = yy,
左辺の3の指数は奇数、右辺の3の指数は偶数(0も含む)
∴ これを満たす格子点(x,y)は存在しない。
かなり牛刀だなぁ。

956 :132人目の素数さん:2021/06/10(木) 18:43:37.15 ID:wOdXYdYP.net
2021年6月号

■出題2
・n=2 と n=4 は (1)は可能、(2)は不可能。
 タイルの総数m ≦ [nn/3]
 奇数行、奇数列のマス目の数 [(n+1)/2]^2 ≦ m,
∴ [(n+1)/2]^2 ≦ [nn/3]
・n=3 と n=5 は不可能。
・n=6n' は 2x3 ブロックを縦横に並べればよいので、全Lで可能。
・n=7 は L +1S で可能。
・n=3n'+1 は 3n'-1 に2行2列(全L)を追加して可能。
・n=3n'+2 は 3n' に2行2列(∋1S)を追加して可能。
・n=6n'+3 は 6n'+1 に2行2列(∋2S)を追加して可能だが、
 S型を減らせないので (2)では使えない。
これらより
 nが6以上の偶数のときは可能。
 nが7以上の奇数のときは (1)は可能、(2)は?
以上まとめて
(1) n=3, n=5 以外のすべて
(2) 6以上の偶数と7と…?

957 :132人目の素数さん:2021/06/10(木) 22:06:37.52 ID:sXtlFBJ1.net
(2)で不可能なのはn=2,4,5のみ
あとは可能

958 :132人目の素数さん:2021/06/10(木) 22:07:39.70 ID:q3ojMB7O.net
n=1も不可能

959 :132人目の素数さん:2021/06/10(木) 22:07:55.83 ID:q3ojMB7O.net
n=3もやorz

960 :132人目の素数さん:2021/06/10(木) 22:31:55.79 ID:nnbbsZTh.net
タイルの問題(1)の意味あるか?なんかヒントになってた?

961 :132人目の素数さん:2021/06/10(木) 23:24:44.72 ID:+Ncm1Ku6.net
>>960
難易度下げて敷居低くしただけと思われ

962 :132人目の素数さん:2021/06/10(木) 23:50:24.63 ID:XvcT0GX9.net
あとn=5の時に不可能である事を示すのは(1)の方が難しい
不可能性の証明は(1)の方が難しくなるけど、そもそも不可能であるのがちょっとしかないからなぁ

963 :132人目の素数さん:2021/06/11(金) 01:31:27.60 ID:lkMvJv8z.net
(2) の答は 6以上のすべて
ですね

964 :132人目の素数さん:2021/06/11(金) 06:23:37.51 ID:ti7gUo9H.net
yes

965 :132人目の素数さん:2021/06/11(金) 06:38:13.23 ID:ti7gUo9H.net
オレ的なキーは
2×5, 7×7, 5×9
が見つけられるかどうかがだった様な気がする
m×nまで解決するには4×7もミソ

┏━┳━┳━━┳━┓
┃ ┃ ┃  ┃ ┃
┃ ┃ ┣━┳┫ ┃
┣━┻┳┫┏┫┗┳┫
┃  ┃┗┫┗┳┛┃
┗━━┻━┻━┻━┛
┏━━┳━┳━┓
┃  ┃ ┃ ┃
┣┳━┫ ┃ ┃
┃┗┓┗┳┻━┫
┣━╋━┫  ┃
┃ ┃ ┣━━┫
┃ ┃ ┃  ┃
┗━┻━┻━━┛
┏━━┳━┳━┓
┃  ┃┏┻┓┃
┣┳━╋┻┓┣┫
┃┗┓┗┓┣┛┃
┗━┻━┻┻━┛
┏┳━┳━┓
┃┗┓┗┓┃
┗━┻━┻┛

966 :132人目の素数さん:2021/06/11(金) 20:04:12.04 ID:q8bkQe7j.net
(2)の条件を緩めて

S字ブロックは2個以内

としてm×nて分割可能てある場合について考える

Prop 2×n (n-=3,n≧5)、3×2n (n≧1)は全て可能
∵ ) 前半は2×3,2×5,2×7可能であることから容易
後半は自明

Prop m≧n≧4のとき(m,n) = (4,4), (5,5), (5,7)を除いて全て可能

∵ ) (m,n)が分割不可能なら(m-6,n)も分割不可能であるから最小反例があるとすればm≦9であるか、(m,n) = (10,4),(11,5),(13,5),(11,7)のいずれかであるが、コレらの場合は下の計算と分割例で反例足り得ないとわかる
(式の意味はエスパーすべし)

(4,4) = impossible
(4,5) = (2,5)+(2,5)
(4,6) = any × 6m
(4,7) = (4,7)
(4,8) = ((2,3)+(2,5))+((2,3)+(2,5))
(4,9) = even × 3n
(5,5) = impossible
(5,6) = any × 6n
(5,7) = impossible
(5,8) = (2,3)+(2,5)+(3,8)
(5,9) = (5,9)
(6,6) = any × 6n
(6,7) = any × 6n
(6,8) = any × 6n
(6,9) = any × 6n
(7,7) = (7,7)
(7,8) = (4,7)+(4,7)
(7,9) = (2,9)+(5,9)
(8,8) = (2,3)+(2,5)+(6,8)
(8,9) = even × 3n
(9,9) = (4,9)+(5,9)

(4,10) = (4,7)+(4,3)
(5,11) = (5,2)+(5,9)
(5,13) = (5,2)+(5,2)+(5,9)
(7,11) = (7,4) + (7,7)

967 :132人目の素数さん:2021/06/11(金) 20:04:40.00 ID:q8bkQe7j.net
9×5
┏━┳━┳┳━┳━┓
┣┓┣┓┃┗┓┣┓┃
┃┗┫┗╋━╋┫┗┫
┣┳┻┳┫┏┫┗┳┫
┃┗┓┃┗┫┗┳┛┃
┗━┻┻━┻━┻━┛
7×7
┏┳━┳━┳━┓
┃┗┓┣┓┣┓┃
┣┳┻┫┗┫┗┫
┃┗┓┗┳╋━┫
┣━╋━┫┗┓┃
┣┓┣┓┣┳┻┫
┃┗┫┗┫┗┓┃
┗━┻━┻━┻┛
7×4
┏┳━┳━┳━┓
┃┗┓┃┏┻┓┃
┣┳┻╋┻┓┣┫
┃┗┓┗┓┣┛┃
┗━┻━┻┻━┛
7×2
┏┳━┳━┳━┓
┃┗┓┗┓┗┓┃
┗━┻━┻━┻┛
5×2
┏┳━┳━┓
┃┗┓┗┓┃
┗━┻━┻┛

968 :132人目の素数さん:2021/06/11(金) 21:42:21.92 ID:q8bkQe7j.net
訂正
(7,8) = (2,7) + (6,7)

ちなみに逆、すなわち(4,4),(5,5),(5,7)のとき不可能の証明はエレガントと呼べる解答は持ってない
使用するSブロックの位置でコツコツ場合わけする方法しか知らない
少なくとも(4,4),(5,5)で不可能は出題されてる内容に含まれてるので答えないと解答としては完成してない

969 :132人目の素数さん:2021/06/11(金) 21:58:22.65 ID:DBEyODvP.net
5×5の不可能性はこれの最後の図が面白い
4×4は単純そうで難しい…のか?
https://fushamath.com/wp-content/uploads/2020/06/2021_06_2.pdf

970 :132人目の素数さん:2021/06/11(金) 23:13:50.42 ID:q8bkQe7j.net
おお、やっぱりこういう解答があったか
5×7だと同じように塗って12枚必要になるから不可能と示せるな

971 :132人目の素数さん:2021/06/14(月) 20:59:31.06 ID:IKVeisyG.net
m×n の長方形の場合
タイルの総数 ≦ [mn/3]
奇数行目、奇数列目のマスの数 = [(m+1)/2][(n+1)/2]
∴ [(m+1)/2][(n+1)/2] ≦ [mn/3]
m,n とも奇数の場合は
 (m+1)(n+1)/4 ≦ mn/3,
 (m-3)(n-3) ≧ 12,
∴ m=3 または n=3 または (5,5) (5,7) (7,5) の場合は敷き詰め不可能。

972 :132人目の素数さん:2021/06/14(月) 21:16:24.45 ID:V4bBAB96.net
オレの答えと違うな
m×nの場合で(2)を同じ向きのSブロック2回までと拡張して

m×n (m≧n)が敷き詰め可能

n=2, m≠2,4 または
n=3, m:偶数 または
n=4, m≠4 または
n=5, m≠5, 7 または
n≧6

になった

973 :132人目の素数さん:2021/06/18(金) 19:27:49.77 ID:wKdBkQ65.net
 


エレガントで無くていいから


まず解けな



注目する点を間違ってるぞ
 




 

974 :132人目の素数さん:2021/06/18(金) 22:04:18.71 ID:yvLS2zTX.net
>>962
ラベルを利用する方法だと難易度は同じかな

975 :132人目の素数さん:2021/07/03(土) 12:56:19.96 ID:WjJQe4Jt.net
今月の2は超難問

976 :132人目の素数さん:2021/07/04(日) 17:24:25.46 ID:B6LaWuY8.net
でもないか
微分可能性はわからん

977 :132人目の素数さん:2021/07/04(日) 18:25:50.01 ID:Bako64+W.net
まぁ超有名問題やからな

978 :132人目の素数さん:2021/07/04(日) 19:15:51.75 ID:p6dwjwYD.net
>>977
微分可能性は簡単なん?

979 :132人目の素数さん:2021/07/04(日) 21:48:51.68 ID:cZ2S4pFd.net
まぁ件の対応がどうかけてるのか結果知ってれば

980 :132人目の素数さん:2021/07/04(日) 23:43:32.89 ID:B6LaWuY8.net
>>979
可微分性は後回し
出題1の2でつまる

981 :132人目の素数さん:2021/07/07(水) 00:09:32.90 ID:kh/fIGyx.net
>>980
出題1の2はなんとか

さて可微分はどすっぺ

982 :132人目の素数さん:2021/07/08(木) 01:56:41.44 ID:X2FQ/6Vg.net
>>979
これ自力で解ける人いるのかな

983 :132人目の素数さん:2021/07/10(土) 09:14:08.82 ID:ki6u/ufM.net
猿でも分かる回答求む

984 :132人目の素数さん:2021/07/10(土) 21:47:46.76 ID:fPqgbzq/.net
エレガントなやつでなくてもいいなら持ってるけど

985 :132人目の素数さん:2021/07/11(日) 09:12:10.77 ID:cEcS7Pdl.net
>>984
go ahead

986 :132人目の素数さん:2021/07/11(日) 22:12:01.41 ID:eAUQRtpO.net
とりあえず概略

区間[λ,ρ]に対して中点録Mnを
M0 = {λ,ρ},
M1 = {λ,λ#ρ,ρ},
Mn = {λ,λ#(λ#ρ),λ#ρ,(λ#ρ)#ρ,ρ},

とM0から始めて前のリストの隣接する2数の中点を追加して得られる有理数の集合を考える
あるMnの隣接する2点として現れる2つの有理数をIにおいてある時点で隣接すると呼ぶ
区間I=[b/a,d/c]が|ad-bc|=1の時よい区間と呼ぶ

命題 I=[q/p,s/r]、J=[b/a, d/c]がともによい区間の時
fIJ(x) = (b(rx -s) + d(px-q))/(a(rx-s)+c(px-q))
である
∵) 右辺をf(x)とおく
行列A = [[p,-q],[r,-s]], B=[[d,b],[c,a]]の引き起こす一次変換は格子点の間の全単射を引き起こす
また有理数v/uに対しBA[[v],[u]] = [[v'],[u']]の時f(v/u) = v'/u'であり、v/uが既約分数ならv'/u'も既約分数となる
よってv/u、t/sが既約分数でBA[[v],[u]] = [[v'],[u']]、BA[[t],[s]] = [[t'],[s']]の時BA[[v+t],[u+s]] = [[v'+t'],[u'+s']]でありv'/u'、t'/s'も既約分数で
f(v/u) # f(t/s)
= (v'+t')/(u'+s')
= f((v+t)/(u+s))
=f(v/u # t/s)
により主張が成り立つ

補題
I,Jを区間、λ,ρがある時点でIの隣接する2点でありfIJ(λ) = μ、fIJ(ρ) = νとするとき任意のx∈[λ,ρ]に対してfIJ(x) = f[λ,ρ][μ,ν](x)である
∵) 容易

補題
Jを区間でb/a∈Jが最大元でないとする
この時ある時点でb/aで右側に隣接するd/cが存在して|ad-bc|=1となる
∵)ある時点でb/aと右側で隣接するd/c全体の中で|ad-bc|が最小となるものをとり|ad-bc|>1と仮定する
ad-bcの素因子とすれば(a,b)と(c,d)はFp係数で平行だから正の数kを(c,d)+k(a,b)≡(0,0) (mod p)となるように取れる
d/cから始めてb/aとの中点を取る操作をk回行うと(bk+d)/(ak+c)でありg=(bk+d,ak+c)>0, c'=(ak+c)/g, d'=(bk+d)/gとおけばd'/c'はb/aと左側で隣接し|ad'-bc'|=|ad-bc|/gとなり矛盾する

987 :132人目の素数さん:2021/07/11(日) 22:12:09.88 ID:eAUQRtpO.net
主張
Iがよい区間、Jを一般の区間にとる
Iの点χが最大元でなければχと右側である時点で右側で隣接するξが存在してfIJは[χ,ρ]において一次分数関数である
同様の主張が最小元でない元の左側で成り立つ
∵) fIJ(χ)=b/aとしてb/aの右側である時点で隣接するd/cをとる
fIJ(ξ)=d/cとなるξをとればよい

主張
Iがよい区間、Jを一般の区間にとる
χ=v/uをIの内点にとりζ、ξ∈IをfIJが[ζ,χ],[χ,ξ]において一次分数関数となるようにとる
この時fIJのx=χにおける左側、右側微分係数は一致する
∵) fIJ = f とおく
I = [0,∞]、χ=1、ζ=0、ξ=∞として一般性を失わない
f(0) =b/a、f(1)=d/c、f(∞)=f/eとする
[0,1]においてf(x)=(b+(d-b)x)/(a+(c-a)x))、
[1∞]においてf(x)=(fx+(d-f))/(ex+(c-e))、
である
上の右辺をg(x)、下の右辺をh(x)とおくときg(1)=h(1)であるよって(log(g))'(1)=(log(h))'(1)を示せばよいが、ともに-/(cd)である

988 :132人目の素数さん:2021/07/12(月) 11:50:52.10 ID:SZPGMf+k.net
   


エレガントで無くていいから


まず解けな



注目する点を間違ってるぞ
 




 

989 :132人目の素数さん:2021/07/12(月) 19:01:12.36 ID:nQ2wYZke.net
エレガントな出題をほしいのだが

990 :132人目の素数さん:2021/07/12(月) 22:27:17.73 ID:/mMRlotm.net
>>986
そのfはどっから出てきたのかっていう

991 :132人目の素数さん:2021/07/12(月) 23:13:12.50 ID:rkg/Cex9.net
>>990
要するにこの問題はそこがミソ
有理数b/aを座標平面の格子点(a,b)に対応させて考える
すると1/2と3/1の中点4/3は(2,1)と(1,3)の和(3,4)を対応させていることになる
なので“中点を保存する写像”とは座標平面上では“和を保存する写像”となりすなわち“一時変換”ということになる
なので考えている区間がたとえば[0/1,1/0]から[2/3,3/4]なら(1,0)を(3,2)に、(0,1)を(4,3)に写す線形写像だから行列[[3,4],[2,3]]で表される写像で、y/xを(2x+3y)/(3x+4y)に写す写像だからf(t)=(2+3t)/(3+4t)と求まる
この“一次分数関数”を“行列”で考えるテクニックは超頻出テクニックで最もよく使われるテクニック
有理数近似の理論とか連分数の理論とかで頻出
時々受験数学レベルでも出てくる
ただしこのテクニックがうまく機能するには出てくる行列の行列式が±1出ないとダメで不通その前提条件で考えるけど、この問題はその前提外したらどうなりますかというお話
このテクニック知ってる人間ならその“出題者の意図”が透けて見えるので考えるべきポイントもすぐわかる
逆にコレ知らないと中々手でないかもね

992 :132人目の素数さん:2021/07/12(月) 23:47:57.17 ID:/mMRlotm.net
>>991
なんか納得いかないなw
LR表記がミスリードにしかならんよ

993 :132人目の素数さん:2021/07/12(月) 23:56:56.22 ID:rkg/Cex9.net
知らんがなw

994 :132人目の素数さん:2021/07/13(火) 00:03:33.18 ID:w61FTnjw.net
次スレ
http://rio2016.5ch.net/test/read.cgi/math/1626102020/

995 :132人目の素数さん:2021/07/17(土) 21:50:50.75 ID:kflQ1m7L.net
>>994
サンキュークリリン

996 :132人目の素数さん:2021/07/17(土) 23:34:14.28 ID:wdJtaXvI.net
こっちもういらねえか

997 :132人目の素数さん:2021/07/17(土) 23:34:20.39 ID:wdJtaXvI.net
うめ

998 :132人目の素数さん:2021/07/17(土) 23:34:28.01 ID:wdJtaXvI.net
うーめ

999 :132人目の素数さん:2021/07/17(土) 23:34:34.33 ID:wdJtaXvI.net
埋め立て

1000 :132人目の素数さん:2021/07/17(土) 23:34:40.53 ID:wdJtaXvI.net
完了

1001 :2ch.net投稿限界:Over 1000 Thread
2ch.netからのレス数が1000に到達しました。

総レス数 1001
331 KB
掲示板に戻る 全部 前100 次100 最新50
read.cgi ver.24052200